UWORLD Set #2

Pataasin ang iyong marka sa homework at exams ngayon gamit ang Quizwiz!

Chloramphenicol abx if taken during pregnancy may cause what?

"Gray baby" Syndrome -Gray skin -vomiting

Women being treated for metastatic breast cancer has no children or siblings. Another woman she had introduced as "my close friend," as been with her the whole time. They became distressed at the hospital after reading a sign that states only family members are permitted to stay after visiting hours. What is the best response by the physician?

"The two of you seem to have a very important relationship. Of course you may stay together."

A new chemotherapy regimen is tested in 100 patients with lung cancer. 40 patients are still alive after one years. Conventional chemotherapy is given to a separate group of 500 patients. and after one year 100 are alive. What is the absolute risk reduction?

(100-40)/100 = 0.6 (this number tells you the proportion of how many people died) (500-100)/500 = 0.8 0.8 - 0.6 = 0.2 ABSOULTE RISK REDUCTION

Patient presents in first few years of life with nausea/vomiting, seizures, liver dysfunction, hypoketotic, and hypoglycemic after prolonged fasting: What enzyme is deficient?

Acyl CoA Dehydrogenase Deficiency *suggests impaired BETA-OXIDATION (the Medium-chain Acyl CoA Dehydrogenase Deficiency is the most common genetic defect in beta oxidation)

Amyloid-beta (AlphaBeta) plaques that accumulates in Alzheimer Disease is an ABNORMAL fragment of what?

Amyloid Precursor Protein (APP)

Down Syndrome have an increased risk of EARLY-ONSET ALZHEIMER DISEASE, what is most likely to be increased in these patient?

Amyloid Precursor Protein (APP) *The APP gene is located on CHROMOSOME 21 which leads to extra accumulation

Alzheimer Disease involves accumulation of what extracellular?

Amyloid-beta (AlphaBeta) plaques

Spinal cord shows axonal degeneration involving Lateral Cortical Spinal Tract and Anterior Horn. What is the condition?

Amyotrophic Lateral Sclerosis

A patient hasn't eaten for 24 hours but despite this her blood glucose is 82. Maintenance of this patient's blood glucose levels is facilitated by hepatic conversion of PYRUVATE into GLUCOSE. What substance is an allosteric activator of the FIRST STEP of this process?

Acetyl-CoA *During gluconeogenesis substances such as LACTATE & ALANINE are converted to Pyruvate *Fatty Acid Oxidation enters into Gluconeogenesis at ACETYL-COA

Nicotinic Receptors stimulated by what neurotransmitter?

Acetylcholine

Esophageal aperistalsis: diagnosis?

Achalasia

Large head, mid-face hypoplasia, humeral and femoral shortening, shortened digits of hand, and space between 3rd and 4th digits: diagnosis?

Achondroplasia

When type of environment does Uric Acid precipitate?

Acidic environment

Microvilli that cover the intestinal epithelial cells: contain?

Actin thin filaments *DO NOT contain Microtubules

Teenager has persistent fever, sore throat, anterior and posterior cervical lymphadenopathy and splenomegaly. A blood smear shows an atypical lymphocyte. The atypical cell is most likely what?

Activated CD8+ CYTOTOXIC lymphocytes *Infectious Mononucleosis (EBV) *CD8+ Cytotoxic T-lymphocytes expand in response to EBV in effort to destroy the virally-infected cells. These reactive CD8+ cells may be seen on PERIPHERAL BLOOD SMEAR. (appear much larger than quiescent lymphocytes with abundant cytoplasm, an eccentrically-placed nucleus, and a cell membrane that appears to conform to borders of neighboring cells.

Cholinergic Agonists are often given for?

Activates BOWEL and BLADDER smooth muscle *Posteoperative ILEUS & URINARY RETENTION (nonobstructive)

Enteropeptidase: function?

Activates Trypsinogen --> TRYPSIN

Palpable neck lump, 'cold' thyroid nodule on thyroid scan. Spindle-shaped cells in an amorphous background, serum calcitonin level is elevated. What gene change is most likely responsible?

Activating mutation of RET proto-oncogene *RET is also commonly found in SPORADIC Medullary Thyroid Cancers *RET is the ONCOGENE that contributes to MEN 2A & MEN 2B

Calcitriol: what is this? what is it used for?

Active form of Vitamin D Postoperative supplementation after Thyroidectomy

Cytochrome P450 Inhibitors

Acute Alcohol Abuse Ritonavir Amiodarone Ciprofloxacin Cimetidine Ketoconazole Isoniazid (INh) Grapefruit Juice Quinidine Macrolides (except azithromycin) AAA Racks in GQ Magazine)

45 yo obese woman with fever and a prolonged episode of severe RUQ abdominal pain after a fatty meal: diagnosis?

Acute Cholecystitis

Man recently traveled to South America. Liver biopsy demonstrates Spotty hepatocyte necrosis and ballooning degeneration, eosinophilic apoptotic hepatocytes, and inflammatory cell infiltration: diagnosis?

Acute Hepatitis A

Fever, malaise, anorexia, nausea/vomiting followed by jaundice, pruritus, dark-colored urine, acholic stool.

Acute Hepatitis A *later symptoms due to CHOLESTASIS (JAUNDICE, PRURITUS, DARK-COLORED URINE) -dark-colored urine (increased conjugated bilirubin levels -acholic stool (pale poop)

How do Eosinophils contribute to host defense against parasites (helminths)?

Antibody-dependent cell mediated cytotoxicity *when stimulated by antibodies bound to a parasitic organism, they destroy the parasite via antibody-dependent cell-mediated cytotoxicity with enzymes from their cytoplasmic granules *Another function of eosinophils is REGULATION OF TYPE I hypersensitivity reactions.

Fever, dry skin and mucous membranes, flushing, blurred vision, altered mental status. Overdose of what?

Anticholinergic Intoxication -Jimson Weed (Datura Stramonium) -Atropine *HOT as a hare, DRY as bone, RED as a beet, BLIND as bat, MAD as a hatter.

Major Depressive Disorder with Psychotic Features: treatment?

Antidepressant & Antipsychotic or Electroconvulsive Therapy

MHC Class II peptides are displayed only on what cells?

Antigen Presenting Cells

Disregard & Violation of right of others?

Antisocial Personality Disorder

What adverse effect is likely to persists after adding Carbidopa to Levodopa treatment?

Anxiety & Agitation *Carbidopa makes more Levodopa available to the brain so ANXIETY & AGITATION can be increased because MORE DOPAMINE is available to the BRAIN.

At autopsy Myxomatous degeneration with pooling of proteoglycans in the media layer is observed in the arteries of these animals. This is associated with what condition?

Aortic Aneurysm *this is found in CYSTIC MEDIAL DEGENERATION (found in MARFAN SYNDROME)

Mammary Glands: what type of glands?

Apocrine

Acute Viral Hepatitis: biopsy shows mononuclear cell infiltrates, hepatocyte swelling and acidophilic bodies. The acidophilic bodies are the result of?

Apoptosis *these are Councilman Bodies or Apoptotic bodies

Phyllodes Tumors: histology

Arborizing (leaf-like) projections

Treacher Collins syndrome & Pierre Robin sequence are caused by defects in the formation of what pharyngeal arches?

Arch 1 & Arch 2

Lesser horn of hyoid: what pharyngeal arch?

Arch 2

Stylopharyngeus: what pharyngeal arch?

Arch 2

Greater horn of hyoid: what pharyngeal arch?

Arch 3

How can you determine the oral bioavailability when comparing it to the IV route?

Area under the ORAL CURVE divided by AREA under the IV CURVE (If they are equal doses) F = (area under oral curve x IV dose ) / (area under IV curve x oral dose)

Arginase Deficiency: Treatment?

Arginine-free, low protein diet

Normal Embryologic Rotation of the Gut?

Around 6 week gestation the midgut (supplied by the SMA) herniates through the umbilical ring in order to grow rapidly. During this process the midgut rotates 90 DEGREES COUNTERCLOCKWISE. After additional growth it returns to the abdominal cavity at 8-10 weeks gestation and turns an ADDITIONAL 180 DEGREES COUNTERCLOCKWISE. so 270 DEGREES TOTAL.

What is another major side effect of Levodopa/Carbidopa treatment?

Arrhythmias

What has a synergistic effect on the development of Lung Carcinoma?

Asbestos exposure Smoking **BAD COMBO!!

Urethritis caused by Gonorrhea: treatment?

Azithromycin Ceftriaxone

X-linked agammaglobulinemia (XLA): what cells are low or absent in these patients?

B CELLS in the peripheral blood and lymphoid tissues

Primary CNS Lymphoma arise from what cells?

B cells

Primary Central nervous system lymphoma associated with Ebstein-Barr Virus (EBV). Tissue biopsy from the frontal lobe lesions would likely predominately show an abnormal population of what cells?

B-lymphocytes *this is a type of DIFFUSE, LARGE-CELL NON-HODGKIN LYMPHOMA

Infectious Mononucleosis: EBV preferentially infects what cells? 2. How?

B-lymphocytes 2. through CD21 cell surface receptor

Cervical biopsy reveals Cervical Carcinoma. What is the most likely finding on histopathology?

BASEMENT MEMBRANE INVASION *Atypical glandular elements INVADING THE BASEMENT MEMBRANE or *Nests of atypical cells in the SUBEPITHELIAL STROMA

von Willebrand disease (vWD): BT, PT, PTT, PC?

BT: Increased PTT: Increase PT: Normal PC: Normal

Bartonella Henselae: can also cause?

Bacillary Angiomatosis (in immunocompromised patients)

What bacteria are found in soil and capable of surviving past boiling point of water, 100 C? 2. How?

Bacillus Clostridium 2. By forming SPORES *also survive dessiccation (dryness), and chemical agents by forming spores

B-BRAS

Bacitracin - gBs Resistant - gAs Sensitive *STREPTOCOCCI

Intracellular organisms

Bacteria -Chlamydia -Legionella -Listeria Monocytogenes (MOTILE) -Mycobacterium -Neisseria Meningitidis -Nocardia -Rickettsia -Salmonella Typhi Fungi -Cryptococcus Neoformans -Histoplasma Capsulatam -Pneumocystis Jirovecii Protozoa -Plasmodium -Toxoplasma Gondii

Chaotic relationship, sensitivity to abandonment, labile mood, impulsivity, inner emptiness, SELF-HARM?

Borderline Personality Disorder *Self-harm is KEY!!

Bordet-Gengou Medium is best for what organism?

Bordetella Pertusis

Regan-Lowe medium is best for what organism?

Bordetella Pertussa

Pseudomonas Aeruginosa: treatment

CAMPFIRE drus -Carbapenems -Aminoglycosides (Gentamycin) -Monobactams -Polymyxins (polymixin B, Colistin) -Fluoroquinolones (Ciprofloxacin, Levofloxacin) -thIRd & 4th Generation Cephalosporins (CEFTAZIDIME, CEFEPIME) -Extended-spectrum penicillins (eg, piperacillin, ticarcillin)

What type of tissue would favor the diagnosis of a Lung Hamartoma?

CARTILAGE tissue in the biopsy * Lung hamartomas often contain islands of mature Hyaline CARTILAGE, fat, smooth muscle lined by respiratory epithelium

Catalase positive organisms?

CATS Need PLACESS to Belch their Hairballs Nocardia Pseudomonas Listeria Asperigillus Candida E. Coli Staphylococcus Aureus SERRATIA Burkholderia Cepacia* H. Pylori

Generalized anxiety disorder: 1st line treatment, 2nd?

CBT, SSRis, SNRis are first line. Buspirone, TCAs, benzodiazepines are second line.

NK Cells express what markers?

CD 56 CD 16

X-linked agammaglobulinemia (XLA): flow cytometry of these patient's peripheral blood is most likely to show deficiency of cells bearing what markers?

CD19 CD20

Sarcoidosis: what cell type is most likely to predominate in their bronchoalveolar lavage?

CD4+ lymphocyte

Patient has elevated AST and ALT and is infected with Hepatitis B Virus (HBV): describe the mechanism of liver injury?

CD8+ T lymphocyte response to viral antigens on the cell surface resulting in hepatocyte damage

What exits in the Jugular Foramen?

CN IX CN X CN XI

What exits in the Internal Auditory Meatus?

CN VII CN VIII

Sternocleidomastoid: innervation?

CN XI (Accessory)

Trapezius: innervation?

CN XI (Accessory)

Cardiac Output equations?

CO = SV x HR CO = Oxygen consumption / (arteriovenous O2 differenence) *FICK EQUATION is the second one

Osteogenesis Imperfecta: gene mutated?

COL1A1

Osteogenesis Imperfecta (OI): mutations in what genes?

COL1A1 and COL1A2

Hemiballism: where is the injury?

CONTRALATERAL INJURY in or near the SUBTHALAMIC NUCLEUS

Eosinophilic globule in liver

COUNCILMAN body (viral hepatitis-> probably HEPATITIS A, yellow fever) *represents hepatocyte undergoing apoptosis

Pain and bluish discoloration of fingers and toes on cold exposure. Tightening of skin over fingers. Accentuated second heart sound over upper left sternal border. Diagnosis?

CREST Syndrome *ACCENTUATED second heart sound indicates elevated pulmonary artery pressure (pulmonary arterial hypertension) -Sclerodactyly -Raynaud Phenomenon

Acute Intermittent Porphyria: What may stimulate their symptoms?

CYP450 Inducers -Barbiturates -Antiepileptics -EtOH -Smoking -Rifampin -griseofulvin

Calcium dependent adhesion proteins?

Cadherins

CREST Syndrome characteristics?

Calcinosis Raynaud's Esophageal Dysmotility Sclerodactyly Telangiectasia

Citrate chelates what in the plasma?

Calcium & Magnesium

Amlodipine: what kind of drug?

Calcium Channel Blocker (Dihydropyridine) *works on vascular smooth muscle

Pseudogout is caused by accumulation of?

Calcium Pyrophosphate Crystals

Patient has received several units of packed RBCs and now has low serum calcium level. Why?

Calcium chelation by Citrate in packed RBC that were transfused *CITRATE can chelate serum calcium causing HYPOCALCEMIA and follows massive transfusions involving more than 5 or 6 liters of blood -Citrate is a substance added to stored blood

Primary Hyperparathyroidism serum levels: Calcium, PTH, Phosphorus?

Calcium: HIGH PTH: HIGH Phosphorus: DECREASE *Phosphorus LOW because PTH is HIGH and it trashes it

Guillain-Barre Syndrome: what bacteria?

Campylobacter Jejuni infection

Immunocompromised patient with Esophagitis, endoscopy shows white pseudomembrane: organism?

Candida

Pseudohyphae and Blastoconidia?

Candida Species *Blastoconidia are spores that grow as buds on the fungal hyphae

Cholinergic Agonist used to lower intraocular pressure in glaucoma?

Carbachol Pilocarpine

Patient has sudden episodes of severe, right-sided facial pain. like a "knife stabbing my face" Pain lasts several seconds, and is often instigated by meals or teeth brushing. Treatment?

Carbamazepine (drug of choice) *Trigeminal Neuralgia (Tic Douloureux) *V2 and V3 (CN V)

What is the treatment of choice for ESBL (Extended-Spectrum Beta-Lactamase) ORGANISMS?

Carbapenems (eg, imipenem)

What is the purpose in adding Carbidopa to Levodopa treatment in Parkinson Disease?

Carbidopa -> INHIBITS PERIPHERAL CONVERSION of LEVODOPA and makes it more available to the brain

PaO2: NORMAL SaO2: DECREASED Oxygen Content: DECREASED

Carbon Monoxide Poisoning *SaO2 would be detected as NORMAL using standard probes.

The action of what enzyme is mostly responsible for the Chloride content being lower in arterial blood than in venous blood?

Carbonic Anhydrase *this forms bicarbonate from CO2 and water. Many bicarbonate ions diffuse out of the RBC into the plasma. When Bicarbonate ions diffuse out of the RBCs then CHLORIDE moves INTO THE RBCS (CHLORIDE SHIFT)

Gastrointestinal (Colorectal) Tumors: what serum marker is the most useful for monitoring this patient?

Carcinoembryonic Antigen

Carcinomas or Sarcomas spread via lymphatics? 2. except?

Carcinomas 2. **Hepatocellular Carcinoma -Renal Cell Carcinoma -Follicular Thyroid Carcioma -Choriocarcinoma

Jugular Venous Tracing: y descent is ABSENT in what?

Cardiac Tamponade

Blood pressure is 190/100 and CT is shown with small hyperattenuated lesions on non contrast head CT. Diagnosis?

Charcot-Bouchard Aneurysm

Typically autosomal dominant inheritance pattern and associated with foot deformities (pes cavus), lower extremity weakness and sensory deficits.

Charcot-Marie-Tooth Disease [Hereditary Motor and Sensory Neuropathy (HMSN)]

Over the span of many months medication errors occur on a hospital unit. Some resulting in significant errors. What is the best next step in addressing the problem?

*CONDUCT STEP-BY-STEP ANALYSIS of what occurred and why (ROOT CAUSE ANALYSIS --> this aims to identify what, how, and why an undesirable outcome occurred) *question example says INTERVIEW STAFF MEMBERS

Lepromatous Leprosy: treatment?

*Clofazimine (added) Dapsone Rifampin

Patient has sharp chest pain in the left side of the sternum. Patient has had episodic pain with deep inspiration or trunk movement. No fever or cough. Lung are clear and cardiac examination reveals normal heart sounds without gallops or murmurs. Diagnosis?

Chest wall/musculoskeletal issue *Costochondritis *worse with MOVEMENT

Checks differences between 2 or more percentages or proportions of categorical outcomes: what test is used?

Chi-square test

What test is used for categorical outcomes when a label is not given?

Chi-square test Chi-tegorical

What diagnostic test results would be most specific for Acute Cholecystitis?

*Ultrasound signs of GALLBLADDER INFLAMMATION (Wall Thickening, pericholecystic fluid) *Failed gallbladder visualization on radionuclide biliary scan

Downward displacement of elongated cerebellar tonsils through foramen magnum and into the upper cervical canal?

Chiari I Malformation

Lymphogranuloma Venereum Disease: organism?

Chlamydia Trachomatis (L1-L3)

Bipolar Disorder: 1st line medications for maintenance?

*VALPROATE (missed this) LITHIUM -I chose Buproprion (NEVER use ANTIDEPRESSANT MONOTHERAPY due to risk of MOOD DESTABILIZATION)

Hyperpolarization is a result of what membrane permeability?

*VOLTAGE-gated K+ channels remain open for a short time after repolarizatoin is completed -when the VOLTAGE-GATED K+ channesl close, the membrane potential returns to the resting value maintained by the NON-GATED K+ channels

Angiogenesis is predominantly driven by what 2 substances?

*Vascular Endothelial growth factor (VEGF) *Fibroblast growth factor (FGF) -Angiogenesis (blood vessel formation)

Tamoxifen & Raloxifene adverse effects?

*Venous THROMBOEMBOLISM**** -Hot Flashes

Lepromatous Leprosy: presents with?

*focally Decreased sensation *Lion-like facies (diffusely over the skin)

Nonpainful penis lesion, groin pain, and swelling. The inguinal swelling are now draining ulcers. Scrapings from his lesions show cytoplasmic inclusion bodies: what is the organism?

Chlamydia Trachomatis (L1-L3) *painless penis ulcers *painful inguinal lymphadenopathy ("buboes") and ULCERATION *INCLUSION BODIES

Pulmonary infection in an alcoholic with foul smelling sputum and lung abscess. Treatment?

Clindamycin *covers Anaerobic oral flora (Bacteroides, Prevotella, Fusobacterium, Peptostreptococcus) *Clindamycin could also cover the Streptococcus Pneumoniae (Most common cause of community-acquired pneumonia)

Bacterial Vaginosis (Gardnerella Vaginalis): treatment?

Clindamycin Metronidazole

Severe jaundice in neonate with muscle rigidity lethargy and seizures.

Crigler-Najjar syndrome *the unconjugated bilirubin is gradually deposited in various tissues, including the brain. Theses deposits can cause KERNICTERUS (bilirubin encephalopathy) -> characterized by SEVERE JAUNDICE and NEUROLOGIC impairment

Abdominal pain, diarrhea, aphthous ulcers of mouth (possibly), abscesses. Diagnosis?

Crohn Disease *ABDOMINAL PAIN, DIARRHEA, and LOW-GRADE FEVER *perianal disease (fissures, abscesses may also occur but do not have to)

Ulcerative Colitis or Crohns is a Granulomatous Disease?

Crohn's Disease *NONCASEATING granulomas

ACE-I: how do these cause potassium retention?

DECREASED ALDOSTERONE causes INCREASED POTASSIUM RETENTION

Alcoholic induced hepatic steatosis appears is most likely caused by?

DECREASED Free Fatty Acid Oxidation which is secondary to EXCESS NADH PRODUCTION by the 2 major alcohol metabolism enzymes (ALCOHOL DEHYDROGENASE & ALDEHYDE DEHYRDOGENASE)

Paget's Disease: new bone characteristics?

*new bone is disorganized and lesions appear on imaging as thickened areas of mixed sclerosis and lucency

Depolarization is a result of what membrane permeability?

*opening of VOLTAGE-gated Na+ channels -rapid influx of Na+ into cell

Gardos Channel Blockers: MOA?

*prevents dehydration of RBC and reduces the polymerization of HbS -hinder the efflux of potassium and water from the cell

Neuroleptic Malignant Syndrome (NMS) from Typical Antipsychotics: treatment?

-Dantrolene (inhibition of calcium release from sarcoplasmic reticulum of skeletal muscle) -Bromocriptine (Dopamine Agonist)

Schistocytes are seen on blood smear. What laboratory abnormalities are expected?

-Decreased serum haptoglobin -Increased Serum lactate dehydrogenase (LDH) -Increased in serum indirect bilirubin -Increased free hemoglobin in serum and urine

Isolated Mitral Stenosis findings?

-ELEVATED LA diastolic pressure (elevated PCWP) -Pulmonary HTN -DECREASED pulmonary vascular compliance -RV Dilatation -possibly functional Tricuspid Regurgitation *Diastolic pressure in the LV is usually near normal or even decreased with severe mitral stenosis

Measles Virus (Rubeola): neurologic complications?

-Encephalitis (within days) -Acute Disseminated Encephalitis (within weeks) -Subacute Sclerosing Panencephalitis (within years)

Biphosphonates: adverse effects?

-Esophagitis -Osteonecrosis of Jaw -atypical stress fractures

Severe Combined Immune Deficiency: explain the defect causing the symptoms?

-Gene defect leads to FAILURE of T CELL DEVELOPMENT (absence Thymic shadow) -B CELL DYSFUNCTION due to ABSENT T CELLS

Acute Intermittent Porphyria: treatment?

-Glucose -Heme

Acute respiratory distress syndrome (ARDS) is characterized by the development of what?

-Hypoxemia -Bilateral Pulmonary infiltrates (in absence of fluid overload or HF)

Listeria Monocytogenes: often infects what patients?

-IMMUNOCOMPROMISED -Pregnant women -newborns

Acute respiratory distress syndrome (ARDS): explain the pathogenesis what the injury to the pneumocytes and endothelium has occurred?

-INCREASED Pulmonary Capillary permeability (fluid enters the alveoli) -DECREASED surfactant production (alveoli collapse) -INSPISSATION of protein-rich fluid & necrotic debris *HYALINE FORMATION)

Stimulation of Alpha-1 receptors causes what

-INCREASED SVR: INCREASED SBP (Vasoconstriction) -DECREASED Renal Blood Flow

Biliary Atresia: what will biopsy show?

-Intrahepatic bile duct proliferation -portal tract edema -fibrosis

Nitroglycerin effects: LVEDP Peripheral venous capacitance Systemic Vascular Resistance

-LVEDP: DECREASED -Peripheral venous capacitance: INCREASED -Systemic Vascular Resistance: DECREASED *Nitrates --> PRIMARILY VENODILATORS but have a modest effect on arteriolar dilation which DECREASES SVR

Autopsy shows INCREASED thickness of the left ventricular wall and DECREASED left ventricular cavity size. The structural changes are most likely associated with what condition?

-Longstanding Hypertension -Aortic Stenosis

Pseudomonas Aeruginosa: bacteriology?

-MOTILE AEROBIC gram-negative rod -NONLACTOSE-fermenting -OXIDASE-POSITIVE

Silicosis impairs the function of what?

-Macrophages (the PHAGOLYSOSOMES may be disrupted by the internalized silica particles) *increases susceptibility to TB

TUBEROUS SCLEROSIS: what neurologic conditions?

-Mental retardation -Seizures

Thalassemia: what is seen on blood smear?

-Microcytosis -Hypochromic RBC -Target Cells (occassionally)

Brand-new Infant has bilious emesis what is the differential diagnosis?

-Midgut Volvulus -Intestinal Stenosis -Atresia

Malabsorption: DECREASED D-Xylose absorption: what is the cause?

-Mucosal Defect -Bacterial Overgrowth

Organophosphate poisoning stimulates what receptors?

-Muscarinic Receptors -Nicotinic Cholinergic Recptors

Osteomyelitis: Pott Disease?

-Mycobacterium Tuberculosis

Listeria Monocytogenes: causes what disease?

-NEONATAL MENINGITIS -MENINGITIS (immunocompromised) -mild gastroenteritis (healthy individuals) -Amnionitis -Septicemia -spontaneous abortion -granulomatosis infantiseptica

Cyanide Poisoning: treatment

-NITRITIES followed by THIOSULFATE -HYDROXYCOBALAMIN (Vitamin B12 precursor) *NITRITIES switches iron to Fe3+ and diminished cyanide's toxic effect

Medical treatment for Endometriosis begins with?

-NSAIDS (to reduce pelvic pain) -Hormonal contraceptives (control stimulation of endometriosis implants) *if these medications do not offer relief, a laparoscopy is performed for definitive diagnosis as well as treatment

INCREASED Maternal Serum AFP?

-Open neural tube defects -ventral wall defects (omphalocele, gastroschisis) -multiple gestation

Ischemic Colitis: colonoscopy reveals what?

-Pale Mucosa -Petechial Hemorrhages

What two tissues produce a bicarbonate-rich fluid in response to a stimulation?

-Pancreas -Biliary Tract (epithelial cells) *Salivary Gland (does not have a stimulus)

Damage to the Orbital Floor could result in what?

-Parasthesia of the upper cheek, upper lip, upper gingiva -Limited SUPERIOR GAZE (Inferior Rectus Muscle may become entrapped)

Newborns of all mothers with active hepatitis B are given what?

-Passive immunity (Hepatitis immune globulin - HBIG) -Active Immunization (HBV Vaccine)

Vincristine: adverse effect?

-Peripheral Neuropathy (usually with overdose)

Osteomyelitis: Diabetes Mellitus?

-Polymicrobial

Osteomyelitis: recent trauma or orthopedic surgery?

-Polymicrobial

Osteomyelitis: Recumbent patients with impaired mobility?

-Polymicrobial *often on Sacrum & heels

Why do newborns have low vitamin K stores?

-Poor transplacental transfer -low content in breast milk

Nuclear iodine has been released into the environment: what do you give your patients to prevent tissue damage in these individuals?

-Potassium Iodide -Perchlorate -Pertechnetate *Potassium Iodide competitively inhibits

Bleomycin: adverse effect?

-Pulmonary Fibrosis -Flagellate Skin discoloration

X-linked agammaglobulinemia (XLA): Increased risk of what type of infections?

-Pyogenic (ENCAPSULATED) BACTERIA -certain viral & parasitic infections (ENTEROVIRUS & GIARDIA LAMBLIA)

Hepatic Encephalopathy: Treatment?

-RIFAXIMIN (DECREASES intraluminal ammonia production) -LACTULOSE (INCREASES conversion of ammonia to ammonium)

Osteomyelitis: Sickle cell disease?

-Salmonella -Staphylococcus Aureus

Kartagener Syndrome triad?

-Situs Inversus -Chronic Sinusitis -Bronchiectasis

Nicotinic Receptors?

-Skeletal Muscles (Nm) -Adrenal Medulla (Nn) -Autonomic Ganglia (Nn)

62 yo with vague feeling of heaviness in her legs, she has osteoarthritis, GERD, works as a cashier at a local department store for the last 20 years. Physical exam shows dilated tortuous superficial veins in her lower legs. Her condition is most likely to be complicated by which of the following?

-Skin Ulcerations -painful thromboses -stasis dermatitis -poor wound healing -superficial infections *Varicose Veins

Premature Ovarian Failure: factors?

-Smoking -Radiation or chemotherapy *Premature atresia of ovarian follicles in women of reproductive age. Patients present with signs of menopause after puberty but before age 40.

New long-acting medication that causes selective direct relaxation of the smooth muscle of arterioles but does not affect the veins. What adverse effects is most likely to be caused by the drug during the trial?

-Sodium and fluid retention and Reflex Tachycardia (sympathetic activation) *stimulation of baroreceptors (sympathetic activation) *stimulation of RAAS (sodium and fluid retention)

Osteomyelitis: Prosthetic Joint Replacement?

-Staphylococcus Aureus -Streptococcus Epidermidis

Vasa Vasorum Endarteritis: associated with?

-Thoracic Aortic ANEURYSM *Syphilis (aka Endarteritis Obliterans)

Minimal Change Disease: inciting events?

-Upper Respiratory Infection -Immunization -Insect Bite (Bee Sting in the case example)

Uric Acid Stones: Treatment?

-Urine Alkalinazation -Allopurinol (Xanthine Oxidase Inhibitor) -Hydration

Mutation in deltaF508 causes what to happen?

-abnormal protein FOLDING -failure of GLYCOSYLATION *this causes the CFTR protein to be DEGRADED before it reaches the cell surface and its COMPLETE ABSENCE from the membrane *Cystic Fibrosis

Acute respiratory distress syndrome (ARDS) is caused by injury and inflammation of what?

-alveolar PNEUMOCYTES -pulmonary ENDOTHELIUM

Political refugee from East Africa has large left-sided tumor on his jaw. Biopsy shows numerous mitotic figures and apoptotic bodies. What genetic feature is likely to be seen?

-c-Myc oncogene overexpression *Burkitt Lymphoma *"Starry Sky" appearance -> due to presence of macrophages and apoptotic bodies in a sea of medium-sized lymphocytes

Achalasia manometry shows?

-decreased amplitude of peristalsis in mid esophagus -increased tone & incomplete relaxation at LES

Facial Nerve Palsy: other symptoms patients may have besides lose of motor function to facial muscles?

-decreased tearing (lacrimal gland) -decreased salivation (submandibular, sublingual glands) -loss of taste sensation over the anterior 2/3 of tongue -Hyperacusis (increased sensitivity to certain frequency and volume ranges of sound)

Severed Combined Immunodeficiency: Types?

-defective IL-2R Gamma Chain (x-linked) -Adenosine Deaminase Deficiency (autosomal recessive)

Peripheral Artery Disease: Treatment?

-graded exercise program *CILOSTAZOL *antiplatelet agent (aspirin or clopidogrel)

Superior Mesenteric Artery Syndrome: risk factors?

-losing weight really fast -pronounced Lordosis -surgical correction of scoliosis

What conditions cause sickling of HbS patients?

-low oxygen -increased acidity -dehydration

Cytoplasmic P bodies play an important role in what?

-mRNA translation regulation & -mRNA degradation

Cocaine is also a potent vasoconstrictor that can cause what?

-myocardial Ischemia -atrophy of Nasal mucosa & Septum

Cystic Fibrosis: the NORMAL transmembrane protein functions to do what?

-pumps CHLORIDE IONS OUT of epithelial cells against a concentration gradient using ATP hydrolysis for energy. *also DRAWS SODIUM & WATER ACROSS THE MEMBRANE AS WELL --> HYDRATES mucosal surfaces like airways and bowels this way.

Insomnia treatment: Sleep hygiene?

-regular sleep schedule -avoid naps -avoid caffeine after lunch -avoid alcohol & smoking near bedtime -avoid large meals near bedtime -adjust bedroom environment to be quiet dark and cool -exercise regularly but right before bedtime

Fibroadenoma: characteristics?

-well-demarcated -painless -mobile -spherical -1-10 cm -multiple and or bilateral lesions ***often INCREASE IN SIZE during PREGNANCY, LACTATION or with ESTROGEN THERAPY and usually REGRESS AFTER MENOPAUSE

When do you see this after an MI: Normal Myocardium?

0-4 hours *remember only the FIRST 4 HOURS there is NO VISIBLE CHANGE!!

The probably that the observed difference is due to chance alone is reported to be 5%. There is also a 20% probability of concluding that there is no difference in blood cholesterol level when there is one in reality. What is the POWER of the study?

0.80 Power = 1 - Beta Power = 1 - 0.20 = 0.80

What is the cause of hypercalcemia in Lymphomas (all types)?

1,25 (OH)2 Vitamin D production (causes gut absorption of calcium)

When do you see this after an MI: Dense interstitial neutrophil infiltrated?

1-3 days *the neutrophils recede around days 5-7

Patient has a knife injury to back and undergoes right nephrectomy. No PMH. What best describes his glomerular filtration rate (GFR) compared to baselines: 1. Immediately after surgery AND 2. 6 weeks after surgery?

1. 50% 2. 80%

Acute Intermittent Porphyria: 5 Key signs!

1. Abdominal Pain 2. Neurological Manifestations 3. No photosensitivity 4. Port-wine colored urine (may darken on standing) 5. Porphobilinogen & Aminolevulinic Acid in urine

Separation of Epidermal Cells: what term? 2. What disease?

1. Acantholysis 2. Pemphigus Vulgaris

Patient has PMH for tuberculosis. He presents with episode of hemoptysis this morning. CT shows an old left upper lobe cavity. Compared to prior scans, the main difference is that the cavity now appears to be filled with a round mass. What organism? 2. Is this considered Contagious or Colonizing?

1. Aspergillus Fumigatus *(Aspergillomas) --> formed in pre-existing lung cavities especially after TB INFECTION 2. Colonizing *Aspergillomas are NOT CONTAGIOUS

Antiarrhythmic medication that causes QRS prolongation & INCREASES QT interval duration. 1. What Class of drug? 2. What are the drug names?

1. Class IA 2. Quinidine, Procainamide Disopyramide "the Queen Proclaims Diso's PYRAMID"

Very painful, red eyer, sudden vision loss, halos around lights, rock-hard eye, frontal headache. 1. Diagnosis?

1. Closed-Angle Glaucoma (Narrow-angle Glaucoma)

Elderly patients either postoperative, intubated or dehydrated develop a fever with right jaw pain. PE shows firm swelling of the angle of the mandible. 1. What do you use to diagnose?

1. Elevated Serum Amylase with normal Lipase & imaging (CT/ultrasound *Acute Parotitis

Patient ate fish, now complains of weakness, dizziness and paresthesias involving his face and extremities. Neurological exam shows reduced muscle strength and decreased deep-tendon reflexes. 1.What part of the action potential curve is impaired? 2. What is the source? 3. Toxin?

1. Fast voltage gated Na+ channels are BLOCKED --> PREVENTING DEPOLARIZATION 2. PUFFERFISH or Red Tide 3. TETRODOTOXIN or Saxitoxin

Clostridium Difficile patient is placed on an oral macrocyclic antibioitic that has bacteriocidal activity and minimal systemic absorption.

1. Fidaxomicin *Macrocylc antibiotic (related to macrolides) --> inhibits the subunit of RNA polymerase leading to protein synthesis impairment and cell death.

In the picture what Drug Metabolism order is represented in #1? #3?

1. First-order 2. Zero-Order

Rapidly Progressive Glomerulopnehpritis (RPGN): what are the 3 types?

1. GOODPASTURE SYNDROME (Type II Hypersensitivity) *LINEAR IF PAUCI-IMMUNE Rapidly Progressive Glomerulopnehpritis (RPGN) -Granulomatosis with Polyangiitis (Wegener) -Microscopic Polyangiitis *ANCA

Most common causes of Acute Pancreatitis? 1st & 2nd?

1. Gallstones 2. Alcohol

Patient has fatigue, rash, flushing, abdominal cramps. Rash is worse with rubbing or scratching and diffuse itching after hot showers. Patient is frequently dizzy and light-headed after prolonged standing and had an episode of syncope while working in hot sun. Skin examination shows maculopapuplar rash. Large clusters of mast cells are positive for KIT (CD 117). What is going on in the stomach?

1. Gastric Hypersecretion (Systemic Mastocytosis) *Mast cell proliferation is associated with mutation sin KIT and excessive HISTAMINE release from DEGRANULATION OF MAST CELLS mediates the symptoms like FLUSHING, HYPOTENSION, PRURITUS, URTICARIA. In addition, histamine induces gastric acid secretion, which can lead to GASTRIC ULCERATION. Excess acid may also INACTIVATE PANCREATIC INTESTINAL ENZYMES --> causing DIARRHEA.

1 week after liver transplantation patient develops nausea, vomiting, abdominal pain, and bloody diarrhea. Patient also has maculopapular rash on his neck, back and extremities. Multiple ulcerations are seen in the intestinal mucosa. 1. What cells are involved in the current condition?

1. Graft T cell sensitization against host MHC antigens 2. Graft-versus-host disease *Type IV Hypersensitivity

Gastric Parietal Cells secrete hydrochloric Acid (HCl) in response to 3 major stimulants: what are they?

1. HISTAMINE -> release from ECL cells and binds to H2 receptors on basolateral parietal cell membrane 2. VAGAL STIMULATION --> causes ACETYLCHOLINE release, which directly stimulates HCl secretion by binding to parietal cell MUSCARINIC M3 receptors. VAGAL STIMULATION also promotes HCl secretion via GASTRIN-RELEASING PEPTIDE (GRP), which stimulates GASTRIN release from G CELLS 3. G cells also release GASTRIN in response to PROTEIN meals. Gastrin binds to CCK receptor on ECL cells (promotes HISTAMINE release)

Thermal Cycling PCR steps?

1. Heating for DNA strand denaturing 2. Cooling for Primer hybridization 3. Rewarming for PRIMER EXTENSION & DNA SYNTHESIS

Gastric Acid secretion by parietal cells in the fundus and body of the stomach is stimulated by what 3 things?

1. Histamine 2. Acetylcholine 3. Gastrin

Patient ate fish, now they have acute burning sensation of mouth, flushing of facet, erythema, urticaria, itching, bronchospasm, angioedema, hypotension. 1. What is the action of the toxin? 2. What is the source? 3. Toxin?

1. Histidine converted to HISTAMINE (by Histidine Decarboxylase) 2. Spoiled dark-meat fish (TUNA, MAHI-MAHI, MACKEREL, BONITO) 3. Histamine (Scombroid poisoning)

When should Denial be confronted?

1. If it IMPAIRS RELATIONSHIPS 2. If it INTERFERES with PATIENT CARE

ELISA steps?

1. Known ANTIGEN IS FIXED TO SURFACE and patient serum is added. plate is washed to remove unbound patient antibodies. 2. Anti-human IMMUNOGLOBULIN ANTIBODY COUPLED TO PEROXIDASE (or another enzyme) is added. These bind to antigen bound antibodies in the wells. Plate is washed again. 3. A Substrate which is modified by the Peroxidase (or enzyme) to ELICIT A DETECTABLE SIGNAL (like COLOR) is ADDED. If the assay is quantitative, the intensity of color change is proportional to the amount of antibody present in serum.

Patient appears schizophrenic gets medication in the ED and the next day is confused, with a temperature of 105 F and muscle rigidity. 1. What is this called?

1. Neuroleptic Malignant Syndrome

Patient ate fish, now they are confused, disoriented, BP 110/70, pulse 50. Pupils are symmetric, 2 mm, and reactive to light. Eyes are tearing considerably. Scattered wheezes bilaterally on lung auscultation. Skin is clammy and he is sweating profusely. 1.What part of the action potential curve is impaired? 2. What is the source? 3. Toxin?

1. Opens Na+ channels --> CAUSING DEPOLARIZATION 2. Reel Fish (Barracuda, snapper, and moray eel) or South American Frogs 3. Ciguatoxin or Batrachotoxin

Increased thickness of Stratum Corneum with RETENTION of nuclei in Stratum Corneum: what term? 2. What disease?

1. Parakeratosis *Psoriasis

Corynebacterium Diphtheriae: Treatment?

1. Passive Immunization (Diphtheria Antitoxin) 2. Penicillin or Erythromycin 3. Active Immunization (DPT Vaccine)

Antiarrhythmic medication that causes QRS prolongation but has little effect on QT interval duration. 1. What part of the action potential curve is most likely affect by this drug? 2. What Class of drug? 3. What are the drug names?

1. Phase 0 of the action potential 2. Class IC 3. Flecainide & Propafenone "CAN i have more FRIES PLEASE"

Childhood brain tumors: top 3 most frequent?

1. Pilocytic Astrocytoma (low-grade) 2. Medulloblastoma 3. Ependymoma

Major clinical manifestations of Asbestos exposure (most frequent first)(4)?

1. Pleural Plaques 2. Asbestosis (pulmonary fibrosis *asbestos bodies) 3. Bronchogenic Carcinoma 4. Malignant Mesothelioma

RUBELLA during PREGNANCY: What is a possible sequelae for the MOTHER?

1. Polyarthritis & Polyarthralgia

What are the initial steps of gluconeogenesis?

1. Pyruvate --> Oxaloacetate (Pyruvate Carboxylase) 2. Oxaloacetate --> Phosphoenolpyruvate (Phosphoenolpyruvate Carboxykinase)

Vitamine B1 (Thiamine): participates in glucose metabolism and is a cofactor for what enzymse?

1. Pyruvate Dehydrogenase 2. alpha-ketoglutarate Dehydrogenase 3. Branched-chain alpha-ketoacid Dehydrogenase 4. Transketolase

Rabies vaccination: postexposure treatment?

1. Rabies Immune Globulin 2. Vaccination (Inactivated Vaccine)

Hemodynamics of severe aortic stenosis?

1. Severe stenosis impairs LV output 2. higher systolic pressure causes LV HYPERTROPHY 3. Atrial contraction necessary for filling of stiffened LV

The integrity of the intestinal mucosa depends on complete and rapid neutralization of HCl in gastric contents. This is accomplished alkaline secretions from 2 primary sources. What are they?

1. Submucosal (Brunner) Glands (these are most numerous at PYLOROUS) 2. epithelial cells of the pancreatic ductules (emptied into the duodenum at the AMPULLA)

Laparotomy is performed in a brand-new infant with bilious emesis and the duodenum ins normal-appearing, and theres is absence of a large segment of jejunum and ileum. What intrauterine process is most likely responsible for this patient's condition?

1. Vascular Occlusion in utero (Diminished intestinal perfusion leads to ischemia of a segment of bowel, with subsequent narrowing or stenosis or obliteration - atresia - of the lumen.

Reactive Arthritis: 4 key signs to watch for?

1. can't pee (Urethritis) 2. can't see (Conjunctivitis) 3. can't bend my knee (large joints) 4. Keratoderma Blennorrhagicum

Patient with Infective Endocarditis has blood culture of Streptococcus species: what process was the most likely initiating step?

1. disruption of normal endocardial surface 2. STERILE FIBIRN-PLATELET formation on this surface

SLE: most specific antibodies 1st and 2nd?

1. dsDNA antibodies 2. Anti-Smith antibodies (ab against small nuclear ribonucleoproteins --> anti-snRNPs) *both are SPECIFIC --> However if they are absent you can't rule out SLE yet.

Metabolism of glucose through the Hexose Monophosphate (HMP) shunt serves two major function: what are they?

1. production of NADPH as a reducing equivalent 2. synthesis of RIBOSE 5-PHOSPHATE for NUCLEOTIDE SYNTHESIS

A patient undergoes a bone biopsy and the pathologist identifies NUMEROUS MULTI NUCLEATED CELLS some containing over 100 nuclei. What factors are essential for the differentiation of the cells described?

1. receptor activator of nuclear factor kappa-B Ligand (RANK-L)** 2. Macrophage Colony-stimulating factor (M-CSF)**

Duodenal acidity triggers what?

1. release of BILE 2. release of BICARBONATE-rich pancreatic fluid (S Cells secreted SECRETIN which stimulates Bicarb release)

Fibrates (Fenofibrate, Gemfibrozil): 3 actions?

1. upregulates Lipoprotein Lipase (***DECREASES TRIGLYCERIDES***) 2. inhibits Cholesterol 7alpha-hydroxylase (increases Cholesterol excretion) 3. activates PPAR-alpha to induce HDL synthesis

z score for 95% of the distribution?

1.96

Duchenne Muscular Dystrophy: family has two sons and older son has DMD. What is the chance that the younger son will have DMD? *Inheritance?

1/2 *X-Linked Recessive

Granulation tissue and neovascularization post MI: when does this occur?

10 - 14 days!!

When do you see this after an MI: Granulation tissue with neovascularization?

10 - 14 days!!

When do you see this after an MI: Hypereosinonphilic myocytes with pyknotic (shrunken) nuclei?

12-24 HOURS *remember this is EARLY MI!

Class 1 antiarrhythmics, sodium-channel-binding strength is what?

1C > 1A > 1 B

Physical exam reveals a large deep wound directed posteriorly adjacent to the LEFT MIDCLAVICULAR LINE in the FOURTH intercostal space. What would be injured?

1st --> LEFT LUNG 2nd --> LEFT VENTRICLE

Long-term therapy with CONTINUOUS LEUPROLIDE: what would be the changes in testosterone and dihydrotestosterone after initiation of therapy through the progression of therapy?

1st LEUPROLIDE (GnRH) causes TRANSIENT INCREASE in pituitary LH SECRETION and in BOTH testosterone and DHT. -however, the CONTINUOUS use of leuprolide will begin to suppress LH release and leads to an eventual DECREASE in TESTOSTERONE PRODUCTION

Diazepam should not be used with what Antihistiamines?

1st generation Antihistamines -Diphenhydramine (Benadryl) -Chlorpheniramine -Promethazine -Hydroxyzine

ALL BENZODIAZEPINES should be excluded from used in conjunction with what?

1st generation Antihistamines Alcohol Barbiturates Neuroleptics *because they cause SEDATION and Benzos cause SEDATION!

Sustained arrhythmias after a STEMI: 1st line and 2nd line medications?

1st line: Amiodarone 2nd line: Lidocaine

Status Epilepticus: Treatment?

1st step: Lorazepam IV 2nd step: Phenytoin (simultaneously load w/) -if still seizing --> start Phenobarbital -if still seizing --> intubate and give general anesthesia

When do you see this after an MI: Dense Collagen Scar?

2 weeks - 2 months *

z score for 99% of the distribution?

2.58

How early can Fatty Streaks be seen?

2nd Decade of Life

When is Hemoglobin F predominant?

2nd and 3rd trimester & FIRST few months after birth

Acute Stress Disorder: time?

3 days to 1 month vs Post Traumatic Stress Disorder (PTSD)

When do you see this after an MI: Macrophage infiltration?

3-10 days *NOT PAST 10 DAYS!!

Marijuana: how long can it bet detected in the urine after daily use as ceased?

30 days

Women with the FGFR2 mutation are followed for 5 years 30 women develop breast cancer and 65 do not. The 2nd group does no have the mutation and 20 women develop breast cancer and 75 do not. What is the relative risk of breast cancer.

30/(30+65) divided by 20/(20+75) 30/20 = 1.5

Failure of neural tube closure occurs at what week?

4 weeks of gestation

When do you see this after an MI: Edema and wavy myocyte fibers with normal-appearing nuclei?

4-12 HOURS

After nephrectomy in a kidney donor, how long to get back to 80% GFR?

4-6 weeks.

High Beta-hCG level. Ultrasound shows no fetus and a uterine cavity filled with multiple small cysts. Most likely karyotype?

46 XX *this contains ONLY PATERNAL DNA (the paternal DNA is duplicated) -less common is the 46 XY because 2 sperm have to fertilize an empty ovum

Ovotesticular Disroder of Sex Development: genetics?

46 XX (most common) 46 XX > 46 XY

Klinefelter genetics?

46 XXY

Gluten enteropathy is associated with Dermatitis Herpetiformis. This skin disorder is most common in what age?

4th and 5th decades *uncommon in children

Atopic Dermatitis (Eczema): most patients present by what age?

5 years of age

DNA synthesis can occur only in what direction?

5' --> 3' direction

Nausea associated with visceral nausea due to GI insults (gastroenteritis, chemotherapy, and general anesthesia): medication targeting what receptors?

5-HT3 Receptors

The right renal artery FLOW is decreased by a factor of 16 compared to the left. By what percentage has the radius of the lumen been reduced?

50%

Celiac Disease: what age does it classically present?

6-24 months

Within 1 week of nephrectomy in a kidney donor, the GFR in the remaining kidney has already increased to what?

65-70%

What ribs overlie the Liver on the Right Posterior surface?

8th - 11th

Post Traumatic Stress Disorder (PTSD): time?

> 1 month

MAJOR DEPRESSIVE criteria?

> 2 weeks & 5 or more of the symptoms

Chronic Bronchitis Reid Index?

> 50%

ADHD Disorder: criteria?

> 6 months -symptoms in 2 different settings -cause functional impairment *Inattentive Symptoms: Difficulty focusing, distractible, does not listen or follow instruction, disorganized, forgetful, loses/misplaces objects *Hyperactive/Impulsive Symptoms: Fidgety, unable to sit still, "driven by a motor", hyper-talkative, interrupts, blurts out answers

Leukemoid Reaction: what will be seen on CBC?

>50,000 WBC Increased Bands Increased Neutrophils

Myosin and Actin filaments?

A band

Systemic, skin, and joint symptoms, hepatomegaly, and elevated transmainase. Mutliple heterosexual partners. No illicit drug use. (hepatitis B or hepatitis C)

Acute Hepatitis B (HBV) *sexually, parenterally or vertically *usually significant ELEVATIONS in AST & ALT. *Urticarial Rash *Why not Acute Hepatitis C (HCV) -> typically asymptomatic; patient does not use IV drugs. Sexual transmission is rare. Dermatologic manifestions (cryoglobulinemia) are seen in chronic infection.

Reddish urine that darkens on exposure to light and air: diagnosis?

Acute Intermittent Porphyria

Down Syndrome: Hematology?

Acute Leukemia

What is the most common pediatric malignancy?

Acute Lymphoblastic Leukemia (ALL)

Down Syndrome patients have increased risk of what hematologic cancers?

Acute Lymphoblastic Leukemia (HIGHEST) Acute Myelogenous Leukemia

Central Retinal Artery Occlusion what happens to the vision?

Acute MONOCULAR Vision loss

35 yo male with abdominal pain has laparotomy which shows chalky white lesions in the mesentery. Histologic evaluation of the lesions reveals fat cell destruction and calcium deposition. This patient most likely suffers from?

Acute Pancreatitis -this was the answer in the questions but could also be Chronic Pancreatitis

Patient has sharp chest pain. Worse with inspiration, worse when lying flat. Diagnosis?

Acute Pericarditis

10 yo boy, Tea-colored urine, BP 130/80, bilateral periorbital edema. Increased Creatine. Many RBC on urinalysis. Diagnosis?

Acute Poststreptococcal Glomerulonephritis *even without a history of infection this is probably the most common diagnosis given AGE, HEMATURIA, HYPERTENSION, & PERIORBITAL EDEMA

40 yo woman. Weakness, fatigue, epistaxis, gum bleeding, mucosal pallor, clonal proliferation of WBC with an abnormal protein: diagnosis?

Acute Promyelocytic Leukemia (M3 variant of AML)

Fever, pruritic skin rash, and arthralgias (joint pain) that begin 7-14 days after exposure to an antigen?

Acute Serum Sickness

Man has syncopal episode. HIs ECG shows bradycardia with regular rhythm and narrow QRS complexes. However, there is complete desynchronization between the P waves and QRS complexes. What is the location responsible for pacing this patient's ventricles?

AV Node *this is probably THIRD-DEGREE AV Block *QRS is narrow because ventricular depolarization proceeds normally

Atrial Fibrillation: what determines the ventricular contraction rate?

AV Node REFRACTORY PERIOD *once the AV NODE is excited, it enters the REFRACTORY PERIOD during which additional atrial impulses cannot be transmitted to the ventricles; consequently, the majority of atrial impulses never reach the ventricles.

Exposure to prior trauma (eg, witnessing death, experiencing serious injury or rape), intrusive reexperiencing of the event (nightmares, flashbacks), avoidance of associated stimuli, changes in cognition or mood (fear, horror), and persistently increased arousal: diagnosis?

Acute Stress Disorder or Post-traumatic Stress Disorder (see other slides for time frame)

Muddy Brown Casts are pathognomonic for?

Acute Tubular Necrosis

35 yo homeless man with fever, abdominal pain, and jaundice: diagnosis?

Acute Viral Hepatitis

Histology of liver shows mononuclear infiltration, ballooning degeneration, and apoptosis: diagnosis?

Acute Viral Hepatitis

Acute Dystonia?

Acute onset of INVOLUNTARY muscle CONTRACTION of neck, mouth, tongue or eye muscles *occurs within days of antipsychotic use.

In Acute Rejections what is primarily damaged?

Acute rejection causes primarily VASCULAR DAMAGE

Antiviral drugs for the treatment of primary genital herpes?

Acyclovir (BEST ORAL bioavailability) Valacyclovir Famciclovir

Burkitt Lymphoma: associated with what virus?

EBV

vWF & Factor VIII are produced where?

ENDOTHELIAL CELLs *although most clotting factors are synthesized in the LIVER

MOLLUSCUM BODY from a molluscum contagiosum biopsy is what?

EOSIONPHILC CYTOPLASMIC INCLUSIONS

Teenager girl with grayish-white vaginal discharge that is fishy-smelling. Vagina is mildly itch. What will be seen on microscopy?

EPITHELIAL CELL covered with GRAM-VARIABLE ROD (*CLUE CELLS*) *Bacterial Vaginosis (Gardnerella Vaginalis)

Syndrome of Inappropriate Antidiuretic Hormone (SIADH): what is the body fluid status?

EUVOLEMIC Hyponatremia *answer to question said Normal Extracellulr Fluid Volume

Keloids result from?

EXCESSIVE COLLAGEN FORMATION during the remodeling phase of wound healing. *present as raised, painful and pruritic nodules that grow beyond the wound borders.

Down Syndrome: Neurology?

Early-onset Alzheimer Disease

Primary CNS Lymphoma: associated with that virus?

Ebstein-Barr Virus (EBV)

Primary Central nervous system lymphoma is the second most common cause of ring-enhancing lesions with mass effect in HIV patients. Usually just a solitary lesion. What virus are these associated with?

Ebstein-Barr Virus (EBV)

Solitary ring enhancing mass in brain associated with HIV?

Ebstein-Barr Virus (EBV)

Antifungal: inhibit the synthesis of GLUCAN, a component of the fungal cell wall?

Echinocandins (any FUNGIN) *Caspofungin* *Micafungin*

A woman who is taking Estrogen and Progesterone supplements for severe hot flashes has a lab test performed. Shows normal TSH level. What will happen to the FREE T4 and FREE T3 levels?

FREE THYROID HORMONES --> REMAIN NORMAL!

Infertility patient: Menotropins mimic what?

FSH

What layer of the Endometrium os sloughed away during Menstruation?

FUNCTIONALIS layer (sloughs away as menstrual flow)

Atopic Dermatitis (Eczema): where does it appear in ifnants?

Face Scalp Extensor Surfaces

Patient with increased sensitivity to everyday sounds, and often withdraws socially as a result: What cranial nerve is responsible for this condition?

Facial Nerve *Stapedius Muscle (innvervated by Stapedius Nerve, a branch of CN VII) *Bell's Palsy may present with IPSILATERAL HYPERSACUSIS

most common cause of inherited thrombophilia?

Factor V Leiden

Patient has bilateral basal crackles, increased JVD, hepatomegaly, ascites, peripheral pitting edema, ecchymoses are present across each extremity. Increased PT, Decreased platelets. Patient is given IM vitamin K and his lab results are unchanged. What is the most likely causing the lab abnormality?

Factor VII deficiency

Treatment for MODERATE to SEVERE Hemophilia A?

Factor VIII replacement

Hydrocele and Indirect Inguinal Hernia:

Failure of PROCESSUS VAGINALIS TO CLOSE (failure of OBLITERATION OF THE PROCESSUS VAGINALIS)

On x-ray of a young infant there is distention of stomach and duodenum: what embryologic process has failed?

Failure of gut recanalization

Liver Injury: why is Prothrombin Time Increased?

Failure of hepatic synthetic function and deficiency of factor VII (which has the shortest half-life of all the procoagulant factors)

T or F: Hepatitis B Virus has Viral Cytotoxic Effect which causes hepatocyte Necrosis?

False *HBV has no direct cytotoxic Effect

Herpes zoster: typical antiviral used?

Famciclovir

3 days after sustaining bilateral femur fractures, patient is confused and agitated, short of breath, oxygen saturation is 84%, chest shows a petechial rash. What histologic changes will be see in this patients lungs?

Fat microglobules in pulmonary arterioles *these impair pulmonary gas exchange and induces HYPOXEMIA *NEUROLOGIC ABNORMALITIES< HYPOXEMIA, & PETECHIAL RASH

Child with hypoketotic hypoglycemia after a period of fasting. These findings are consistent with a defect in what process?

Fatty Acid Beta-Oxidation (MITOCHONDRIA) *Acyl-CoA Dehydrogenase Deficiency (most common) --> this catalyzes the first step in Beta-oxidation pathway

What is the earliest lesion of atherosclerosis?

Fatty Streaks

Campylobacter Jejuni: transmission? what causes?

Fecal-oral *undercooked contaminated poultry or meat, unpasteurized milk *contact with INFECTED ANIMALS (dogs, cats, pigs)

Femoral Triangle: order?

Femoral Nerve -> Femoral artery -> Femoral Vein -> deep inguinal nodes/lymphatic vessels

Hypertensive Emergency with signs of Acute Kidney Injury: best treatment option?

Fenoldopam ****RENAL VASODILATION --> leads to INCREASED RENAL PERFUSION, DIURESIS, and NATRIURESIS (excretion of sodium in the urine) --> **makes this an excellent choice for patients with ACUTE KIDNEY INJURY *results in VASODILATION in most arterial beds and a DECREASE in Systemic BP

Blood in the lateral ventricles of a newborn infant: what structure is the most likely source of the bleeding? 2. What is this complication called?

Germinal Matrix (a highly cellular a vascularized layer in the subventricular zone from which neurons and glial cells migrate out during brain development.) 2. Intraventricular Hemorrhage (IVH)

Mutation only in egg or sperm cells.

Germline Mosaicism *Children are affected

Elderly woman with a dull ache in left side of her jaw while chewing and resolves when she stops eating and episode of dimming of vision in one eye that occurs spontaneously in 20 minutes: Diagnosis?

Giant Cell Arteritis (Temporal Arteritis) *amaurosis fugax (transient monocular visual loss) *Jaw Claudication *Polymyalgia Rheumatica (also common)

Euphoria, lethargy, and coma, constricted pupils, decreased bowel sounds, low-normal BP and HR. Life threatening respiratory depression: substance?

Heroin Intoxication

HIV negative male has confusion and inappropriate behavior for one week. He assaulted his co-worker for "staring" at him. MRI shows an area of increased signal in the right temporal lobe. What organism will be isolated from the affected area?

Herpes Simplex Encephalitis

The most common form of Viral Encephalitis?

Herpes Simplex Encephalitis

Toddler with fever and mouth pain. Swollen gums and vesicular, inflamed lesions on hard palate and lips, lymphadenopathy: diagnosis?

Herpes Simplex Virus Type 1 (HSV-1) *Gingivostomatitis *vesicles on lips, hard palate, fever, lymphadenopathy

Most common cause of SPORADIC ENCEPHALITIS?

Herpes Simplex Virus-1 (HSV-1) *acute onset of fever, headache, seizures, aphasia, mental status changes WITH NO APPARENT CAUSE!

Patients with Cholesterol Gallstone: what is the treatment options for patients refusing cholecystectomy or with high surgical risk?

Hydrophilic Bile Acids (eg, Ursodeoxycholic Acid) *this decreases biliary cholesterol secretion *IMPROVES CHOLESTEROL SOLUBILITY *really on good for smaller gallstones

Vitamin C is necessary for what process during Collagen Synthesis?

Hydroxylation of PROLINE & LYSINE

In Hyperacute rejection: what is primarily damaged?

Hyperacute rejection causes DIFFUSE INTRAVSCULAR COAGULATION with RESULTANT ISCHEMIA *called "white graft" reaction

Diabetic Ketoacidosis: what additional finding besides Hyponatremia, Hyperkalemia, ketonemia and hyperglycemia might be observed?

Hyperammonemia -> due to muscle degradation

Williams Syndrome: calcium level?

Hypercalcium (Increased sensitivity to Vitamin D)

Cirrhosis: Palmar Erythema is caused by what mechanism?

Hyperestrinism

Psoriasis: what is the most likely Dermatoligic Microscopic term?

Hyperkeratosis Parakeratosis

Subperiosteal resorption with cystic degeneration is characteristic of?

Hyperparathyroidism

Intrasellar microadenoma, recent weight gain, fatigue, BP 150/90, elevated plasma ACTH levels that are persistently high on repeat measurements. What pathologic changes will be see in this patient's adrenal glands?

Hyperplasia of the FASCICULATE layer of the Cortex

Non-neoplastic polyps?

Hyperplastic Hamartomatous Inflammatory

What is the SINGLE MOST IMPORTANT RISK FACTOR for the development of AORTIC DISSECTION?

Hypertension

Adolescent, progressive gait ataxia, impaired joint and vibration sense, kyphoscoliosis, pes cavus. They have the greatest risk of dying from?

Hypertrophic Cardiomyopathy *Friedreich Ataxia

Muscle Cramps, Perioral paresthesias, hypotension, neuromuscular hyperexcitability: cause?

Hypocalcemia (*thyroid surgery or injury to parathyroid glands)

Obstructive Sleep Apnea: electrical stimulation of what nerve may improve the pathophysiologic cause of this patient's symptoms?

Hypoglossal *cause the tongue to move forward slightly during sleep.

U wave is prominent when?

Hypokalemia Bradycardia

Legionella Pneumonia: what laboratory abnormality?

Hyponatremia

Tuberculoid Leprosy presents with?

Hypopigmentation Hair Follicle loss *focally Decreased sensation

Cardiac Tamponade: what physical exam finding are often found?

Hypotension Elevated Jugular Venous pressure Muffled Heart sounds *Pulsus Paradoxus

During programmed cell death (apoptosis), the activation of CASPASES results in eIF degradation leading to interruption of translation. As a result man y of the proteins necessary for apoptosis are translated using an alternated method: How does this method function?

INTERNAL RIBOSOME ENTRY *the ribosome is allowed to begin translation in the middle of an mRNA and not at the start codon

Damage to the Left Temporal Hemiretina will disrupt the transmission of visual information from lesion to the primary visual cortex?

IPSILATERAL Optic Nerve Lateral Optic Chiasm IPSILATERAL Optic Tract Left Lateral Geniculate Body IPSILATERAL Optic Radiations LEFT Primary Visual Cortex

What routes bypass the livers first-pass metabolism?

IV Sublingual Rectal

Physical exam reveals a large wound through the back immediately to the right of the vertebral bodies around T6. What would be injured?

IVC *I think this is the only location it could be injured

Pentameric Immunoglobulin?

IgM

Which immunoglobulin is a much better activator of the complement system?

IgM because it circulates in the pentameric form so it has more sites to bind C1

Peyer Patches: where?

Ileum

Crohn Disease: Biopsy of what part of the bowel is most helpful for diagnosing this condition?

Ileum *most frequently involved site

Where is the optimal cannulation site for the femoral vein compared to the femoral artery?

Immediately medial to the femoral artery

Cytomegalovirus (CMV) Encephalitis affects what type of patient?

Immunocompromised hosts

Toxoplasma Gondii Encephalitis affects what type of patient?

Immunocompromised hosts

32 yo man with intermittent dysphagia for solids and liquids and chest pain. Esophageal manometric study shows periodic, simultaneous, and non-peristaltic contractions of large amplitude and long duration. What is the pathogenesis of this condition?

Impaired Neural Inhibition within the esophageal MYENTERIC plexus (called Diffuse Esophageal Spasm (DES) *disorganized non-peristaltic contractions and "corkscrew" esophagus on barium esophagram *usually the food bolus is propelled downward in a coordinate fashion

Flumatide: MOA?

Impaired androgen receptor interaction

Thrombotic Thrombocytopenic Purpura: pathogenesis of this condition?

Impaired regulation of von Willebrand factor *impaired function ADAMTS13 ad von Willebrand factor (vwF) cleaving protease

Wernicke-Korsakoff: what is neurologic finding is resistant despite treatment?

Impairment in MEMORY and LEARNING

Patient has prolonged Erythema and a yellow "honey" crust on a cutaneous wound?

Impetigo

Classically occurs as a superinfection of a preexisting lesion (eczema, minor traumatic lesion) and has a golden crust appearance?

Impetigo *Streptococcus Pyogenes (GAS)

Streptococcus Pyogenes (GAS) causes what skin infections?

Impetigo Erysipela

Damage to the Left Nasal Hemiretina will disrupt the transmission of visual information from lesion to the primary visual cortex?

Ipsilateral Optic Nerve Medial Optic Chiasm CONTRALATERAL Optic Tract RIGHT Lateral Geniculate Body CONTRALATERAL Optic Radiations RIGHT Primary Visual Cortex

39 yo female with reduced taste sensation, burning sensation in her mouth, lip dryness, decreased hemoglobin, MCV of 67. What is the most likely cause of her symptoms?

Iron Deficiency *Hypochromic, Microcytic Anemia *Glossal Pain, dry mouth, atrophy of the tongue papillae, alopecia, and pica

Lysosomal rupture: Reversible or Irreversible injury?

Irreversible Cell Injury

Neuronal body shrinks, deep eosinophilia of cytoplasm, pyknosis of the nucleus, and loss of Nissl substance: what does this indicate?

Irreversible Cell Injury

Nuclear pyknosis (condensation), karyorrhexis (fragmentation), karyolysis (fading): Reversible or Irreversible injury?

Irreversible Cell Injury

phospholipid-containing amorphous densities within mitochondria: Reversible or Irreversible injury?

Irreversible Cell Injury

Mitochondrial Vacuolization: Reversible or Irreversible injury?

Irreversible Cell Injury *appearance of VACUOLES & PHOSPHOLIPID-containing AMORPHOUS DENSITIES WITHIN MITOCHONDRIA generally signifies IRREVERSIBLE INJURY -->cell cannot make ATP via oxidative phosphorylation and the cell cannot recover.

Class IB drugs are useful for treating what type arrhythmias?

Ischemia-induced Ventricular Arrhythmias *one of the most common causes of death in the short term following acute MI.

Patient with abdominal pain and bloody diarrhea following a complicated surgical procedure: diagnosis?

Ischemic Colitis *question will often state surgery complicated by HYPOTENSION

Class 1C drugs should NOT be used in patients with?

Ischemic heart disease Structural heart disease

CSF cultures grow Mycobacterium Tuberculosis with significantly decreased activity of intracellular catalase peroxidase. The isolates would most likely exhibit resistance to what agent?

Isoniazid (INH)

Common Peroneal Nerve: where is it located at the knee?

It courses superficial and laterally around the HEAD OF THE FIBULA. -often injured by BLUNT TRAUMA to the LATERAL KNEE

Why should Mycobacterium Tuberculosis never be treated with drug monotherapy?

It has fast emergence of MYCOBACTERIAL ANTIBIOTIC RESISTANCE from rapid, selective GENE MUTATIONS

Unvaccinated toddler with fever, cough, congestion and red eyes. Rash started on face yesterday and spread to her trunk arms and legs today. Rash is a diffuse, maculopapular and erythematous rash: Diagnosis?

Measles Virus (Rubeola) *Conjunctivitis, Coryza, Cough, Koplik Spots *Blanching Maculopapula Rash

Plaque Stability largely depend on what?

Mechanical Strength of the fibrous cap.

Failure to Obliterate the omphalomesenteric vitelline duct: what does this refer to?

Meckel Diverticulum

True Diverticulum?

Meckel's Normal Appendix

Swelling and tenderness to the medial aspect of right knee, and widening of the medial joint line can be felt when a gentle force is applied to the lateral knee with the lower leg kept stationary. 1. What is injured?

Medial Collateral Ligament (MCL)

Hepatitis Encephalopathy (jerky involuntary hand movements, yellow sclerae, confusion): what is a know treatment to improve this confusion?

LACTULOSE (Intestinal Content Acidification) *ACIDIFIES the contents of the gastrointestinal tract by converting NH3 into NH4+ which causes more NH3 to diffuse from the blood into the gut.

Change in Executive function, PERSONALITY, APATHY & DEPRESSION: What lobe of the brain is affected be specific!

LEFT Frontal Lobe *Apathy & Depression --> Left-sided lesions

Imaging studies reveal a fracture of the LEFT 12th RIB? What structure is most likely to be lacerated by the fractured bone?

LEFT KIDNEY

Sensitivity is calculated from what side? Specificity is calculated from what side?

LEFT side of the 2 x 2 RIGHT

Klinefelter Syndrome: Serum LH, FSH, testosterone, Estrogen and Sperm count

LH: Increased FSH: Increased Testosterone: Decreased Estrogen: Increased Sperm Count: NONE *INHIBIN is also decreased *both Sertoli Cells and Leydig cells are typically dysfunction

Since Class 1B bind to Sodium Channels LESS avidly. What does this effect cause?

LITTLE USE DEPENDENCE -there is minimal cumulative effect over multiple cardiac cycles

Poststreptococcal Glomerulonephritis: LM, IF, EM?

LM: Enlarged, hypercellular glomeruli IF: GRANULAR, deposits of IgG & C3 EM: SubEPITHELIAL HUMPS

LMNOP: mnemonic?

LMNOP - Lithium side effects: M ovement (tremor) N ephrogenic diabetes insipidus Hyp O thyroidism Pregnancy problems

Patient with pulmonary and peripheral edema due to heart failure and chronic kidney disease (CKD) was treated in the hospital 1 week ago. Now she presents with difficulty hearing. What was she likely treated with in the hospital for her heart failure and edema?

LOOP diuretics (Furosemide) *OTOTOXICITY -> usually occurs with higher dosages, preexisting CKD, or rapid administration

Retinoblastoma (RB1) a tumor suppressor gene. In order for tumorigenesis to occur what must happen?

LOSS of HETEROZYGOSITY ("2-hit Hypothesis")

Patient has Generalized Anxiety Disorder. Mild Liver disease and moderately elevated hepatic transaminases. SSRI is initiated. 2 days later patient reports the anxiety is worsened. Physician wants to prescribed a benzodiazepine for anxiety but wants to avoid prolonged or excessive daytime fatigue and impaired judgement. What is the most appropriate choice?

LOT Lorazepam Oxazepam Temazepam **LOT are PREFERRED in patients with IMPAIRED HEPATIC METABOLISM

Congestive Heart Failure: what would the LDH level be in pleural effusion caused by CHF?

LOW Lactate Dehydrogenase Content

Lepromatous Leprosy is characterized by what type of immunity?

LOW-Cell mediated immunity with a HUMORAL TH2 RESPONSE *COMMUNICABLE

Ehlers-Danlos syndrome is caused by an inherited defect in what enzymes?

LYSL HYDROXYLASE PROCOLLAGEN PEPTIDASES

Older Children and adults with bloating, fatulence, abdominal discomfort and diarrhea: diagnosis?

Lactose Intolerance *biopsy is not indicated

50 yo with Difficulty in getting up from a chair and climbing stairs. Diplopia, dry mouth, 45 pack years smoking history, man has trouble with erections. decreased deep tendon reflexes, chest x-ray shows irregular round mass in the upper lobe of right lung. What is most likely diagnosis?

Lambert-Eaton Myasthenic Syndrome (LEMS) *IMPROVES with muscle use *DRY MOUTH & IMPOTENCE (not in Myasthenia Gravis)

Post-traumatic Stress Disorder: time frame?

Last longer than 1 month

Patient has been losing excess weight, what hypothalamus nuclei may be involved?

Lateral Area *If you zap your lateral area, you shrink laterally.

Microcytic anemia, constipation, and mental status change in the setting of construction work: diagnosis?

Lead Poisoning

The apparent prolongation of survival in patients who underwent a screening test that allowed for earlier diagnosis but did not improve prognosis?

Lead-time Bias

PCWP closely reflects the pressure in what?

Left Atrium & LVEDP

ST elevations in leads I and aVL: what artery?

Left Circumflex Artery

Left Temporal Hemiretina damage: what geniculate body is affected?

Left Lateral geniculate Body

Right Nasal Hemiretina damage: what geniculate body is affected?

Left Lateral geniculate Body

Patient has hoarseness for past 2 weeks. CT reveals dilation of the aortic arch. What nerve is affected?

Left Recurrent Laryngeal Nerve (wraps around the aortic arch)

Right hip drops every time patient drops every time he raises his right foot off the ground: what nerve?

Left SUPERIOR Gluteal Nerve

Virchow's Nodes?

Left Supraclavicular lymph nodes *this node receives drainage from the thoracic duct and samples lymph from ALL the ABDOMINAL VISCERA

Sputum gram stain showing many neutrophils but few or no organisms: pathogen?

Legionella

Dry cough, GI, mild confusion, interstitial infiltrates in setting of a recent cruise: organism?

Legionella Pneumonia *DIARRHEA, CONFUSION, HEADACHE, HIGH FEVER< PATCHY INFILTRATES (may progress to consolidation) *suspect in recent exposure to CONTAMINATED WATER (cruise ships, spas, hospitals, or air-conditioned hotels)

Hormone that causes Satiety?

LeptIN *keeps you thIN

Parinaud Syndrome is due to direct compression of what?

Lesion of the SUPERIOR COLLICULI (Pretectal region of the midbrain) *Pinealoma, Stroke, hydrocephalus could cause this

Leukocyte Alkaline Phosphatase test score is ELEVATED: diagnosis?

Leukemoid Reaction

Allergic Asthma: numerous substances are thought to play a role in the pathogenesis, but only two substances have pharmacologic antagonists that offer clear therapeutic benefit. What are they?

Leukotrienes Acetylcholine

Increased thickness of the Stratume GRANULOSUM: what disease?

Lichen Planus

When does the Duodenum change to the Jejunum?

Ligament of Treitz

Nicotinic Receptors are what type of type of receptors?

Ligand-gated Na+/K+ channels

What antibiotic could precipitate Serotonin syndrome?

Linezolid

What is the toxic component of Lipopolysacharide (LPS)?

Lipid A

Fatty Streaks: microscopically appear as?

Lipid-laden MACROPHAGES (foam cells) in the INTIMA -eventually may progress to atherosclerotic plaques

E. Coli Bacteremia & Septic Shock: Virulence Factor?

Lipopolysaccharide *Macrophage activation causes widespread release of IL-1, IL-6 & TNF-alpha

Postprandial short-acting insulin?

Lipro *Aspart Glulisine

Gram-positive rod with Tumbling motility?

Listeria Monocytogenes *pretty sure only organism with Tumbling motility

Patient is receiving treatment for mood swings and sleep problems now has constipation, dry skin, hair loss, weight gain. If this is a medication side effect. BP is 110/70 and pulse is 55/min. What drug is it most likely?

Lithium (MOOD STABILIZER for BIPOLAR DISORDER) Weight gain, dry skin, hair loss, and constipation with bradycardia are due to HYPOTHYROIDISM.

Measles Virus vaccine what type?

Live attenuated vaccine

Angiotensinogen is produced by what?

Liver

Red Infarcts: occur in what organs?

Liver Lung Testis Intestine

What organ makes IGF-1?

Liver (in response to GH binding to receptor)

Drug-Induced Lupus Erythematosus: What metabolic process underlies this condition?

Liver Acetylation *patients who are slow acetylators have increased risk.

The effects of cocaine on the nasal mucosa are mediated in large part by what effect of the drug?

Local Vasoconstriction with tissue ischemia

Dissection of the axillary lymph nodes can injury what nerve?

Long Thoracic Nerve

3 yo is brought to ED with high fevers and malaise for 4 days. He began limping yesterday and won't use his right leg. Scintigraphy is most likely to reveal increased focal radiotracer uptake in which of the following areas?

Long bone METAPHYSIS *most likely has HEMATOGENOUS OSTEOMYELITIS (predominantly affects children)

Pregnancy-associated DVT: what is best treatment

Low Molecular-weight Heparins (LMWHs) *Enoxaparin, Dalteparin

Hamartoma: what organ is the most common location?

Lung

ALK oncogene: associated neoplasm?

Lung Adenocarcinoma (non-small cell lung cancer)

Women: cancer with most deaths?

Lung Cancer

What cancer may arise in a history of a women with radical mastectomy with axillary lymph node dissection for breast cancer?

Lymphangiosarcoma *PERSISTENT LYMPHEDEMA (with chronic dilatation of lymphatic channels) predisposes to the development of Lymphangiosarcoma *rare malignant neoplasm of the endothelial lining of lymphatic channels

Euphoria, depersonalization, visual illusions, oriented and knows the effects are due to drug ingestion, and no nystagmus: substance?

Lysergic Acid Diethylamide (LSD) *potent HALLUCINOGEN *patients are ORIENTEd

If a patient has Pyruvate Dehydrogenase Deficiency. What amino acids would be the most safe if supplemented to the patient?

Lysine or Leucine *these are exclusively KETOGENIC --> they would not lead to INCREASED formation of lactic acid

Thick filaments are bound to structural proteins where?

M Line

Vinca Alkaloids (Vincristine, Vinblastine) affect what phase of the cell cycle?

M Phase (Mitosis this is when Cell Division occurs) *the replicated chromosomes are unable to align and subsequently unable to segregate into the daughter cells

What stage of the cell cycle does Paclitaxel affect?

M phase (Mitosis) *Microtubule Inhibitor

Temporal Arteritis is an inflammatory condition that affects the walls of what sized arteries?

MEDIUM & LARGE Arteries

Haemophilus Influenzae Type B vaccine prevents what disease?

MENINGITIS EPIGLOTTITIS Pneumonia Bacteremia *The vaccine has no effect on NONTYPABLE Haemophilus Influenzae

Most cells of the human body express what type of MHC?

MHC Class I

Failure to acidify lysosomes would lead to deficient expression of what MHC class?

MHC Class II *this leads to DEFICIENT EXPRESSION of MHC CLASS II bound to foreign antigen and subsequent lack of interaction between APC's and T-cells

Superior Vena Cava Syndrome condition is caused by which of these? (Mediastinal mass OR Superior sulcus tumor)

Mediastinal Mass -Superior Sulcus tumor (Pancoast tumors) are at the lung apex and frequently cause shoulder pain and compression of the brachial plexus leading to Horner's Syndrome. Superior Vena Cava Syndrome can sometimes result from Pancoast Tumors but Shoulder Pain and Horner's Syndrome are more frequent manifestations.

Trachea & Esophagus: what lymph nodes?

Mediastinal Nodes

Fine-needle biopsy of thyroid: Nests of polygonal cells with congo red-positive deposits?

Medullary Thyroid Cancer *nests or sheets of POLYGONAL or SPINDLE-shaped cells with EXTRACELLULAR AMYLOID deposits

Presents with unilateral cafe-au-lait spots, fibrous dysplasia, precocious puberty, multiple endocrine abnormalities.

McCune-Albright syndrome

12 yo boy with headaches and gait instability. MRI shows tumor in Cerebellum. Histology shows sheets of small blue cells with hyperchromatic nuclei and scant cytoplasm: Diagnosis?

Medulloblastoma *highly MALIGNANT

Down Syndrome: What are the 3 mechanisms that may cause this?

Meiotic Nondisjunction (95%) Unbalanced Translocation (2-3%) Mosaicism (<2%)

Skin lesion that is S-100 positive?

Melanoma

Brain has several lesions in the right temporal lobe. Biopsy shows neoplastic tissue with a mutation in the gene that encodes BRAF, a protein kinase. The point mutation results in a valine --> glutamic acid substitution at position 600. What is the diagnosis?

Melanoma *BRAF V600E mutation is seen in 40 - 50% of patients with Melanoma

Young man with an ECG that shows QT-interval prolongation: assuming this is inherited, the relevant mutation most likely affects what structure?

Membran potassium channel proteins (contributes to the delayed rectifer current of the cardiac action potential)

The Hallmark of IRREVERSIBLE injury is?

Membrane Damage

Repaglinide: what is the molecular target of this medication?

Membrane ion channel *increases insulin secretion by INHIBITING POTASSIUM CHANNELS on beta-cell membranes *this INCREASES Calcium INFLUX and stimulates beta cell INSULIN release

Protein in urine and antibodies found to the phospholipase A2 receptor: diagnosis?

Membranous Nephropathy

Recurrent vertigo, earfullness/pain, unilateral hearing loss & tinnitus: diagnosis?

Meniere Disease

Apocrine Sweat Glands: what type of glands?

Merocrine

Eccrine Sweat Glands: what type of glands?

Merocrine

Salivary Glands: what type of glands?

Merocrine

Dopaminergic pathway functions to regulate behavior

Mesolimbic-Mesocortical

Schizophrenia: is related to a dysfunction in what dopaminergic pathway?

Mesolimbic-Mesocortical *functions to regulate behavior

low pH, low serum bicarbonate, low PaCO2

Metabolic Acidosis *usually associated with a DECREASE in PaCO2 as a result of RESPIRATORY COMPENSATION

Coronary CT reveals a patient has several non obstructive atherosclerotic plaques in the coronary arteries. One year later the patient has a thrombotic occlusion of LAD artery. High Intraplaque activity of which of the following enzymes most likely resulted in this patient's myocardial infarction.

Metalloproteinases *Macrophages that infiltrate the atheroma (plaque) contributed to the BREAKDOWN of extracellular matrix proteins (Collagen) by SECRETING METALLOPROTEINASES *During the chronic inflammatory progression of an atheroma, the fibrous cap is continually being remodeled. The balance of collagen synthesis and degradation determines the mechanical strength of the cap.

Pheochromocytoma & Paraganglioma: what will be elevated in serum & urine?

Metanephrines & Normetanephrine VANILLYLMANDELIC ACID *Paraganglioma is a catecholamine-secreting tumor arising from CHROMAFFIN CELLS. But NOT IN THE ADRENAL GLAND. In a different location.

Most common adult brain tumor?

Metastases from other cancers *lungs, breast, kidney, skin (melanoma)

Graves treatment: preferred thionamide?

Methimazole *PTU causes Hepatic failure

Maturing erythrocytes lose their ability to synthesize HEME when they lose what organelle?

Mitochondria *Mitochondria is responsible for the FIRST and the FINAL 3 steps of the HEME synthesis

What phase of the cell cycle does Cell division occur?

Mitosis

Most common valve affected by Infective Endocarditis.

Mitral Valve

What is the most common underlying valvular disease predisposing to the development of Infective Endocarditis?

Mitral Valve Prolapse (MVP)

Acute Rheumatic Fever: early valve lesion?

Mitral Valve Regurgitation

Two major differences between MULLERIAN AGENESIS and COMPLETE ANDROGEN INSENSITVITY SYNDROME?

Mullerian Agenesis -ovaries (46 XX) -Normal pubic hair (receptor works!) Androgen Insensitivity -cryptorchid testes (hidden somewhere) -minimal to absent hair (receptor doesn't work)

Expression of what is responsible for absence of a regression of the Uterus during development of a male?

Mullerian Inhibiting Factor

Patients with Zollinger-Ellison Syndrome (GASTRINOMAS) should under go testing to exclude what?

Multiple Endocrine Neoplasia Type 1 (MEN type I)

Leukemoid Reaction: what is seen on blood smear?

NEUTROPHILS may include the following: -Dohle Bodies (BASOPHILIC OVAL INCLUSIONS) -TOXIC GRANULATIONS -VACUOLES

Urinary acid excretion occurs primarily in the form of what?

NH4+ and Titratable acids (H2PO4-)

Chlamydia: is it sensitive to Penicillin?

NO

Does bacterial Meningitis typically cause Intracranial Hemorrhage? 2. key differences from the two?

NO 2. Meningitis will have Fever or Hypothermia.

Mycoplasma: It is sensitive to Penicillin?

NO *NO PEPTIDOGLYCAN CELL WALL

Cavernous Hemangioma: is biopsy indicated?

NO --> *because of RISK of Hemorrhage

Klinefelter Syndrome: Sperm count?

NONE (Azzoospermia) --> causes INFERTILITY

Patient with acute MI and bradycardia or heart block, hypotension or overt heart failure (pulmonary edema): what beat blocker should you use?

NONE they are contraindicated!!

Majority of H Influenzae strains isolated from middle aspirates of children with acute otitis media are what type?

NONTYPABLE Haemophilus Influenzae

What type of Haemophilus Influenzae causes Otitis Media?

NONTYPABLE Haemophilus Influenzae

NONTYPABLE Haemophilus Influenzae: what is different from Type B?

NONTYPABLE does NOT produce a capsule

Patient has a vertebral fracture what are the serum levels of Calcium, phosphorus, and PTH?

NORMAL *In primary Osteoporosis (not caused by a medical disorder) all of these values are typically normal!

Teenager who is frequently mood and spends most of her time on the computer in her locked bedroom. Mother says she gets upset for no reason. When Mother complimented her on her party outfit, the girl screamed, "I look hideous and I'm not going," but later that days she seemed find and went to the party. What is the best explanation for this girls symptoms?

NORMAL adolescent Behavior

NSAID use like Ibuprofen and naproxen with loop diuretics can result in a decreased diuretic response. Why?

NSAIDS-->Inhibit PROSTAGLANDIN SYNTHESIS

How is using Misoprostol for NSAID-induced peptic ulcers effective?

NSAIDs block PGE 1 production * Misoprostol is a Prostaglandin E analog (agonist) so it will INCREASE production and secretion of gastric mucous barrier and DECREASE acid production.

Diabetic Ketoacidosis: Serum levels of Na, K, Glucose?

Na: DECREASED (Hyponatremic) K: INCREASED (Hyperkalemic) Glucose: INCREASED *Hyperkalemia is due to the K/H exchange in the cells

What cells are responsible for destruction of cells with decreased or absent MHC class I proteins on their surfaces?

Natural Killer (NK) cells

DNA is what charge? Histones are what charge?

Negatively charged Positively charged

RANK-Ligand & MONOCYTE COLONY-STIMULATING FACTOR (M-CSF): these 2 factors stimulate what?

Osteoclastic precursors to become mature osteoclasts

RANK/RANK-L interaction is essential for the formation and differentiation of what?

Osteoclasts

Paget's disease initial abnormality is classified by what cell type?

Osteoclasts *Osteoclastic bone resorption

Brittle bones, short stature, scoliosis, blue sclerae, hearing loss, increased skin, and ligament laxity, and easy bruisability: diagnosis?

Osteogenesis Imperfecta (OI)

Thin Sclerae with visibility of the underlying choroid layers: diagnosis?

Osteogenesis Imperfecta (OI) *BLUE SCLERA --> caused by DECREASED COLLAGEN content which makes the sclerae abnormally thin and translucent

Osteomalacia: characterized by decreased mineralization of?

Osteoid

Bone pain and muscle weakness in the setting of limited nutritional intake and minimal sunlight exposure: diagnosis?

Osteomalacia

Trabecular thinning with fewer interconnections: characteristic of?

Osteoporosis

Paget Disease is associated with development of what cancer?

Osteosarcoma

Benign Paroxysmal Positional Vertigo (BPPV): What is the cause?

Otoliths in semicircular canals *Brief episodes of vertigo brought on by HEAD MOVEMENT

Patient has Neisseria Meningitidis: what microbial components is most likely to show the closest correlation with these patients' morbidity and mortality?

Outer Membrane LIPOOLIGOSACCHARIDE LOS is analogous to the lipopolysaccharide (LPS) *LOS lacks the O Antigen other than this deficit it is structurally similar to LPS

Sudden onset unilateral pelvic pain in a women. Right ovary shows a mass with no blood flow to the ovary: Diagnosis?

Ovarian Torsion

Endometrial sampling reveals coiled glands and occasional cytoplasmic vacuoles in the glandular epithelium: when did this biopsy occur?

Ovulation (Day 14)

E. Coli Urinary Tract Infections: Virulence Factor?

P Fimbriae *allows cell adhesion to uroepithelium

Renal Plasma flow is estimated by?

PAH Clearance = (urine [PAH] x urine flow rate) / plasma [PAH]

Hernia that is LATERAL to the Inferior Epigastric Vessels: what is the pathophysiology?

PATENT PROCESSUS VAGINALIS *Indirect Inguinal Hernia

What is the mechanism used to clear the respiratory tract of deposited particles less than 2 micrometers in size?

Phagocytosis (MACROPHAGES) *SMALLES dust particles *cause PNEUMOCONIOSES by STIMULATING CONNECTIVE TISSUE GROWTH

Pheochromocytoma: what medication should be initiated first in preparation for surgery in this patient?

Phenoxybenzamine *prevents INTRAOPERATIVE HYPERTENSIVE CRISIS (during manipulation of the adrenal gland causing massive vasoconstriction)

Violent behavior, ataxia, nystagmus, poor memory, seems to be responding to internal stimuli and symptoms resolve within 8 hours: what substance?

Phenycyclidine *NYSTAGMUS *VIOLENT *HALLUCINATIONS *ATAXIA

What drug has no effect on heart contractility but causes DECREASED RENAL BLOOD FLOW?

Phenylephrine *ONLY stimulates Alpha-1 (DECREASES renal blood flow)

52 yo Woman has 6 month history of episodic headaches associated with fast heartbeat, nausea, and diaphoresis. During a screening colonoscopy she had another symptomatic episode associated with BP of 220/145 and pulse of 122/min. CT reveals right adrenal mass. Diagnosis?

Pheochromocytoma

Milrinone, Inamrinone: MOA?

Phosphodiesterase-3 Inhibitors *which leads to INCREASE in intracellular cAMP (heart) which leads to promotes intraellular calcium influx and INCREASES cardiac CONTRACTILITY *also INCREASED cAMP (vascular smooth muscle) which leads to VASODILATION*

Fructose 2,6-bisphosphate activates what?

Phosphofructokinase-1 (INCREASES Glycolysis)

Lesch-Nyhan Syndrome: what enzyme is likely INCREASED?

Phosphoribosyl Pyrophosphate Amidotransferase (PRPP)

Enteroviruses are a part of what virus family?

Picornavirus

12 yo boy with headaches and gait instability. MRI shows tumor in Cerebellum. Most likely diagnosis?

Pilocytic Astrocytoma *#1 brain tumor in children

Young child has headache, blurry vision, bilateral papilledema, inability to gaze upwards, and bilateral ptosis. Most likely tumor location?

Pineal Gland (Pinealoma) *precocious puberty may also occur in a young boy

In a patient with diabetes who has a necrotic (foul-smelling) infected ulcer: treatment?

Piperacillin-Tazobactam *BROAD-spectrum antimicrobial coverage *effective against a large number of gram-positive, gram-negative, and anaerobic activity *Bacteroides and other organisms may be present

Antimicrobials with anaerobic activity?

Piperacillin-Tazobactam Metronidazole Clindamycin (in most cases) Carbapenems

Bacteroides Species: treatment?

Piperacillin-Tazobactam Metronidazole Clindamycin (in most cases) Carbapenems

Sciatic nerve is possibly entrapped in the greater sciatic foramen. What structure passes through the foramen and occupies most of its volume?

Piriformis Muscle *Sciatic Nerve -> passes BELOW Piriformis M.

Headaches and decreased libido are present for months before the actual hemorrhage event.

Pituitary apoplexy (acute hemorrhage into the pituitary gland) --> occurs in patients w/ preexisting PITUITARY ADENOMAS *bleeding presents acutely w/ SEVERE HEADACHE & VISUAL DISTURBANCES (mimics SUBARACHNOID HEMORRHAGE)

The likelihood of plaque ruptures is typically related to what?

Plaque STABILITY rather than plaque size or degree of luminal narrowing.

Woman has a UTI from Escherichia Coli. Her urine culture shows extended-spectrum beta-lactamase E. Coli. What feature is most likely to be found in this organism?

Plasmid with drug resistance gene *ESBL (Extended-Spectrum Beta-Lactamase) enzymes are often located on PLASMID and therefore are transferred between organisms through CONJUGATION

Pituitary Apoplexy: what did the patient have prior to the hemorrhage?

Preexisting Pituitary Adenoma *Headaches *Decreased Libido (man)

Botulinum Neurotoxin blocks what?

Presynatpic Exocytosis of ACh vesicles

Fatigue, itching, hepatomegaly, positive antimitochondrial antibody?

Primary Biliary Cholangitis

45 yo woman with long history of pruritus and fatigue who has pale stools and xanthelasma: diagnosis?

Primary Biliary Cirrhosis

A liver biopsy shows extensive lymphocyte infiltration and granulomatous destruction of interlobular bile ducts. Diagnosis?

Primary Biliary Cirrhosis

Florid Duct Lesion: diagnosis?

Primary Biliary Cirrhosis

The most frequent tumor in an immunosuppressed patient is what?

Primary CNS LYMPHOMA

Man is found to have infertility from abnormal sperm tail function: what additional findings is he most likely have have?

Primary Ciliary Dyskinesia (PCD) -Chronic Sinusitis -Recurrent Otitis Media -Bronchiectasis -Situs Inversus (reversed right/left positioning of internal organs)

Granular "salt-and-peper" skull: what is this associated with?

Primary Hyperparathyroidism

Subperiosteal resorption with cystic degeneration? What is that characteristic of?

Primary Hyperparathyroidism

38 yo man with long history of ulcerative cholitis who has fatigue and an elevated alkaline phosphatase: diagnosis?

Primary Sclerosing Cholangitis

Histology of liver shows diffuse inflammation/fibrosis of the large intra- and extrahepatic bile ducts with periductal concentric fibrosis and obstruction of the small ducts?

Primary Sclerosing Cholangitis *periductal concentric fibrosis ("onion skin pattern"

Infant has an defect that he can't transport Ornithine into the Mitochondria. Restriction of what substances in his diet could improve this condition?

Protein *Ornithine is part of the Urea Cycle to remove excess Ammonia

Nephrotic Syndrome: To compensate for the decreased plasma albumin concentration, the liver increases it synthesis of what?

Proteins (including LIPOPROTEINS)

Factor V is a cofactor for the conversion of what?

Prothrombin to Thrombin *INCREASED THROMBIN --> this is another reason that leads to Hypercoaguability

Lansoprazole: type of drug?

Proton Pump Inhibitor *H+/K+ ATPase Proton Pump

The majority of the total amount of water filtered by an individual's glomeruli is reabsorbed in what portion of the nephron?

Proximal Convoluted Tubule (PCT) *Proximal Tubule reabsorb > 60% of the water filtered (doesn't matter if dehydrated or not this is where it happens!)

Like Sodium, Lithium is filtered and reabsorbed mostly where in the nephron?

Proximal Tubules

Hemosiderin can be identified with what stain?

Prussian Blue Stain

Acute Pancreatitis may be complicated several weeks later by formation of?

Pseudocyst

Knee pain and rhomboid shaped crystals in knee aspirate. Elevated WBC with neutrophil predominance.

Pseudogout *crystals are POSITIVELY BIREFRINGENT under polarized light

5 yo boy has recurrent muscle spasms. He has not gained any height for the last 1-2 years. He looks very different from his siblings. He has prominent skeletal abnormalities, including short stature, short fingers, and a round face. Serum calcium level is low, serum phosphorus and parathyroid hormone levels are elevated. What is the diagnosis?

Pseudohypoparathyroidism *Hypocalcemia *ELEVATED PTH *SHORT FINGERS

A patient's burn is infected with a gram-negative rod that is oxidase positive and non-lactose fermenting. What is it?

Pseudomonas Aeruginosa

Ecthyma gangrenosum: what bacteria?

Pseudomonas Aeruginosa

Otitis Externa: organism?

Pseudomonas Aeruginosa

Patient on chemotherapy has a T of 101 BP 90/50, pulse 124. He has multiple skin patches with an ulcerated appearance and occasional necrotic center. What organism?

Pseudomonas Aeruginosa *Ecthyma Gangrenosum

Glioblastoma: histology?

Pseudopalisading pleomorphic tumor cells

Histology shows hyperparakeratosis, acanthosis, elongations of the rete ridges, reduced or absent stratum granulosum.

Psoriasis

Fecal incontinence: what nerve is injured?

Pudendal nerve (S2-S4)

31 yo female, shortness of breath, right ventricular hypertrophy. What is the disease?

Pulmonary Arterial Hypertension

Pulmonary Endothelial Dysfunction is characteristic of?

Pulmonary Arterial Hypertension

Lung Hamartoma aka?

Pulmonary Chondroma

Physical exam reveals a large deep wound directed posteriorly adjacent to the LEFT sternal border in the SECOND intercostal space. What would be injured?

Pulmonary Trunk

Scleroderma: what is the most common cause of death?

Pulmonary sclerosis (intimal thickening of pulmonary arterioles) *both SYSTEMIC & DIFFUSE types

At 120 mm Hg intermittent Kortokoff sounds are heard only during expiration. At 100 mm Hg, Korotkoff sounds are heard throughout the respiratory cycle. What is this called?

Pulsus Paradoxus *defined by a decrease in systolic blood pressure of >10 mm Hg with INSPIRATION

Ulcer with a clean base in the duodenal bulb, what is the most likely site to demonstrate the infectious agent responsible for this?

Pyloric Antrum (Prepyloric Area) *density of H. Pylori is highest here, although H. Pylori could colonize other parts of the stomach.

Toxoplasma Gondii: treatment

Pyrimethamine & Sulfadiazine

Pyruvate Dehydrogenase: function?

Pyruvate --> Acetyl-Coa (*in the presence of oxygen)

Organism has a mutated DNA gyrase: resistance to what class of antibiotics?

Quinolones

Thiopental slow or rapid entry into brain?

RAPID entry into brain

Practice Renal Blood Flow? PAH Urine: 100 mg/mL PAH Serum 0.2 mg/mL Hematocrit of 50% urine flow of 1 mL/min

RBF = [(100 x 1.0) / 0.2] / (1 - 0.5) = 500/0.5 = 1000 mL/ min (ANSWER)

Renal Blood Flow vs Renal Plasma Flow

RBF: volume of BLOOD flow RPF: volume of PLASMA flow *Be so careful with this

Cystic Fibrosis: what is the unique NORMAL function of the transmembrane protein in sweat glands?

REABSORB CHLORIDE *dysfunctional transmembrane protein causes EXCESS CHLORIDE IN SWEAT!!

What ribosomes are involved in making the proteins for the cell membrane?

RER Ribosomes

Hereditary Spherocytosis: what is the cornerstone of treatment?

Splenectomy after patient has received vaccines

What part of the spleen are involved in removal of damaged RBCs?

Splenic RED PULP

Patient has hematemis. History of chronic pancreatitis. Black stools that are guaiac-positive. Upper GI endocscopy reveals a bleeding spot within a cluster of enlarged tortuous veins in the gastric fundus. The rest of the stomach and esophagus appear normal. Increased pressure what vascular structure is the cause of this condition?

Splenic Vein *Splenic Vein Thrombosus (from chronic pancreatitis, pancreatic cancer, and abdominal tumors) *SHORT GASTRIC VEINS drain the FUNDUS

Epidermal accumulation of edematous fluid in intercellular spaces: what term? 2. What disease?

Spongiosis *Eczematous Dermatitis

Diabetes Mellitus drugs: What 2 drug classes target the membrane ion channels?

Sulfonylureas Meglitinides (Nateglinide, Repaglinide)

Lymph Nodes: Anus below the dentate line (pectinate line)?

Superficial Inguinal Lymph Nodes

Lymph Nodes: skin from Umbilicus down?

Superficial Inguinal Lymph Nodes

Anal Canal below the pectinate line drain to what lymph nodes?

Superficial Inguinal Nodes

Skin Below Umbilicus: what lymph nodes?

Superficial Inguinal Nodes

Varicose Vein Thromboses are restricted to what system?

Superficial Venous System *don't usually cause Pulmonary Embolism

CT shows an irregular mass in the 3rd portion of the duodenum that is infiltrating beyond the gut wall. What structure is in danger of being compromised if this mass continues to enlarge?

Superior Mesenteric Artery

The Inferior Mesenteric Artery is ligated, collateral circulation from what blood vessel is responsible for preventing left colon ischemia?

Superior Mesenteric Artery

Jejunum, Ileum, Cecum, Ascending Colon, Transverse Colon to Splenic Flexure: what lymph nodes?

Superior Mesenteric Nodes

Lower Duodenum: what lymph nodes?

Superior Mesenteric Nodes

Cardinal Veins are an embryological derivative of what?

Superior Vena Cava

Patient has a deep intramuscular buttock injection and now has a Trendelenburg sign where is the Lesion?

Superomedial Quadrant of the left buttock

Combined hormonal contraceptives: what is the primary mechanism of pregnancy prevention?

Suppresses GnRH in the Hypothalamus --> DECREASES gonadotropins (FSH & LH)

Hypthalamic nuclei responsible for circadian rhythm?

Suprachiasmatic

Daptomycin: inactivated by?

Surfactant *not used in Pneumonia

Hyopnatremia and a lung mass is suggestive of?

Syndrome of Inappropriate Antidiuretic Hormone (SIADH)

Fluid filled cavity in the lower brainstem?

Syringobulbia

Fluid filled cavity in the spinal cord?

Syringomyelia

What other condition is seen with Chiari I Malformation?

Syringomyelia or Syringobulbia

Angiotensinogen is converted to Angiotensin I: where?

Systemic Circulation

CONJUGATED Encapsulated bacteria vaccines for Streptococcus Pneumoniae, H influenzae Type B, Meningococcal promote what activation of what cells?

T-CELL Activation

Acute Lymphoblastic Leukemia: which type is more likely to have a large mediastinal mass?

T-Cell Acute Lymphoblastic Leukemia *often with Superior Vena Cava Syndrome *Dysphagia *Dyspnea *Stridor

8 yo boy. Immature hematopoietic cells (blasts) in blood smear. Fever, throat pain, severe, dyspnea, tachypnea, inspiratory stridor, worsening dysphagia with solid food over the last 2 weeks. Diagnosis? 2. The neoplastic cells causing this patient's condition normally give rise to what cell?

T-Cell Acute Lymphoblastic Leukemia 2. T-lymphocytes *Dysphagia (compression probably from mediastinal mass) *Dyspnea, Stridor (caused by mediastinal mass)

Amitriptyline: what kind of drug?

TCA

HIV drugs that are Integrase Inhibitors?

TEGRA = inTEGRAse Raltegravir Elvitegravir Dolutegravir

IFN-gamma: secreted by?

TH1 cells

The vascular endothelium in the blood vessels of the heart secretes what prostaglandin that ENHANCES PLATELET AGGREGATION AND CAUSES VASOCONSTRICTION.

THROMBOXANE A2 (TXA2)

Clinically apparent Edema appears only when net plasma filtration has risen sufficiently to OVERWHELM the resorptive capacity of what?

TISSUE LYMPHATICS

32 yo women comes in for evaluation of a breast lump that is getting larger. She had a leg amputation at age 17 from Osteosarcoma. Her mother died of an adrenal tumor and her younger sister died of leukemia. What gene mutation is the most likely etiology for this patient's condition?

TP53 *Li-Fraumeni Syndrome TP53 Mutation is INACTIVATING MUTATION (normally its a TUMOR SUPPRESSOR GENE)

Chron's Disease involves what part of the intestinal wall?

TRANSMURAL Inflammation

What is recommending in patients on long-term Lithium therapy?

TSH level.

Hypothyroidism: TSH, Free T4 and Total T3 levels?

TSH: INCREASED Free T4: DECREASED Total T3: NORMAL *T4 production is IMPAIRED which stimulated TSH INCREASE. T3 is produced in peripheral tissues and usually remains NORMAL until relatively late-stage hypothyroidism.

80-90% of Patients with bilateral renal ANGIOMYOLIPOMAS have what?

TUBEROUS SCLEROSIS

Spinal cord shows axonal degeneration involving posterior columns. What is the most likely condition?

Tabes Dorsalis

B-Cell Acute Lymphoblastic Leukemia (ALL): makers?

TdT: BOTH (antigen of lymphocyte precursor) pre-B: CD10+, CD19+, CD20+

T-Cell Acute Lymphoblastic Leukemia (ALL): makers?

TdT: BOTH (antigen of lymphocyte precursor) pre-T: CD2+, CD3+, CD4+, CD5+, CD7+, CD8+

Herpes Simplex Encephalitis: affects what lobe?

Temporal Lobe

Herpes Simplex Virus-1 (HSV-1) SPORADIC ENCEPHALITIS: affects what part of the brain?

Temporal Lobe *brain autopsy will show EDEMA and HEMORRHAGIC NECROSIS

Several weeks of pain and muffled hearing in ear. Sensation of jaw clicking when chewing food. Episodic headaches, left-sided facial pain when moving jaw. Grinds teeth while sleeping. What is this?

Temporomandibular Disroder *Mandibular division of Trigeminal nerve supplies middle ear, and muscles of mastifcation so TMD presents with both OTOLOGIC and JAW PAIN.

HIV drugs that are reverse transcriptase Inhibitors? (too many to count)

Tenofovir Lamivudine Abacavir Didanosine Emtricitabine Lamivudine Stavudine Zidovudine

Antifungal that inhibts the fungal enzyme squalene epoxidase and decreases the synthesis of Ergosterol?

Terbinafine

Drug that is a recombinant PTH analog?

Teriparatide

What mediates the development of male INTERNAL genitalia?

Testosterone

The bacteria is able to decrease the intracellular concentrations of the drug by increased efflux or decreased influx and via synthesis of a protein that allows ribosomes to perform translation even in the presence of the antibiotic. 1. The bacteria are most likely resistant to what antibiotic?

Tetracycline

Where is the complement binding site on the immunoglobuline molecule?

The C1 molecule binds to the FC REGION of the HEAVY immunoglobulin chain in the region near the HINGE POINT

What happens if a tRNA is mischarged with an incorrect amino acid (and not corrected by the aminoacyl-tRNA synthetase proofreading?)

The INCORRECT AMINO ACID will be incorporated into the growing polypeptide chain *for example the tRNA anticodon reads for Cysteine but the amino acid somehow gets placed on the tRNA as alanine. The tRNA would still place amino acid at a site reading for a CYSTEINE amino acid.

Neonate delivered at home. Someone had the crazy idea to put dirt on the umbilical stump. Child is unable to open his mouth and hands are clenched. In addition to not being stupid and putting dirt on the umbilical cord, what is the most effective strategy to prevent this condition?

Vaccination of pregnant women with INACTIVATED TETANUS TOXIN (Tetanus TOXOID) *women provide transplacental IgG to the fetus *Clostridium Tetani SPORES

Newborn, lethargy, vomiting, hypotonia. Metabolic acidosis, large anion gap, ketosis, hypoglycemia. Propionic acid is markedly increased in plasma and urine. Metabolism of what amino acid contributes to this condition?

Valine or Isoleucine (Branched chain) Threonine or Methionine (could have also contributed) *Deficiency of Propionyl CoA Carboxylase

Patient with Tonic-Clonic seizures and Absence seizures?

Valproate

Both Indirect Inguinal Hernia and Hydroceles may INCREASE in size during what maneuver?

Valsalva

Antibiotics that bind to: Cell wall Glycoproteins?

Vancomycin

Mutated peptidoglycan cell wall of bacteria. 1. The bacteria are most likely resistant to what antibiotic?

Vancomycin

What drug directly binds to D-alanine-Dalanine residues, preventing the incorporation of new subunits into the cell wall?

Vancomycin

Patient is started on IV antibiotic by develops severe pruritus and a rash immediately upon infusion. When tested in several healthy volunteers as you increase the rate of drug infusion the blood histamine concentration increases. What antibiotic was given to this patient?

Vancomycin "Red Man" Syndrome

Cholesterol 7alpha-hydroxylase: function?

catalyzes the rate-limiting step in SYNTHESIS OF BILE ACIDS

What does bcl-2 gene normally inhibit?

cell death cascade *Bcl-2 protein inhibits apoptosis of tumor cells, thereby with overexpression of Bcl-2 in cancer it thereby facilitates neoplastic growth.

TUBEROUS SCLEROSIS: what things?

characterized by multiple hamartomas *Ash-leaf skin patches -Brain Hamartomas *ANGIOMYOLIPOMAS (Bilateral kidney tumors) -Cardiac Rhabdomyomas -Facial Angiofibromas

IgA Protease function?

cleaves IgA at its hinge region which FACILITATES MUCOSAL ADHERENCE of the bacteria

Ligand binds to a tyrosine kinase receptor and formation of receptor dimers occurs then what is the next effect?

conformation change that exposes TYROSINE KINASE-active sites!! *DIFFERENCE in non-receptor Tyrosine Kinase is that JAK (Janus Kinase binds to the receptor)

What is the mechanism that causes peripheral neuropathy in diabetes?

conversion of Glucose --> Sorbitol (aldose reductse) *Sorbitol may also be converted to FRUCTOSE (sorbitol deydrogenase). THIS INCREASES OSMOTIC PRESSURE & STIMULATES INFLUX OF WATER LEADING TO OSMOTIC CELLULAR INJURY of SCHWANN CELLS which contributes to peripheral neuropathy

Pyruvate Kinase: function?

converts Phosphoenolpyruvate to Pyruvate *generates a molecule of ATP

Anesthesia is injected between the right anterior and middle scalene muscles for blockade of the brachial plexus. Which of the following muscles is most likely to be paralyzed during the anesthesia? a. Platysma b. Sternocleidomastoid c. Trapezius d. Diaphragm

d. Diaphragm *see what b & c are innervated by *Ipsilateral Diaphragm is paralyzed

Persistent Depressive Disorder (dysthymia): timing?

lasting AT LEAST 2 YEARS

Phenoxybenzamine: MOA?

long-acting alpha-adrenergic blocker

DCC gene mutation: activating mutation or loss of function mutation?

loss of function mutation

VHL gene mutation: activating mutation or loss of function mutation?

loss of function mutation

WT1 gene mutation: activating mutation or loss of function mutation?

loss of function mutation

APC, Beta-Catenin gene mutation: activating mutation or loss of function mutation?

loss of function mutation *TUMOR SUPPRESSOR GENE

BRCA1, BRCA2 gene mutation: activating mutation or loss of function mutation?

loss of function mutation *TUMOR SUPPRESSOR GENE

RB gene mutation: activating mutation or loss of function mutation?

loss of function mutation *TUMOR SUPPRESSOR GENE

TP53 gene mutation: activating mutation or loss of function mutation?

loss of function mutation *TUMOR SUPPRESSOR GENE

Poisonous mushrooms are ingested. Synthesis of what would directly be inhibited?

mRNA

RNA Polymerase II: function?

mRNA synthesis

HBeAg: what is this?

marker of VIRAL REPLICATION and ***INCREASED INFECTIVITY*

RET oncogene codes for what?

membrane-bound Tyrosine Kinase Receptor

Neuroblastoma: histology?

solid sheets of small cells with dark nuclei and scan cytoplasm (small blue round cells) *Homer-Wright Rosettes)

Blastomyces Dermatitidis: location?

states adjacent to and EAST of MISSISSIPPI and OHIO RIVER VALLEYS

Nitrites: how do these treat Cyanide Poisoning?

switches iron to Fe3+ and diminished cyanide's toxic effect --> Cyanide binds to ferric ion (Fe3+) MORE AVIDLY than to MITOCHONDRIAL CYTOCHROME enzymes

Auer Rods present in Myeloblasts: what is the translocation?

t(15;17) Acute Promyelocytic Leukemia (APL)

Burkitt Lymphoma: translocation

t(8;14)

Follicular Lymphoma: chromosomal translocation?

t(l4;18)

What test is used for numerical values (ie, in mg/dL) to test means?

t-test or z-test

Checks differences between means of 2 groups.

t-test or z-test Tea is mean t for 2

RNA Polymerase III: function

tRNA synthesis

Closed-loop communication?

team members repeat back the information received to ensure the correct information has been conveyed. *reduces medical errors!

With respect to skeletal muscle what is afterload?

the FORCE AGAINST which the muscle fiber must contract

Hepatitis B Virus (HBV) what is the Integrative phase?

the HBV DNA is incorporated in the host genome of the infected hepatocytes that survived the immune response of the (CD8+ T lymphocytes). -Infectivity ceases and liver damage tapers off when the antiviral antibodies appear and viral replication stops. *Risk of HEPATOCELLULAR CARCINOMA, remains elevated because the HBV DNA has been integrated into the host genome.

What happens when the Stapedius Muscle is paralyzed (or Facial Nerve is injured)?

the Stapes oscillates more widely producing HYPERACUSIS (INCREASED sensitivity to every day sounds)

Power of a study?

the ability of study to detect a difference between groups when SUCH A DIFFERENCE TRULY EXISTS

The sequence of amino acids in a growing polypeptide chain is dictated by?

the interaction of the mRNA CODON with the tRNA ANTICODON.

Mode: definition?

the most FREQUENTLY observed data point

Penetrance?

the probability that a person with a given mutant genotype will exhibit the corresponding phenotype.

Secondary Erythrocytosis: what is the EPO?

this is an Absolute Eryhtrocytosis *with INCREASED EPO

Transferrin?

transports iron through plasma

ACE-I: how does this drug potentially increase Creatine initially?

DECREASED GFR *ACE-I's block the conversion of angiotensi I to angiotensin II. This prevents the EFFERENT arteriole from CONSTRICTING more than the afferent arteriole causing GFR to decrease. It is expected for the GFR to DECREASE in all patients initially. Most clinicians are generally not concerned by this unless Creatinine increases by greater than 30%. Because long-term benefits are so great!

HIGH Osteoprotegerin/receptor activator of nuclear factor kappa-B Ligand (RANK-L) RATIO?

DECREASED Osteoclast formation & survival

Corticosteroid how do they cause peptic ulcers or GI bleeds?

DECREASED Prostaglandins

SLE: what is the complement level? why?

DECREASED complement level (C3, C4, and CH50) *due to IMMUNE COMPLEX formation

Acute Serum Sickness: what will be decreased in the serum?

DECREASED serum C3 DECREASED serum C4

Iron Deficiency Anemia: associated with?

DECREASED serum Ferritin INCREASED serum Transferrin INCREASED Total Iron Binding Capacity *Ferritin and Transferrin are opposites

Any disease that causes what will lower the minimum infectious dose of Vibrio Cholerae?

DECREASES ACIDITY of stomach

Calcitonin: function?

DECREASES Bone reabsorption DECREASES Calcium reabsorption

Ethosuximide: MOA?

DECREASES CALCIUM current in THALAMIC neurons

Vitamin D supplementation: what affect on PTH secretion?

DECREASES PTH hormone secretion

Griseofulvin: use?

DERMATOPHYTE infections *accumulates in keratin-containing tissues

Homeobox (HOX) genes encode what?

DNA-binding TRANSCRIPTION FACTORS

NNRTI: require what to be active?

DO NOT require nucleoside phosphorylation

What about the question "Do you identify as heterosexual, homosexual, or bisexual?"

DON'T ASK! *forces the patient to choose a LABEL!

Resistant bacteria have DECREASED INTRACELLULAR CONCENTRATIONS OF THE ABX of interest. When the scientist ADDS PROTONS to the growth media, INTRABACTERIAL CONCENTRATIONS OF THE DRUG sharply INCREASE. The most likely mechanism of acquired abx resistance?

DRUG PUMPED OUT OF THE CELL *adding protons to the growth medium ELIMINATES the HYDROGEN IO CONCENTRATION GRADIENT ENERGY SOURCE -> thereby inhibiting the efflux pumps.

During exercise what is the most significant factor limiting coronary blood supply?

DURATION of diastole *Increased HR shortens the time of ventricular relaxation (duration of diastole)

Hypoplasia/absence of the cerebellar vermis & cystic dilation of the 4th ventricle?

Dandy-Walker Malformation

Neuroleptic Malignant Syndrome

Dantrolene D2 agonists (Bromocriptine)

Tuberculoid Leprosy: treatment?

Dapsone Rifampin

Patient has MRSA, and is allergic to Vancomycin. They are started on once-daily IV ABX. 1 week later develops MUSCLE PAINS. and found to have ELEVATED CREATINE PHOSPHOKINASE. What was used to treat?

Daptomycin

Why do patients receiving total parenteral nutrition often have gallstones?

Decreased Cholecystokinin release due to LACK OF ENTERAL STIMULATION

Polycythemia Vera: what is the EPO level?

Decreased EPO

Phenytoin: MOA?

Decreased sodium current in cortical neurons

Phenytoin: MOA?

Decreases sodium current in cortical neurons

Midshaft fracture of Humerus: What artery and nerve may be damaged?

Deep Brachial Artery Radial Nerve

Chronic Granulomatous Disease: defect?

Defect of NADPH oxidase --> DECREASED reactive oxygen species (eg, superoxide) --> DECREASED respiratory burst in neutrophils;

Duchenne Muscular Dystrophy: associated with what change in the gene?

Deletion involving a muscle structural protein gene *FRAMESHIFT or NONSENSE Mutation (deletions that are not a multiple of 3 change the reading frame; result in NON-FUNCTIONAL PROTEIN and SEVERE CLINICAL MANIFESTATIONS like DUCHENNE.

23 yo women says her and her boyfriend have developed a deeper level of bonding over the past several months due to her adopting his belief that God has chosen him to carry out a special mission. She says that he is the new Messiah, destined to save the world. She denies seeing or hearing things that no one else does. Diagnosis?

Delusional Disorder *characterized by FIXED DELUSIONS W/O OTHER PSYCHOTIC SYMPTOMS. Functioning is intact, apart from the impact of the delusion.

Five Stages of Grief?

Denial Anger Bargaining Depression Acceptance

RBC show increased hemolysis when exposed to increasing NaCl concentration: what would be in this patient's blood smear?

Dense RBC without central pallor *Hereditary Spherocytosis

Elderly man with low-grade fevers, and fatigue. No history of heart disease Has a diastolic murmur at left sternal border. Ha nontender macules on his hands and feet. Blood culture reveals gram-positive cocci that are catalase negative, optochin-resistant, bile insoluble. This patient's medical history is most likely to include what procedure in the past month?

Dental work

Persistent feelings of detachment or estrangement from one's own body, thoughts, perceptions, and actions or one's environment.

Depersonalization/Derealization Disorder

Histamine H1 Receptor antagonist: medication name?

Dephenhydrimine Meclizine

What treatment can be used for MILD Hemophilia A?

Desmopressin (DDAVP)

Lithium therapy: 2 major side effects?

Diabetes Insipidus Hypothyroidism

Myocardial perfusion occurs mainly during systole or diastole?

Diastole

Benzodiazepines: Long Term?

Diazepam Chlordiazepoxide Flurazepam

Lupus: what is the nephritic syndrome?

Diffuse proliferative Glomerulonephritis

Patient has CHF and has upset stomach that won't go away, disturbed color perception, anorexia, nausea/vomiting and diarrhea that has worsened over 2 weeks. What is causing this?

Digoxin Toxicity

What mediates the development of Male EXTERNAL genitalia?

Dihydrotestosterone (DHT)

Loffler medium is best for what organism?

Diphtheriae

Medication that prolongs PTT, PT, but has no effect on thrombin time. What is it?

Direct Factor Xa inhibitor -RivaroXaban -ApiXaban ***Xa in the name***

Man has intense rage when he recalls childhood episodes of physical abuse by his stepfather. He takes his anger on his anger on his children or other people. What defense mechanism is this?

Displacement *the patient DISPLACES his anger at his stepfather onto his children or someone else

Resection of the ILEUM can also INCREASE the RISK of GALLSTONES: how?

Disruption of normal enterohepatic bile acids

Young female, Fevers, arthralgias, unprotected sex with a new partner, pustules on dorsal aspect of forearms, right wrist and ankle are tender on palpation: diagnosis?

Disseminated Gonorrheal Infection *ARTHRITIS *DERMATITIS *TENOSYNOVITIS *sexually active female

Polyarthralgias, skin lesions, tenosynovitis.

Disseminated gonococcal Infection (DGI) *Septic Arthritis

Purulent Arthritis without skin lesions?

Disseminated gonococcal Infection (DGI) *Septic Arthritis

Acute Promyelocytic Leukemia: patients may present with what bleeding disorder?

Disseminated intravascular Coagulation (DIC)

Presence of 2 or more distinct identities or personality states.

Dissociative Identity Disorder *more common WOMEN *associated with sexual abuse, PTSD, depression, substance abuse, borderline personality, somatoform conditions

Where is the lowest pH of the nephron found?

Distal Tubules and Collecting Ducts

What happens when a patient is taking Metronidazole and they consume alcohol?

Disulfiram-like reaction (severe flushing, tachycardia, hypotension)

What drug a low dose causes Increased Contractility and Increased Renal Blood Flow?

Dopamine (stimulates Beta-1 & Dopamine-1)

Nausea associated with migraine: medication targeting what receptors?

Dopamine Receptors

Fenoldopam: MOA?

Dopamine-1 receptor agonist *results in VASODILATION in most arterial beds and a DECREASE in Systemic BP ****RENAL VASODILATION --> leads to INCREASED RENAL PERFUSION, DIURESIS, and NATRIURESIS (excretion of sodium in the urine) --> **makes this an excellent choice for patients with ACUTE KIDNEY INJURY

Upslanting palpebral fissures, a protruding tongue, and excessive skin at the nape of the neck: Diagnosis?

Down Syndrome

46, XX, t (14;21): Diagnosis?

Down Syndrome *Unbalance Robertsonian Translocation (account for a minority of Down syndrome cases)

Lyme Disease: treatment?

Doxycycline or Ceftriaxone

Breast biopsy: ducts distended by pleomorphic cells with prominent central necrosis. Lesion does not extend beyond the ductal basal membrane. What is the origin of the lesion 2. Name?

Duct 2. Ductal Carcinoma in Situ (DCIS)

Fetal Circulation: Ductus Venosus or Ductus Arteriosus has the more oxygenated blood?

Ductus Venosus

Why is the pO2 in the left atrium and ventricle lower than that in the pulmonary capillaries?

Due to mixing of OXYGENATED BLOOD (pulmonary veins) with DEOXYGENATED BLOOD (Bronchial arteries and thebesian veins) *Anatomic shunting due to bronchial circulation

Infant, bilious emesis, air is seen in the stomach and proximal duodenum but nowhere else. What is this?

Duodenal Atresia

Recanalization failure at 8-10 weeks of gestation in Utero: diagnosis?

Duodenal Atresia

Down Syndrome: GI complications?

Duodenal Atresia Hirschsprung Disease

1 patient receives intramuscular inactivated vaccine (SALK) and other receives a live attenuated oral vaccine (SABIN). One month after vaccination, the levels of what poliovirus antibodies will differ the most between these patients?

Duodenal Luminal IgA (basically a STRONGER MUCOSAL SECRETORY IgA IMMUNE RESPONSE) *Live attenuated oral vaccine produces a stronger mucosal secretory IgA immune response than the inactivated poliovirus vaccine

Brunner Glands: where?

Duodenum

GI endoscopic procedures are performed and confirm the presence of an ulcer. The patient is told that the ULCER ITSELF IS VERY UNLIKELY TO BE MALIGNANT BASED ON ITS LOCATION. Where is the ulcer?

Duodenum Duodenal Ulcers are NOT ASSOCIATED with an increased risk of carcinoma IN THE SAME LOCATION.

Sustained, involuntary muscle contractions?

Dystonia *SUSTAINED *Spasmodie Toricollis, Blepharospasm (tight closure of the eyelid), Writer's Cramp

Lymph Nodes: Prostate?

Internal Iliac Nodes

Anal Canal above the pectinate line drain to what lymph nodes?

Internal Iliac Nodes & Inferior Mesenteric Nodes

Diffusion Impairment: what disease is an example?

Interstitial Fibrosis

Hypertrophic CardiomyopathyL left ventricular outflow obstruction created by what structures?

Interventricular Septum & Mitral Valve Leaflet

Vascular Injury in Utero: diagnosis?

Intestinal Atresia of the MIDGUT (Jejunum, Ileum or Proximal Colon)

Young alcoholic man has hematemesis in the ER. Endoscopy shows longitudinal mucosal tears at the gastroesophageal junction. This physiologic cause has created the mucosal tears?

Intrabdominal and intraluminal gastric pressure -pressure increases during retching and vomiting *Mallory-Weiss Syndrome (very commonly associated with alcoholism & bulimics)

Patients with a decreased-cell mediated immunity are at risk for infection from what types of organisms?

Intracellular organisms

Patient has a STEMI and is treated with Alteplase or Tenecteplase and several hours later is found comatose with asymmetric pupils. What caused this?

Intracerebral Hemorrhage

An older lesion where the nevus cells extend into the dermis. They are skin-to-tan-colored, dome-shaped, and sometimes pedunculated.

Intradermal Nevi

Deep Tendon Reflex (Myotatic Reflex) is tested: what structure in the muscle is responsible for the reflex?

Intrafusal muscle fibers (Muscle Spindles)

Odynophagia, dysphagia, with fever, burning chest pain of an immunocompromised individual. Endoscopy shows linear and shallow ulcerations in lower esophagus. What will histology show?

Intranuclear Inclusions *Cytomegalovirus (CMV) Esophagitis

Pyruvate Kinase Deficiency: causes what kind of disease?

Intrinsic Hemolytic Anemia

What is the best substance to measure GFR?

Inulin

Which of the following HLA Class II components is digested by Macrophages during antigen processing?

Invariant Chain *during antigen processing the INVARIANT CHAIN brings the alpha-chains and Beta-chains to form a stable complex in the Rough ER. Then the INVARIANT CHAIN is DEGRADED. and a EXTERNAL PROTEIN IS INSERTED BETWEEN THE ALPHA & BETA CHAINS for ANTIGEN PRESENTATION!

Ductal Carcinoma in Situ (DCIS) is a precursor to what cancer?

Invasive Ductal Carcinoma

The most common type of breast cancer?

Invasive Ductal Carcinoma

A patient's relapse and rehospitalization is a result of poor medication adherence which likely stems from cognitive impairment. What interventions would likely have prevented this outcome?

Involvement of a SOCIAL WORKER IN DISCHARGE PLANNING *discharge planning entails interdisciplinary collaboration among social worker, nurse, physician to consider patient's COGNITIVE STATUS, FUNCTIONAL STATUS, FAMILY SUPPORT, TRANSPORTATION TO FOLLOW-UP VISITS, SAFETY ISSUES, ***MEDICATION ISSUES.

Alkaline Phosphatase (ALP) what are the primary sources for this?

Bone & Liver

Ganciclovir: adverse effects?

Bone Marrow Suppression (leukopenia, anemia) Renal Toxicity *more toxic than Acyclovir

Type I collagen is an important structural component of what?

Bones Tedons Ligaments Skin Sclerae Fibrous Tissues

A new chemotherapy regimen is tested in 100 patients with lung cancer. 40 patients are still alive after one years. Conventional chemotherapy is given to a separate group of 500 patients. and after one year 100 are alive. How many patients need to be treated with the new regimen to save one life as compared to the conventional regimen?

(100-40)/100 = 0.6 (this number tells you the proportion of how many people died) (500-100)/500 = 0.8 0.8 - 0.6 = 0.2 ABSOULTE RISK REDUCTION 1/0.2 = 5 Number Needed to Treat (NNT) *for every 5 people treated with the new drug, one additional life is saved!

Vandillylmandelic Acid: is a byproduct of what?

(Norepinephrine & Epinephrine) are broken down to (Metanephrines & Normetanephrine) then they are broken down to (Vandillylmandelic Acid) *INCREASED Vandillylmandelic Acid in PHEOCHROMOCYTOMA

Low-Potency Typical Antipsychotics: Names 2. Side Effects?

* Chlorpromazine * Thioridazine (Cheating Thieves are Low) 2. Sedation, *Anticholinergic side effects *Orthostatic Hypotension

Drug-induced Lupus Erythematosus: what antibodies?

**ANTI-HISTONE ANTIBODIES** Anti-nuclear antibodies (ANA)

Li-Fraumeni Syndrome: gene mutation

**TP53** Inactivating mutation --> (normally its a TUMOR SUPPRESSOR GENE)

CD10+: is a marker of what?

**pre-B Cells** ONLY

In an experiment, various endogenous compounds are measured in the spinal fluid after application of noxious stimuli. One is a pentapeptide with a strong affinity to delta- and mu-receptors. What other endogenous substance has a common molecular origin with the pentapeptide described above?

*ACTH *Pentapeptide is BETA-ENDORPHIN that is derived from PROOPIOMELANOCORTI (POMC). ACTH and BETA-ENDORPHIN are both derived from POMC. *The fact that Beta-endorphin and ACTH are both derived from the same precursor suggests that there may be a close physiologic relationship between stress axis and opioid system

When the endometrium is no longer exposed to PROGESTERONE: what endometrial processes INIATES THE ONSENT OF MENSTRUATION (the bleeding)?

*APOPTOSIS of the endometrial cells

At 120 mm Hg intermittent Kortokoff sounds are heard only during expiration. At 100 mm Hg, Korotkoff sounds are heard throughout the respiratory cycle. This is found in what disease?

*CARDIAC TAMPONADE (most common) -severe asthma -COPD -Constrictive Pericarditis

Nonbacterial Thrombotic Endocarditis (NBTE): are associated with?

*Advance Malignancy (most common) Antiphospholipid Syndrome SLE (Libman-Sacks Endocarditis)

y-glutamyltransferase (GGT)- is increased in what?

*Alcohol

Diphenhydramine: may have what adverse effects?

*Anticholinergic Side Effects (incomplete bladder emptying, constipation, confusion, hallucinations, dry mouth, blurry vision, impaired sweating, tachycardia, increased risk of falls.

Clostridium Tetani: treatment?

*Antitoxin +/- vaccine booster *Diazepam (for muscle spasms)

Focal Intimal tear: associated with?

*Aortic Dissection

Digoxin Toxicity: if the toxicity is not treated what severe complication is most likely to develop?

*Arrhythmia

Macrolides

*Azithromycin *Clarithromycin *Erythromycin

ACE-I: use with caution in what conditions?

*C1 esterase Inhibitor deficiency *Bilateral Renal Artery Stenosis

Glioblastoma Multiforme: MRI

*Can cross the Corpus Callosum ("butterfly glioma") or it might be limited to 1 cerebral hemisphere

What is one of the most common competitive inhibitors of Acetylcholine?

ATROPINE (muscarinic antagonist)

Start Codon?

AUG (Methionine)

Local cutaneous adverse effects of chronic topical corticosteroid administration include what?

*DERMAL Atrophy *loss of dermal collagen *Tightening of skin *Telangietctasias *Ecchymoses -Drying, Cracking

Fever, leukocytosis, hypotension, tachycardia suggest a patient is in septic shock. A patient in septic shock is likely to have a decreased bicarbonate level and have an increased anion gap Metabolic Acidosis due to increased Lactic Acid. What is the likely cause of the increased anion gap?

*Decreased oxidative phosphorylation -End-organ hypoperfusion in SEPTIC SHOCK impairs tissue oxygenation and DECREASES OXIDATIVE PHOSPHORYLATION leading to a buildup of NADH and shunting of PYRUVATE to LACTATE following glycolysis.

Pathogen is haploid, has inner membrane and outer membrane and no nuclear membrane. Organism is sensitive to some penicillin antibiotics but not all. (Is this Chlamydia Trachomatis, E. Coli, or Mycoplasma Pneumonia)

*E. Coli* NOT Chlamydia because they have circular, plasmid-like genomes. (similar to typical gram-negative bacteria they have double membranes and periplasmic spaces. However, they have an unusual peptidoglycan cell wall and are INSENSITIVE TO PENICILLIN.

Vitiligo: caused by?

AUTOIMMUNE destruction of of melanocytes

Why does Pregnancy and usage of Oral Contraceptives predispose to gallstone formation?

*Estrogen causes CHOLESTEROL HYPERSECRETION *Progesterone -> causes GALLBLADDER HYPOMOTILITY 5-12% of all pregnant women develop gallstones during pregnancy

How does Beta-2 receptor agonist production vasodilation?

*Gs receptor -> increases cAMP -> Relaxes bronchial smooth muscles cells through INHIBITION OF MYOSIN PHOSPHORYLATION and LOWERING OF INTRACELLULAR CALCIUM CONCENTRATION

Causes of Gout?

*INCRASED Urate Production -Primary Gout -Myeloproliferative/Lymphoproliferative disorders -Tumor Lysis Syndrome -Hypoxanthine Guanine Phosphoribosyl Transferase Deficiency *DECREASED Urate Clearance -Chronic Kidney Disease -Thiazide/Loop Diuretics

What causes immunosuppression in patients taking corticosteroids?

*INCREASED Neutrophils in Serum *Neutrophil DEMARGINATION --> Neutrophil recruitment to fight infection IN TISSUES is DECREASED, potentially contributing to increased infection risk

Why does with Total T4 and Total T3 INCREASE with INCREASED Estrogen?

*INCREASED synthesis of THYROXINE-BINDING GLOBULIN

Acetylochline or Norepinephrine release from presynpatic terminal vesicles at the NEUROMUSCULAR JUNCTION depends upon what event?

*INFLUX of EXTRACELLULAR CALCIUM into the PRESYNAPTIC TERMINAL

Zidovudine: MOA?

*INHIBITS 3'-5' PHOSPHODIESTER BOND FORMATION *NRTI --> Requires Nucleoside phosophorylation to be active. -binds to reverse transcriptase and is incorporated into the viral genome as a thymidine analog. Zidovudine has an azido group in place of hydroxyl group which is normally found on the 3' end of thymidine.

Atopic Dermatitis (Eczema) has high levels of what in serum?

*IgE *peripheral Eosinophilia

A physician must respond to a patient who has just revealed an emotionally upsetting topic. What type of question?

*OPEN ENDED* Explore feelings ---->"How did that make you feel?" *BAD EXAMPLE --> "I can see how that would be upsetting; do you think it's related to your headaches and insomnia?" *Do not use questions that a patient can respond with a YES or NO!

Type II DM: what are the 2 most important pathophysiologic mechanisms?

*Insulin RESISTANCE *PROGRESSIVE Beta Cell FAILURE

What mRNA processes takes place outside the nucleus?

*Interaction with P bodies

The resting membrane potential is maintained by what membrane permeability?

*K+ efflux via NON-gated K+ channels (leak channels)** -LOW permeability to Na+

Disulfiram inhibits what?

Acetaldehyd Dehyrogenase causing ACETALDEHYDE ACCUMULATION

Infertility patient: she then receives ovulation induction therapy with a short course of MENOTROPINS followed by a single injection of hcG. The use of hCG therapy primarily mimics what physiologic event?

*LH Surge*

Superior Vena Cava Syndrome: what is the most frequent causes?

*Lung Cancers (most common) *non-Hodgkin Lymphoma (odd) DON'T FORGET!!

Primary Biliary Cholangitis & Graft-versus host disease: histology?

*Lymphocyte Infiltrate *destruction of INTRAHEPATIC BILE DUCTS

Adjustment Disorder with Depressed Mood: Is not diagnosed when?

*MAJOR DEPRESSIVE episode If the patient meets the full criteria (sufficient number and severity of symptoms then it's NOT Adjustment Disorder)

Autopsy shows enlargement of the LV cavity and INCREASED thickness of the left ventricular wall. The structural changes are most likely associated with what condition?

*Mitral Valve Regurgitation -Aortic Valve Regurgitation *Myocardial Infarction -Dilated Cardiomyopathy

Neuritic Plaques are characteristic of?

Alzheimer Dementia *aka SENILE Plaques

Cisplatin: 2 major adverse effects?

*Nephrotoxic *Ototoxic

What causes patients taking corticosteroids to experience NEUTROPHILIA?

*Neutrophil DEMARGINATION -this demargination of neutrophils that were previously attached to the vessel wall causes the NEUTROPHIL COUNTS TO INCREASE!!!

Tryptophan is also a precursor to what else?

*Niacin (Vitamin B3) *Serotonin --> Melatonin

Aminoglycosides: 2 big adverse effects?

*Ototoxicity (hearing loss, tinnitus) *Nephrotoxicity

Ovotesticular Disroder of Sex Development: findings?

*Ovaries & Testicles (BOTH present) *Ambiguous Genitalia -previously called True Hermaphroditism

Cardiac tissue conduction velocity speeds?

*Park At VENTure AVenue* Purkinje (Fastest) Atrial Muscle Ventricular AV Node (Slowest)

What are found in a variety of poisonous mushrooms that are toxic to RNA Polymerase II?

Amatoxins *stops production of mRNA

Double Y Males (XYY): findings?

*Phenotypically normal -TALL -severe ACNE -LEARNING DISABILITY -AUTISM spectrum disorders

17alpha-hydroxlase is responsible for what? 2. What are the results clinically?

*Pregenolone --> 17-OH pregnenolone *Progesterone --> 17-hydroxyprogesterone NO CORTISOL NO ANDROGENS!

Smoking can cause what problems in a fetus?

*Prematurity (low-birth weight/intrauterine growth restrictions-IUGR) *Preterm Labor *placental problems

Pseudodiverticulum caused by?

*Pulsion* --> INCREASED intraluminal pressure *created during strained bowel movements (chronic constipation

In respiratory and gastric glands, impaired function of the CFTR transmembrane protein causes what changes in chloride, sodium, and water?

*REDUCES luminal chloride *INCREASED Sodium ABSORPTION *NET Water ABSORPTION *results in dehydrated mucus and a more negative transepithelial potential difference.

An external stimulus applied to a cell increases the activity a several enzymes, including dihydrofolate reductase and DNA polymerase. What immediately precedes the observed affect? (2)

*Retinoblastoma protein phosphorylation (makes it inactive) *p53 -both modulate G1 -> S progression *increased in the enzymes listed indicates we are in S phase

Generalized anxiety disorder is being treated with Buspirone. What are 3 important characteristics?

*SLOW onset of action *carries NO RISK OF DEPENDENCE -lacks muscle relaxant or anticonvulsant properties *Buspirone (2nd line for GAD)

Ischemic Colitis: what areas are high risk to be affected?

*SPLENIC FLEXURE (border between SMA * IMA) *RECTOSIGMOID JUNCTION (border between Sigmoid artery & Superior Rectal Artery) -these are "watershed areas"

Patient being treated with an opioid analgesic experiences new-onsent upper abdominal pain that makes it difficult for her to lie down. Tenderness in RUQ.An adverse drug effect involving what structure is most likely responsible for this condition?

*Smooth Muscle Cells *the Morphine (Opioid) is causing CONTRACTION of smooth muscle cells in the SPHINCTER OF ODDI, leading to spasm and increase in common bile duct pressures.

Congestive Heart Failure: what medications IMPROVE SURVIVAL?

*Spironolactone *ACE inhibitors or angiotensin II receptor blockers *Beta-blockers (except in acute decompensated HF)

Patient has Severe aortic stenosis then develops Atrial Fibrillation. Chest x-ray shows bilateral pulmonary edema. What hemodynamic changes most likely caused this patient's acute pulmonary findings?

*Sudden DECREASE in left ventricular PRELOAD 1. Severe stenosis impairs LV output 2. higher systolic pressure causes LV HYPERTROPHY 3. Atrial contraction necessary for filling of stiffened LV 4. **Atrial Fibrillation the ATRIAL CONTRACTION IMPAIRED --> INCREASED MEAN PULMONARY VENOUS PRESSURE from buildup of blood in LEFT ATRIUM & PULMONARY VEINS 5. this leads to ACUTE PULMONARY EDEMA *these patients can also have SEVERE HYPOTENSION from DECREASED PRELOAD and SEVERE Aortic Stenosis.

A 4 month old child leaves in a home of smokers. What is she at significant risk of from exposure to cigarette smoke?

*Sudden Infant Death Syndrome* -Middle Ear disease (otitis Media) -*Asthma* -Respiratory tract infection (bronchitis, pneumonia)

IV antibiotic is startbed patient develops tinnitus and hearing loss. This antibiotic is from what class?

Aminoglycoside

Hemolytic Anemia, Thrombocytopenia, Renal Failure, Neurologic manifestations, fever. Normal PT. Normal PTT.

*Thrombotic Thrombocytopenic Purpura

Why is the Tibial Nerve not the primary concern In posterior and anterior knee dislocations?

*Tibial Nerve is also subject to injury (However it is more SUPERFICIAL than the POPLITEAL ARTERY (MAJOR concern)

Head and Neck Squamous Cell Carcinomas tend to originate from what primary sites?

*Tongue* -and within mouth (floor of mouth, lower lip, soft palate, gingiva)

High-Potency Typical Antipsychotics: Names 2. Side Effects

*Trifluoperazine, Fluphenazine, Haloperidol (Try to Fly High) 2. Extrapyramidal Symptoms (Muscle Rigidity, Acute Dystonia, Akathisia, Parkinsonism)

Patient has Type I DM insulin will lower blood glucose it also INCREASES GLYCOGEN SYNTHESIS IN HEPATOCYTES. ACTIVATION OF WHAT MOLECULES MOST LIKELY PROMOTES THIS EFFECT?

*Tyrosine Kinase/Phosphatidylinositol-3-kinase stimulation promootes glycogen synthesis by activating PROTEIN PHOSPHATASE *Protein Phosphatase

Certain cells have decreased or absent MHC class I proteins on their surfaces. Such changes in MHC I antigen expression occur in what type of cells?

*Virus infected cells *Tumor cells

Merocrine: mechanism?

*Watery Cells secrete via exocytosis

What would result if the Enzyme normally responsible for Progesterone --> 17-hydroxyprogesterone was defective?

*all patients PHENOTYPICALLY FEMALE *MINERALOCORTICOID EXCESS (HTN & Salt Retention)

Ovarian Tumor: what serum marker is the most useful for monitoring this patient?

CA 125

Renal Blood Flow is estimated by?

*basically RPF/ 1- hematocrit (YOU MUST TAKE HEMATOCRIT INTO ACCOUNT) RBF = (PAH clearance) / (1 - heamtocrit) PAH Clearance = (urine [PAH] x urine flow rate) / plasma [PAH]

Marijuana intoxication: what are the 4 most characteristics signs? 2. And other signs?

*conjunctival injection (RED EYES) *Increased appetite *Dry Mouth *TACHYCARDIA -inappropriate laughter, sedation, slowed reflexes, impaired motor coordination, distorted sensory perceptions, and cognitive impairment (decreased attention, concentration, short-term memory and judgement)

Daptomycin: MOA?

*creates transmembrane channels DEPOLARIZES cellular membranes

Pancreatic Tumor: what serum marker is the most useful for monitoring this patient?

CA 19-9

Suicide risk management involves controlling modifiable factors that increased the risk of suicide completion. What are these MODIFIABLE RISK FACTORS?

*limiting ACCESS TO FIREARMS -treating psychiatric illness -TREATING SUBSTANCE ABUSE -MANAGING PAIN -decreasing stress -improving pscyhosocial support

What causes the Vancomycin "Red Man" Syndrome?

*results from widespread HISTAMINE RELEASE (mast cell degranulation) when VANCOMYCIN is INFUSED TOO RAPIDLY *DO NOT CONFUSE THIS**** (this is not an IgE mediated allergic reaction)

The major immune mechanisms against Giardia involves?

*secretory IgA production* -CD4+ T helper cells -helps prevent and clear infection by binding to trophozoits and impairing their adherence to the upper small-bowel mucosa.

Homeobox genes: mutations cause? 2. important for development along what axis?

*severe abnormalities like skeletal malformations & IMPROPERLY POSITIONED LIMBS & APPENDAGES 2. Embryo along the CRANIO-CAUDAL axis

Why are Low Molecular-weight Heparins (LMWHs) the best choice during pregnancy when Venous Thromboembolic Disease is an issue?

*they do not cross the placenta *they have short elimination times and can be stopped shortly prior to delivery (to reduce risk of hemorrhage postpartum)

Constrictive Pericarditis

*thickening and calcification of the pericardium *Acute Pericarditis (another name)

Porcelain gallbladder: cause?

CALCIFIED (seen on imaging) due to CHRONIC CHOLECYSTITIS

Failure of fusion between the maxillary prominence and intermaxillary segment: what does this cause?

Cleft Lip

Treatment of fever, abdominal pain and discharge from the cervix must ALWAYS include?

-3rd-generation Cephalosporin (CEFTRIAXONE) --> Gonorrhea AND -Azithromycin or Doxycycline --> Chlamydia

What processes take place before the mRNA leave the nucleus?

-5'-capping -poly A tail addition -Intron Splicing

If a drug binds to GABA-A receptors in the CNS and the resting potential is typically -70 mV. What will will the new resting membrane potential be? (-75 mV OR -55 mV)?

-75 mV GABA-A receptors regulate flow of chloride ions (negatively charged) *a drug that binds to and activates GABA-A receptors will INCREASE CONDUCTANCE of CHLORIDE IONS causing membrane potential to become HYPERPOLARIZED!!

The potency of a drug is primarily affected by what?

-AFFINITY of the drug for its receptor and -amount of drug that is ABLE to REACH the target tissue

Biphosphonates: names?

-ALENDRONATE -Ibandronate -Risedronate -Zoledronate

Chronic Transmural Inflammation: associated with?

-Abdominal Aortic Aneurysm *chronic inflammation leads to LOSS OF ELASTIN and ABNORMAL COLLAGEN remodeling (these changes lead to weakening and progressive expansion of the aortic wall) -Risk Factors: advanced age (>60), smoking, HTN, and male sex

IFN-gamma: function?

-Activates Macrophages -Increases Antigen presentation

Hypertensive Emergency: what are the examples of end-organ damage?

-Acute Coronary Syndrome -Encephalopathy -Pulmonary Edema -Acute Kidney Injury -Intracranial or Sub arachnoid Hemorrhage -Aortic Dissection -Papilledema, retinal hemorrhages, or exudates

Common medical conditions with POLYGENIC inheritance?

-Androgenitic Alopecia -Type II DM -Schizophrenia -Epilepsy -Glaucoma -Hypertension -Ischemic Heart Disease

DECREASED Maternal Serum AFP?

-Aneuploidies (Trisomy 18 & 21)

Histamine (Scombroid poisoning) Fish seafood toxin: what do you treat with?

-Antihistamine -Albuterol & Epinephrine (if needed)

Cystic Medial Necrosis: associated with?

-Ascending ANEURYSM Root Dilation (may lead to dissection) *Marfan Syndrome

32 yo with pain and a mass in right flank. Large mass arising from right kidney and similar smaller masses in left kidney. Histology shows fat, smooth muscle, and blood vessels in tumor. This patient is most likely to have what additional findings?

-Ash-leaf skin patches -Brain Hamartomas *ANGIOMYOLIPOMAS *condition called TUBEROUS SCLEROSIS

During an experiment a researcher cuts unmyelinated nerve fibers. These fibers are most likely a component of what type of nerve?

-Autonomic Postganglionic Nerve -Afferent Neurons (conducting heat sensation) -Afferent Neurons (transmit slow-onset dull, burning or visceral pain) *only ones!

Hydralazine & Minoxidil are often co-administered with?

-Beta-Blocker (to prevent reflex tachycardia) -Diuretics (prevent sodium retention)

Valproic Acid: MOA?

-Blocks NMDA receptors in HIPPOCAMPAL neurons (affects POTASSIUM current) -Increases NA+ channel INACTIVATION -INCREASES GABA CONCENTRATION (INHIBITS GABA TRANSAMINASE)

If Giant Cell Arteritis (Temporal Arteritis) is suspected what lab values should be determined promptly?

-C-Reactive Protein (CRP) -Erythrocyte Sedimentation Rate (ESR)

The CLASSICAL complement cascade begins with binding of what?

-C1 Complement component to either -2 molecules of IgG or -2 molecules of IgM

Elderly patient my experience what side effects from BENZODIAZEPINES because of slower metabolism of this drug?

-CONFUSION -Anterograde Amnesia -Psychomotor Retardation

Femoral Hernia: anatomy?

-Content protrudes THROUGH FEMORAL RING -Travels INFERIOR to INGUINAL LIGAMENT

Occlusion of Anterior Cerebral Arteries can cause?

-Contralateral Motor & Sensory deficits of LOWER extremities -Behavioral Changes -Urinary Incontinence *Medial Portions of 2 hemispheres (Frontal * Parietal)

ACE-I: adverse effects?

-Cough -Angioedema -Hyperkalemia -Increased Creatinine -Teratogen (Renal Malformations)

Pseudomana Aeruginosa pneumonia in what types of patients?

-Cystic Fibrosis -Ventilated Patients

Albinism: defect?

-DECREASED TYROSINASE activity or -DEFECTIVE TYROSINE TRANSPORT

Corticosteroids: how do they cause Osteoporosis?

-DECREASED bone formation -DECREASED Calcium & Phosphorus absorption

Bruton Agammaglobulinemia (X-linke agammaglobulinemia): what are the B cells and T cells like?

-DECREASED to Absent B cells (which mean Antibodies not produced) -NORMAL T Cells

Validity (Accuracy) definition?

A test's ability to measure what it is supposed to measure

MOST IMPORTANT Proto-Oncogenes/Tumor Promoters?

ABL BRAF HER1/HER2 MYC RAS (KRAS) SIS TGFA

Sarcoidosis patients often have elevated serum levels of what?

ACE and Calcium

A patient had a small increase in their potassium from 4.8 mEq/L to 5.2. And an elevation in creatinine from 1.2 mg/dL to 1.6 mg/dL. Administration of what drug is most likely the culprit?

ACE-I (blocks conversion from angiotensin I to angiotensin II)

Zone Fasciculata: what is the major trophic hormone?

ACTH

Zone Reticularis: what is the major trophic hormone?

ACTH

Proopiomelanocorti (POMC) is a precursor to what molecules?

ACTH MSH Beta-Endorphin

In a severely dehydrated state ADH synthesis increases and improves water permeability at what part of the nephron?

ADH promotes AQUAPORIN insertion into *PRINCIPAL CELLS (DCT & COLLECTING DUCTS) Even in a severely dehydrated state the Kidney produces about 0.5 L of Urine/DAY.

Acute Intermittent Porphyria: DEXTROSE or HEME infusion improves their condition by inhibiting what enzyme?

ALA Synthase

Anorexia Nervosa hormone levels: GnRH, LH, FSH, Estrogen?

ALL DECREASED!! *FUNCTIONAL HYPOTHALAMIC AMENORRHEA (usually have menarche normal and then stop a a later age due to Anorexia)

Recurrent Laryngeal Nerve: Innervates what muscles?

ALL INTRINSIC MUSCLES OF LARYNX responsible for sound production (ARYTENOID!!) -Posterior CricoARYTENOID -Lateral CricoARYTENOID -Oblique ARYTENOID -Transverse ARYTENOID -ThyroARYTENOID

Thoracic Outlet Syndrome: what SCALENE MUSCLES are involved? 2. What rib?

ANTERIOR & MIDDLE 2. 1st Rib

Protein Kinesin is important for what?

ANTEROGRADE transport of intracellular vesicles and organelles toward the plus ends of microtubules (Plus ends typically lie at sites DISTANT FROM THE NUCLEUS!!) *so for example a defect in Kinesin could impair vesicle transportation OUT of a cell

What age does Shingles (Herpes Zoster) affect?

ANY AGE after initial varicella infection! > age 50 are at greatest risk

MOST IMPORTANT Anti-oncogenes/tumor suppressors?

APC/Beta-catenin BRCA-1/BRCA-2 DCC NF1 RB TP53 VHL WT1

Absolute Risk Reduction?

ARR = event rate in the CONTROL GROUP - event rate in the TREATMENT GROUP usually the event rates are %

Cystic Fibrosis: The dysfunctional transmembrane protein is what type?

ATP-gated

P wave?

ATRIAL Depolarization

Highest Atherosclerotic burden.

Abdominal aorta > coronary artery > popliteal artery > carotid artery

Crigler-Najjar syndrome: what causes the hyperbilirubinemia (what defect)?

Absent liver conjugation enzymes *genetic lack of the UGT (Uridine diphosphate-glucuronyltransferase) enzyme needed to catalyze bile glucuronidation

X-linked agammaglobulinemia (XLA): what is a major physical finding?

Absent/scanty lymph nodes and tonsils.

Hematocrit > 48% in a woman but RBC Mass is Increased what does this mean?

Absolute Erythrocytosis

Epidermal Hyperlasia (INCREASED SPINOSUM)?

Acanthosis Nigricans

Women with strong family history of ovarian and uterine cancer. She has a biopsy of axillary lymph nodes. Cells are noted to overexpress a 185 kD glycoprotein. This protein spans the cell membrane and has intracellular tyrosine kinase activity. What is the most likely function of this protein?

Accelerates Cell Proliferation *HER2/neu oncogene encodes for a 185-kD transmembrane glycoprotein with intracellular TYROSINE KINASE

Neoplastic polyps?

Adenomatous Polyps (2 types) *Villous Adenomas (HIGHEST RISK) *Tubular Adenomas

Woman with heavy and painful menstrual bleeding for 3 months. Uterus is uniformly enlarged. Diagnosis?

Adenomyosis

Emotional symptoms (anxiety, depression) causing impairment following an identifiable psychosocial stressor (eg, divorce, illness, moving)

Adjustment Disorder

Tamoxifen indication?

Adjuvant treatment of breast cancer

Neuroblastoma: most common location?

Adrenal Medulla

Young boy 4 yo old develops pubic hair, his penis is growing, and he has axillary odor. Lab assesment reveals high serum concentrations of 17-hydroxyprogesterone and testosterone. Treatment is directed at suppressing what hormones? 2. How?

Adrenocorticotropic hormone (ACTH) *this is 21-Hydroxylase Deficiency 2. give low (physiologic) doses of exogenous CORTICOSTEROIDS to suppress ACTH secretion which will remove the excessive ACTH stimulation. ACTH is overstimulated because ALDOSTERONE production and Cortisol production is DECREASED.

Post-Streptococcal Glomerulonephritis: what characteristic at presentation is the MOST important POOR PROGNOSTIC FACTOR?

Adult Onset (INCREASED AGE) *only 60% of cases resolve completely. the rest develop Chronic Hypertension, Recurrent Proteinuria, Chronic Renal Insufficiency, or Rapidly Progressive GN.

Patient has multiple small, nondestructive masses attached the edges of the mitral valve leaflet. Microscopy reveals they are platelet-rich thrombi, but cultures reveal NO BACTERIAL growth. What is this most commonly associate with?

Advance Malignancy *Nonbacterial Thrombotic Endocarditis (NBTE)

An organism is found to have a methyltransferase that methylates ribosomal RNA. This enzyme confers resistance to what class of antibiotics?

Aminoglycosides

An Enterococcus organism produces an enzyme that is involved in transfer of different chemical groups (ACETYL, ADENYL, PHOSPHATE groups) to the aminoglycoside antibiotic molecule outside of the bacterium 1. The bacteria are most likely resistant to what class of antibiotics?

Aminoglycosides -Gentamicin, -Neomycin, -Amikacin, -Tobramycin, -Streptomycin.

Listeria Monocytogenes: treatment of choice?

Ampicillin

Huntington's Disease: what substances are decreased?

DECREASE in GABA DECREASE in substance P DECREASE in Acetylcholine

Osteoarthritis: What is the most significant risk factor for this condition?

Age

When does maternal age become a factor in decreased fertility?

Age 35

Carbamazepine: most feared, though uncommon side affect?

Agranulocytosis or Aplastic Anemia

What is often found with Gallstone Ileus?

Air in the biliary Tract *Gallstone Ileus: is a mechanical bowel obstruction caused from a large gallstone erodes into the intestinal lumen

What is the major amino acid responsible for transferring nitrogen to the liver for disposal?

Alanine

Perianal itching, tape test reveals eggs: treatment

Albendazole (1st line) -pyrantel pamoate (pregnant) *PINWORM (Enterobius Vermicularis)

POOR Melanin formation in melanocytes: diagnosis?

Albinism

What should be avoided while taking the abx Metronidazole?

Alcohol

Patient requests medication for severe alcohol addiction she is recovering from. She is an abstinent patient and has strong motivation to maintain abstinence. A medication with what mechanism of action would be most effective in this patient?

Aldehyde Dehydrogenase Inhibition *Disulfiram *this is ONLY USED if patient is ABSTINENT with STRONG MOTIVATION TO STAY ABSTINENT *If they drink ACETALDEHYDE accumulates and causes N/V, flushing, sweating, headache, dyspnea, lowered BP, palpitation

High bloods levels of glucose may result in formation of sorbitol in tissues. What forms Sorbitol from glucose?

Aldose Reductase

What converts Galactose to Galactitol?

Aldose Reductase

urine turns black on prolonged exposure to air: diagnosis?

Alkaptonuria

Acute Promyelocytic Leukemia: treatment?

All-trans-retinoic acid (ATRA) a *VITAMIN A* derivative

What type of liver injury will you see prolonged prothrombin time?

All-types

Disease that result when different mutations in the same genetic locus cause similar phenotypes?

Allelic Heterogeneity

Some patients were found to have missense mutation in exon 1, a few with nonsense mutation in exon 3, and others with promoter site mutation. However all have similar clinical manifestations of the the disease? What genetic principle best explains this?

Allelic Heterogeneity

Atopic Dermatitis (Eczema): these patients have risk of developing what?

Allergic Rhinitis Asthma Atopic Dermatitis (Eczema) *Allergic Triad

Hyperphosphorylated (Inactive) of Retinoblastoma (Rb)?

Allows the damaged cell to ENTER MITOSIS

MHC Class II is made of?

Alpha & Beta polypeptide Chains

Hepatocellular Carcinoma: what serum marker is the most useful for monitoring this patient?

Alpha Fetoprotein

Germ Cell Tumors: what serum marker is the most useful for monitoring this patient?

Alpha Fetoprotein hCG

What subunit of hCG is similar to LH?

Alpha subunit of hCG is structurally similar to LH *stimulates the LH surge and induces ovulation

Adrenergic Receptor that causes Mydriasis?

Alpha-1 Agonist

Before Alanine can be converted to glucose its amino group is transferred to what?

Alpha-ketoglutarate

Checks differences between means of 3 or more groups?

Analysis of Variance (ANOVA)

Fine-needle biopsy of thyroid: Pleomorphic giant cell nests with occasional multinucleated cells?

Anaplastic Thyroid Cancer

PaO2: NORMAL SaO2: NORMAL Oxygen Content: LOW What is the likely cause of these results?

Anemia (Chronic blood blood, other other causes of anemia)

Hereditary Spherocytosis: What complications (2) can be prevented by performing a splenectomy?

Anemia (improves) Gallstones (Bilirubin Gallstones from hemolysis)

Women had a mastectomy 10 years ago with axillary lymph node dissection. Post-operatively, the patient developed chronic lymphedema of her arm. Now she has multiple firm violaceous nodules. What is the the skin lesion?

Angiosarcoma *Chronic Postmastectomy Lymphedema *appears that the entity arises from blood vessels (not lymphatic vessels)

Zona Glomerulosa is primarily regulated by?

Angiotensin II *stimulates production of Aldosterone

Levels of which of the following substances is higher in the pulmonary vein compared to the pulmonary artery? (Angiotensin I, Angiotensin II, Angiotensinogen)

Angiotensin II Angiotensin I converted by ACE primarily in PULMONARY VASCULATURE *ACE is ENDOTHELIAL BOUND in Pulmonary vasculature

Patient receiving treatments for allergic rhinitis has flushed cheeks and dilated pupils. These findings are best explained by Antagonism of what receptors?

Antagonism of MUSCARINIC receptors *patient probably receiving H1 receptor antagonist (Diphenhydramine) *may also cause inhibition of eccrine sweat gland secretion

Mamillary bodies anterior to midbrain or posterior?

Anterior *boobs in front

Patient has been getting excessively hot. What hypothalamus nuclei may be involved?

Anterior Hypothalamus *Anterior nucleus= cool off (cooling , pArasympathetic). A/C = anterior cooling *ZAPPED your AC

CYP450 Substrates?

Anti-epileptics Theophylline Warfarin OCP Always Think When Outdoors

Primary Biliary Cirrhosis: positive for what antibody?

Anti-mitochondrial antibody

Truncus Arteriosus is an embryological derivative of what?

Ascending Aorta & Pulmonary Trunk

Leg pain, red spots on legs, bad diet, gums are swollen and tender, subperiosteal hematoma. What nutrient deficiency is this?

Ascorbic Acid (Vitamin C) *Scurvy *PETECHIAL HEMRRHAGES, HEMARTHROSIS, impaired wound healing, GINGIVITIS, MYALGIAS

Septate Hyphae with dichotomous branching

Aspergillus Fumigatus

Aflatoxins are produced what by organism?

Aspergillus species

Medication that inhibits COX-1 & COX-2 enzymes via IRREVERSIBLE ACETYLATION?

Aspirin

Absent P waves, irregularly irregular R-R intervals, and narrow QRS complexes: what is that?

Atrial Fibrillation

Jugular Venous Tracing: A wave is notably absent in patients with what?

Atrial Fibrillation

Aplasia of the Atrial Septum Secundum: what is this defect?

Atrial Septal Defect (ASD)

What is indicated in the treatment of Bradycardia?

Atropine *Atropine BLOCK VAGAL INFLUENCE on SA & AV nodes --> INCREASES HR

Inferior Colliculi is part of what pathway?

Auditory pathway

The medial geniculate bodies is part of what pathway?

Auditory pathway

How do you kill the SPORES formed by Bacillus and Clostridium?

Autoclaving

Patient has a mass in his right lung with neurologic symptoms of dizziness, dysarthria and bilateral limb ataxia. At pt death, the autopsy shows extensive cerebellar Purkinje Cell degeneration. What best describes the etiology of these neurologic conditions?

Autoimmune *Subacute Cerebellar Degeneration *Small cell lung cancer, Breast, Ovarian, and Uterine cancer *etiology --> Anti-Yo, anti-P/Q & anti-Hu ANTIBODIES

Eye pain and decreased vision and intention tremor: patient's condition most likely caused by?

Autoimmune Disease (Multiple Sclerosis) *Optic Neuritis (eye pain & decreased vision)

Hashimoto Thyroiditis: cause?

Autoimmune destruction of thyroid gland

Acute Intermittent Porphyria: inheritance?

Autosomal Dominant

Frontotemporal Dementia: Inheritance?

Autosomal Dominant

Li-Fraumeni Syndrome: inheritance?

Autosomal Dominant

Neurofibromatosis Type 1: inheritance?

Autosomal Dominant

von Willebrand Disease (vWD): inheritance?

Autosomal Dominant

Homogentisic Acid Dioxygenase Deficiency: inheritance?

Autosomal Recessive

Avoidance due to fears of criticism and rejection?

Avoidant Personality Disorder

Limited social relationship due to fears of being judged, embarrassed or rejected. Struggle with feelings of inadequacy and pursue relationships only when they feel assured of uncritical acceptance.

Avoidant Personality Disorder

Surgical Neck of the Humerus: Artery and Nerve may be damaged?

Axillary Nerve Posterior & Anterior Circumflex Humeral Arteries

Skin Above Umbilicus: what lymph nodes?

Axillary Nodes

Transudates: LDH & protein levels?

DECREASED

Defects in any of the urea cycle steps result in disorders of the Urea Cycle. Treatment of Urea Cycle disorders consists of what?

Balancing DIETARY PROTEIN INTAKE & PROTEIN OUTPUT *PROTEIN RESTRICTION! (body still need essential AA needed for growth and development but not in excess so that EXCESSIVE AMMONIA is formed.)

Thiopental is what class of drug?

Barbidurate

Fever, Tender regional lymphadenopathy (often axillary), scratches on arm, works as a veterinarian. Organism?

Bartonella Henselae *Cat-scratch disease

Man has enlarged lymph node in anterior cervical chain. Biopsy reveals abnormal lymph node architecture and numerous lymphocytes and POLYCLONAL lymphocytic proliferation: this is strong evidence for malignant or benign lymph node?

Benign *polyclonal proliferation of lymphocytes

Brief episodes of vertigo brought on by HEAD MOVEMENT. No auditory symptoms: Diagnosis

Benign Paroxysmal Positional Vertigo (BPPV)

Extrapyramidal System Side Effects from Typical Antipsychotics: treatment?

Benztropine (Muscarininc Antagonist)

Patient with acute MI and obstructive lung disease what beta blocker should you use?

Beta Blocker-1 (Cardioselective) -Metoprolol -Atenolol -Bisoprolol -Nebivolol *Asthma, COPD

Acute Obstructive pulmonary exacerbations: treatment?

Beta-2 Adrenergic Receptor

Adrenergic receptor that would inhibit uterine contraction (tocolysis)?

Beta-2 Agonist

Pentapeptide with a strong affinity to delta- and mu-receptors?

Beta-Endorphin

Italian man with microcytic anemia and increased hemoglobin A2 concentration. Blood smear shows hypochromia and poikilocytes (including spherocytes and target cells).

Beta-Thalassemia Minor *microcytic anemia is often INCORRECTLY attributed to IRON DEFICIENCY -> IRON SUPPLEMENTS DO NOT HELP THESE PATIENTS *most common thalassemia in patients of Mediterranean descent

Patient has acute myocardial infarction complicated by acute severe heart failure. Started on DOBUTAMINE. What is the MOA for this drug?

Beta-adrenergic agonist *predominantly BETA-1 (positive INOTROPIC effect) *minimal activity on Beta-2 & Alpha-1 (weakly chronotropic effect)

Hydatidiform mole (complete): what should be closely monitored in the patient after a dilation and curettage is performed?

Beta-hCG *persistently rising levels may signify the development of an INVASIVE MOLE or CHORIOCARCINOMA

Progressive neurodegeneration, macular cherry-red spot: what is the deficiency?

Beta-hexosaminidase A *Tay-Sachs *GM2 (ganglioside) Accumulates

Acute Stress Disorder: Time frame?

Between 3 days and 1 month -those are both greater than or equal to signs

In Acidotic states what is completely reabsorbed from the tubular fluid?

Bicarbonate Urine HCO3- will be SIGNIFICANTLY DECREASED

What drug prevents intestinal reabsorption of bile acids; liver must use cholesterol to make more?

Bile acid resins -Cholestyramine -Colestipol -Colesevelam

Newborn presenting within first 2 months of life. Elevated Direct Bilirubinemia. Jaundice, dark urine, acholic stools.

Biliary Atresia (OBSTRUCTION of EXTRAHEPATIC BILE DUCTS) *DARK URINE *ACHOLIC STOOLS *CONJUGATED HYPERBILIRUBINEMIA

Skin lesion of the eyelid is biopsied. Light microscopy shows lipid-laden macrophages. These are associated with liver/gallbladder issues?

Biliary Tract Diseases (cause INCREASED CHOLESTEROL) *Primary Sclerosing Cholangitis *Primary Biliary Cirrhosis *Secondary Biliary Cirrhosis

Brown Pigmented gallstones: cause?

Biliary Tract Infection

NNRTI: MOA?

Bind to reverse transcriptase at site different from NRTis. *have VIR in the name

Regular episodes of excessive uncontrollable eating WITHOUT compensatory behaviors?

Binge Eating Disorder

Preferred initial treatment for osteoporosis?

Biphosphonates

Which type of pigment stones are radiopaque?

Black Pigment *Calcium Bilirubinate

STEMI in the Inferior leads: what is a complication that result?

Bradycardia *Right Coronary Artery (RCA) is responsible for SA & AV node perfusion

Women: cancer with highest incidence?

Breast Cancer (aside from skin cancer)

Phyllodes Tumors of the breast arise from what part of the breast?

Breast STROMA

Diabetic Ketoacidosis, facial pain, headache, black necrotic eschar on inferior turbinate. What is seen on histology?

Broad non-septate hyphae with right angle branching *Mucormycosis (Mucor or Rhizopus)

What type of pigment stones are radiolucent?

Brown Pigment

Regular episodes of excessive uncontrollable eating WITH compensatory behaviors?

Bulimia Nervosa

21 yo female, headaches, irregular menses, smokes a pack of cigarettes a day. BP 90/60, pulse 118. BMI is 25. Enlarged and firm parotid glands bilaterally. Sodium is mildly low. Teeth are eroded: Diagnosis?

Bulimia Nervosa *self-induced vomiting, using laxatives or diuretics, fasting, excessive exercise. *PAROTITIS *METABOLIC DISTURBANCES *ENAMEL EROSION *ALKALOSIS

Portal Hypertension in an alcoholic produces splenomegaly how?

By causing CONGESTION of blood within the spleen, which effectively produces an EXPANSION OF THE RED PULP

Phrenic Nerve contains what nerve roots?

C3-C5

Brachial Plexus originates from what spinal nerves?

C5-T1

Atenolol: what type of beta blocker?

Cardioselective Beta Blocker

Metoprolol: what type of beta blocker?

Cardioselective Beta Blocker

Williams Syndrome: associated with what major organ problem?

Cardiovascular

Pituitary Apoplexy: what does these patients often die of once the hemorrhage starts?

Cardiovascular Collapse because of ADRENOCORTICAL DEFICIENCY

Malonyl-CoA is responsible for inhibiting Beta Oxidation...What does it inhibit?

Carinitine Acyltransferase

Chronic Granulomatous Disease: what type of organisms affect these patients?

Catalase producing organisms

What tissues are sensitive to damage by Sorbitol accumulation?

Cataracts Retinopathy Peripheral neuropathy

What is the structure affected by Huntington Disease?

Caudate Nucleus *Chorea and athetosis

Huntington's Disease: where is the neuron damage?

Caudate Nucleus & Putamen

Liver biopsy shows cavernous, blood-filled vascular spaces of variable size lined by a single epithelial layer: Diagnosis?

Cavernous Hemangioma *Most COMMON BENIGN LIVER TUMOR

Duodenal biopsy that shows villous atrophy and crypt hyperplasia?

Celiac Disease

Most likely younger child with Diarrhea (Steatorrhea), Flatulence, Nutrient Deficiencies, Weight Loss: diagnosis?

Celiac Disease *also a biopsy may be shown

Liver, Stomach, Spleen, Pancreas: what lymph nodes?

Celiac Nodes

Upper Duodenum: what lymph nodes?

Celiac Nodes

Holocrine: mechanism?

Cell lysis releases entire contents of the cytoplasm and cell membrane

Motile Gram-positive rods that produce a very narrow zone of Beta-hemolysis on blood agar. Bacteria grows well at a refrigeration temperature. What processes is most important in eliminating these bacteria from body? (Cell-mediated immunity, Immunoglobulin secretion, Terminal complement cascade)

Cell-mediated immunity *Tumbling MOTILITY *INTRACELLULAR

Apocrine: mechanism?

Cells secrete via membrane-bound vesicles containing secretory product

Cervical biopsy reveals low-grade cervical intraepithelial neoplasia. What is the most likely finding on histopathology?

Cells with nuclear atypia only in the basal epithelial layer (Cells have not invaded past the lower one-third of the cervical epithelium)

Ferritin?

Cellular iron storage protein and en excellent marker or iron stores.

Cherry Red spot at the macula in an elderly patient: cause?

Central Retinal Artery Occlusion -atheroscloerosis -cardioembolic disease -vasculitis (giant cell arteritis)

Macular lesions usually result in what type of vision loss?

Central Scotomas (impaired central Vision)

What type of emphysema to smokers get?

Centriacinar Emphysema

Antibiotics that bind to: Transpeptidases?

Cepahlosporins Penicillins

How does the presentation differ between Classic Galactosemia and Galactokinase Deficiency?

Classic Galactosemis: **FAILURE TO THRIVE Galactokinase Def: less severe

GABA-A receptor regulates the flow of what ions? Are these ions negative or positively charged?

Chloride ions (NEGATIVELY CHARGED)

Somatostatins decrease the secretion of what hormones?

Cholecystokinin Secretin Glucagon Insulin Gastrin

Patient had surgery and a physician administers a new drug. Patient now has flushing, diaphoresis and nausea, BP is 100/70, HR 55. Constricted pupils but reactive to light. What was the mechanism of action of the drug?

Cholinergic Agonist

Anterior horn cells of the spinal cord demonstrate cell body rounding, peripheral displacement of the nuclei, and dispersion of Nissl substance to the periphery of the cells. What is this called? What does it indicate?

Chromatolysis (Axonal Reaction) *occurs in Wallerian Degeneration to Proximal to injury

CYP450 Inducers?

Chronic Alcohol Use St. Johns Wort Phenobarbital Phenytoin Nevirapine Rifampin Griseofulvin Carbamazepine Chronic Alcoholics Steal Phen-Phen and Never Refuse Greasy Carbs

Failure of neutrophils to turn blue on Nitroblue Tetrazolium testing?

Chronic Granulomatous Disease

Black Pigmented Gallstones: cause?

Chronic Hemolysis (sickle cell anemia, Beta-thalassemia, hereditary spherocytosis or increased enterohepatic cycling of bilirubin -> ILEAL DISEASE)

Leukocyte Alkaline Phosphatase test score is LOW: what disease?

Chronic Myelogenous Leukemia *usually LOW in CML but may not be given either

What type of liver injury will you see DECREASED SERUM ALBUMIN?

Chronic, end-stage liver disease

Which of the following drugs DO NOT require a viral kinase (thymidine kinase) for conversion to their active form? Acyclovir Ganciclovir Famviclovir Valacyclovir Foscarnet Cidofovir

Cidofovir Foscarnet

What drug would you use for a VZV herpes rash if the viral strains isolated from the patient are found to lack viral phosphorylating enzymes?

Cidofovir Fosccarnet

What drug causes direct DILATION OF ARTERIES and INHIBITION OF PLATELET AGGREGATION?

Cilostazol or Dipyridamole *VASODILATION IS HUGE!!

Patient is on Warfarin for A-Fib then is given Cimetidine: What happens?

Cimetidine: Inhibitor of P-450 *SLOWS the metabolism of Warfarin resulting in a GRADUAL INCREASE of the PROTHROMBIN TIME over a PERIOD OF SEVERAL DAYS

16 yo Patient has difficulty falling asleep at night and excessive daytime sleepiness. Had these symptoms for several years. Typically doesn't fall asleep until 4:00 AM but sleeps comfortably once he does. Parents have to wake him for school. Grades are declining. On weekends he sleeps until the afternoon. Father had a similar behavior at that age. What is the diagnosis?

Circadian Rhythm Disorder *the Suprachiasmatic Nucleus in the Hypothalamus (the internal clock) is not in sync with the normal 24-hour light-dark cycle. Work or school often forces them awake before their sleep cycle is complete and makes them excessively sleep during the day. So they complete a full sleep cycle on the weekends.

Class 1 antiarrhythmics: which drugs bind LESS avidly to sodium channels

Class 1B -Lidocaine -Mexiletine -Tocainide

Patient is taking medication for Atrial Fibrillation. If the QRS complex duration is shorter at rest and significantly increased during exercise what medication class is used?

Class 1C *these block the INWARD SODIUM current and this would PROLONG QRS DURATION *Class 1C binds the STRONGEST to sodium channel out of all the Class 1

Class 1 antiarrhythmics: which drugs bind MORE avidly to sodium channels

Class 1C -Flecainide -Propafenone

HLA-B27: Class I or Class II Serotype human leukocyte Antigen?

Class I

Dofetilide is what class of antiarrhythmic drug?

Class III antiarrhythmic

Patient has Paroxysmal Atrial Fibrillation and is started on medication to reduce his symptoms. T2 weeks later he has QTc prolongation, and a short episode of Torsades de Pointes. What drugs could cause this?

Class Ia (Quinidine, Procainamide, Disopyramide) Class III (**SOTALOL**, Ibutilide)

What is the most commonly fractured bone?

Clavicle

Steroid Drug Abuse in women?

Clitoromegaly Hirsutism Deepening of voice Atrophy of breast *basically become a man

Alcoholic with numbness of his feet and decreased vibratory sensation over the feet and ankles: what is causing these complaints?

Cobalamin Deficiency

Agitation, Tachycardia, Hypertension, and dilated pupils that are reactive to light : intoxication of what?

Cocaine *light-responsive mydriasis

Chest pain, tachycardia, blurry vision, euphoria, agitation typically lasting an hour or less. What substance?

Cocaine Intoxication

Spherules containing endospores are found in tissue samples: what organism?

Coccidiodes Immitis

Multinucleated Spherule: organism?

Coccidioides Immitis (anything Spherule is this; no other organism is FA says Spherule)

Diabetic Ketoacidosis: Total body potassium level?

DECREASED (due to urinary losses and possible GI losses from vomiting)

3 polypeptide ALPHA CHAINS which form a rope-like TRIPLE HELIX

Collagen

Ehlers-Danlos syndrome is caused by an inherited defect in what protein?

Collagen

20 yo with low bone mineral density, mild scoliosis, thin sclerae with visibility of underlying choroid layers. What proteins is improperly synthesized in this patient? *Diagnosis?

Collagen *Osteogenesis Imperfecta (OI)

The principal site of uric acid precipitation would be which of the following?

Collecting ducts due to low urine pH

Carvedilol: what type of drug?

Combined beta and alpha receptor blockers

Labetalol: what type of drug?

Combined beta and alpha receptor blockers

Where is the optimal site for cardiac catheterization?

Common Femoral Artery BELOW Inguinal Ligament *above the ligament can significantly increase risk of retroperitoneal hemorrhage

Biopsy of a skin lesion reveals nests of uniform round cells with inconspicuous nucleoli and few mitotic figures at the basal portion of the epidermis and upper dermis: Diagnosis?

Compound Melanocytic Nevus

Decreased Outward potassium flow and resultant prolongation of the action potential is suggestive of what syndrome?

Congenital Long QT Syndrome

Cri-du-chat syndrome: genetics?

Congenital microdeletion of short arm of chromosome 5

Williams Syndrome: genetics?

Congenital microdeletion of short arm of chromosome 7

Lung that is red, heavy and boggy: what phase of lobar pneumonia?

Congestion

Microscopic Appearance: alveoli contain mostly bacteria?

Congestion

Pioglitazone: can exacerbate what underlying condition?

Congestive Heart Failure *FLUID RETENTION --> due to INCREASED SODIUM REABSORPTION

Cheyne-Stokes Breathing: seen in what major disease?

Congestive Heart Failure -also seen in stroke, brain tumors, traumatic brain injury *poor prognostic sign

One-way transfer of chromosomal or plasmid DNA between bacterial cells through direct physical contact. Donor cells either have F FACTOR or SEX PILUS. what is this called?

Conjugation

When do you see Hyperplasia of the GLOMERULAR layer of the cortex?

Conn's Syndrome (primary hyperaldosteronism)

Jugular Venous Tracing: y descent is PROMINENT in what?

Constrictive Pericarditis

Listeria Monocytogenes: how is it transmitted?

Contaminated Food -Unpasteurized Milk -Undercooked Meats -Unwashed raw vegetables

Facial Nerve palsy: if the forehead muscle contraction is spared where is the lesion?

Contralateral Upper Motor Neuron Lesion (UMN) *bilateral UMN innervation

Thrombin: function?

Converts Fibrinogen --> Fibrin *promotes coagulation

Left Horn of Sinus Venosus is an embryological derivative of what?

Coronary Sinus

What is the mechanism used to clear the respiratory tract of deposited particles 10 -15 micrometers in size?

Coughing, sneezing, and upper airway trapping *Largest Dust particles

Microcephaly and high-pitched crying, epicanthal folds, VSD?

Cri-du-chat Syndrome

Budding Yeast with a thick capsule?

Cryptococcus Neoformans

Ulcers arising in the setting of severe trauma/burns are called?

Curling's Ulcers *Local ISCHEMIA --> DECREASED MUCOSAL protection

Ulcers arising from intracranial injury are called?

Cushing's Ulcers *caused by direct VAGAL stimulation --> increased ACID secretion

Industry worker develops reddish discoloration of skin, tachypnea, headache, and tachycardia. Lab studies reveal severe lactic acidosis. What substance has caused these symptoms?

Cyanide Poisoning

PaO2: NORMAL SaO2: NORMAL Oxygen Content: NORMAL

Cyanide Poisoning

Serotonin Syndrome: what is antidote?

Cyproheptadine (5-HT2 receptor antagonist)

Physical exam reveals a child has a grey pharyngeal exudate and gram stain shows gram positive organisms. Child has not had vaccinations. What culture type would facilitate the growth of this organism?

Cysteine-Tellurite Agar

In more than 90% of cases Acute Cholecystitis is caused by gallstone obstruction where?

Cystic Duct

Immunocompromised patient with Esophagitis, endoscopy shows linear ulcers: organism?

Cytomegalovirus (CMV)

Heme synthesis occurs partly in the mitochondria and partly in what?

Cytoplasm

Pentose Phosphate Pathway: Cytoplasm or Organelle?

Cytoplasm

Transketolase: Cytoplasm or Organelle?

Cytoplasm

Elderly man with low-grade fevers, and fatigue. No history of heart disease Has a diastolic murmur at left sternal border. Ha nontender macules on his hands and feet. Blood culture reveals gram-positive cocci that are catalase negative and able to grow in hypertonic 6.5% saline and bile. This patient's medical history is most likely to include what procedure in the past month?

Cytoscopy (most common) Colonoscopy (GI) Obstetric procedures also PYR-positve*

If Preproinsulin had several base pair deletions in the N-terminal region that consists of hydrophobic amino acids. What cellular would you likely find the accumulation of hormonal precursors?

Cytosol

If the Pancreas is removed why can a patient still absorb D-Xylose?

D-Xylose is a MONOSACCHARIDE *Amylase is needed for monosaccharides and polysaccharides.

Penicillin has structural similarities with which of these (D-alanine-D-alanine OR L-alanine-Dlglutamine)?

D-alanine-D-alanine

RUBELLA during PREGNANCY: What is the child at risk for?

DEAFNESS Cataracts *PDA*

Acute Pancreatitis: what does the pancreas look like grossly?

Edematous

Tamoxifen adverse effect?

Endometrial hyperplasia & Carcinoma

Guillain-Barre Syndrome is characterized by?

Endoneural Inflammatory Infiltration

Xeroderma Pigmentosum: defect in what enzyme?

Endonuclease *NOT Exonuclease

*N-terminal peptide signal sequences with hydrophobic regions signal them to what Cytosolic organelle?

Endoplasmic Reticulum *If this region is mutated they would probably accumulate in the Cytosol

Severe hypertension in retinal precapillary arterioles causes what pathologic changes?

Endothelial disruption, leakage of plasma into the arteriolar wall, and FIBRINOUS NECROSIS

Pseudomonas Aeruginosa: toxin?

Endotoxin (fever, shock) Exotoxin A (inactivates EF-2)

HIV drugs that are fusion inhibitors?

Enfuvirtide (Binds gp41) Maraviroc (Binds CCR-5 inhibiting interacting with gp120)

Viral meningitis in a 6 yo?

Enterovirus (especially Coxsackievirus)

What type of virus accounts for >90% of viral meningitis cases?

Enteroviruses

Herpes Simplex Virus Type 1 (HSV-1): virology?

Enveloped - dsDNA *all HERPESVIRUS

Pilocytic Astrocytoma: Histology?

Eosinophilic, corkscrew fibers

Molluscum Contagiosum biopsy will show?

Epidermal hyperplasia with MOLLUSCUM BODIES

Closed-Angle Glaucoma (Narrow-angle Glaucoma): do not give?

Epinephrine *because of its Mydriatic effect (worsens the problem)

What drug causes Increased Contractility and Decreased Renal Blood Flow?

Epinpehrine *stimulates both alpha-1 (DECREASES renal blood flow) & Beta-1 (increased contractility)

True Cysts have walls lined with?

Epithelial cells

Reid Index?

Epithelium --> Stop at the Cartilage

A superficial skin lesion characterized by a painful erythematous plaque with raised, well demarcated borders.

Erysipelas *Streptococcus Pyogenes (GAS)

Parvovirus B19 replicates in progenitors of what cell line AND where?

Erythrocytes in Bone Marrow

What other GI malignancies are frequently identifies as ulcerative lesions on endoscopy and MUST BE BIOPSIED FOR CONFIRMATION?

Esophageal Cancers Gastric Cancers Colorectal Cancers *these are frequently identified as ULCERATIVE LESIONS

Asymptomatic patient with history of an inborn error of sugar metabolism most likely has?

Essential Fructosuria *Deficient Fructokinase *I chose Lactase Def. However these patients have BLOATING, DIARRHEA. That is not asymptomatic.

Pulsus Paradoxus: what?

Exaggerated drop in SBP (>10 mm Hg) during INSPIRATION (decreased Systolic PULSE Pressure during inspiration

Introns or Exons are the part of the mRNA that is kept for protein translation?

Exons

What must be removed from pre-mRNA for protein translation to occur?

Exons

Cervical biopsy reveals high-grade cervical intraepithelial neoplasia. What is the most likely finding on histopathology?

Expansion of immature basal cells to the epithelial surface (BEYOND THE LOWER ONE-THIRD OF CERVICAL EPITHELIUM)

People with the disease of interest and people without this disease are asked about previous exposure to the variable being studied. What measurement are investigators likely to report?

Exposure Odds Ratio *Case-Control Study (The exposure of people with the disease (cases) to the exposure of people without the disease (controls)) "Looks for prior exposure or risk factor"

Lymph Nodes: Superior portion of bladder?

External Iliac Nodes

Direct Hernias: are covered by what?

External Spermatic Fascia

Fas receptor is important for?

Extrinsic pathway of apoptosis

What drug prevent cholesterol absorption at small intestine brush border?

Ezetimibe

Conjugation: bacteria that lack what serve as the recipient cells?

F FACTOR: are recipient

Where is the Macrophage binding site on the immunoglobuline molecule?

FC REGION of the HEAVY immunoglobulin chain

Achondroplasia: gene mutated?

FGFR3 gene mutation

Rapidly Progressive Glomerulonephritis: crescents consists of?

FIBRIN Glomerular Parietal cells Monocytes Macorphages Type I (Goodpastures): IgG & C3b Pauci-Immune: No IgG & C3b

Why are Pheochromocytomas EPISODIC in nature?

FLUCTUATIONS in Catecholamine synthesis by the tumor

Patient comes to ED with 3-day history of persistent headaches. Has PMH of HTN. BP is 224/115. Bilateral Papilledema. Serum creatinine is 1.4. Pt appears mildly confused. Pt started on an IV medication that causes arteriolar dilation, improves renal perfusion, and increases natriuresis. Repeat BP an hour later is 180/90 with improvement of symptoms. What drug is most likely used in this patient?

Fenoldopam *Hypertensive Emergency --> Increased Creatine --> Papilledema --> Confusion (Encephalopathy)

Iron Deficiency Anemia: Ferritin high or low?

Ferritin Low

What drug activates PPAR-alpha to induce HDL synthesis?

Fibrates (Fenofibrate, Gemfibrozil)

What drug inhibits Cholesterol 7alpha-hydroxylase leading to increased Cholesterol excretion?

Fibrates (Fenofibrate, Gemfibrozil)

What drug upregulates Lipoprotein Lipase?

Fibrates (Fenofibrate, Gemfibrozil) *this will DECREASE TRIGLYCERIDES!!

Alteplase & Tenecteplase are what type of medications?

Fibrinolytics *Percutaneous Coronary Intervention (PCI) is PREFERRED OVER Fibrionlytics *these have risk of Intracerebral Hemorrhage and RECURRENT MI (but PCI may not be available everywhere)

Breast biopsy: Benign-appearing cellular or myxoid stroma that encircles epithelium-lined glandular and cystic spaces.

Fibroadenoma

The most common benign tumor of the breast, typically arising in young women age 15-35?

Fibroadenoma

Acute Pancreatitis: A Pseudocyst is lined by?

Fibrous & GRANULATION TISSUE *not epithelium

Recurrent Clostridium Difficile infection: treatment?

Fidaxomicin or Fecal Microbiota Transplant

Histology evaluation of hyperemic mucosa seen during GI endoscopy reveals hyperplasia of branched, tubular submucosal glands containing alkaline secretions. What area is the most likely site of biopsy in this patient?

First part of the duodenum *Submucosal BRUNNER GLANDS (secrete copious amounts of alkaline mucus into the duodenum)

A constant proportion of the drug is metabolized?

First-Order drug metabolism *constant proportion (fraction or %)

PO2 readings of 65 mm Hg in Right Atrium and 65 mm Hg in Right Ventricle: what will be heard on auscultation?

Fixed splitting of S2 on cardiac auscultation *Atrial Septal Defect (ASD)

Potter's Sequence?

Flattened Facies Limb Deformities Pulmonary Hypoplasia *not Vertebral Deformities

Class IC antiarrhythmic drugs?

Flecainide Propafenone

Atopic Dermatitis (Eczema): Where does it appear in Older Children?

Flexural Distribution (Neck, wrists, antecubital and popliteal fossa)

Candida Vaginitis (Candida Albicans): treatment?

Fluconazole

Candida Vaginitis: treatment?

Fluconazole

Bacteria are drug resistant due to mutations in the DNA gyrase or toposiomerase genes. 1. The bacteria are most likely resistant to what class of antibiotics?

Fluoroquinolone

Impaired Androgen receptor interaction: drugs (3)?

Flutamide Dyproterone Spironolactone

Alcoholic with a hypersegmented neutrophil?

Folate Deficiency *Alcoholism is one of the most COMMON causes of Folate Deficiency. *A patient who consumes large amounts of alcohol will experience anemia within a FEW WEEKS with a poor diet.

Methotrexate: MOA?

Folic Acid Antagonist

What part of the lymph node houses B Cells?

Follicles (GERMINAL CENTERS)

bcl-2 gene

Follicular Lymphoma

Middle Meningeal Artery: what foramen?

Foramen Spinosum

FMR1 gene?

Fragile X Syndrome (Fragile X Mental Retardation 1)

Trinucleotide repeats to remember: CGG, GAA, CAG, CTG?

Fragile X syndrome = (C G G) Friedreich ataxia = (GAA) H untington disease= (CAG) Myotonic dystrophy = (CTG) *X-G irlfriend's F irst A id H elped Ace My T est.

Autopsy of a infant born at 25 weeks gestation reveals bilateral thrombi in the lateral ventricles: what is the most likely cause of this finding?

Fragility of the GERMINAL MATRIX *called Intraventricular Hemorrahge (IVH)

Progressive gait instability, dysmetria, dysarthria for the past several years. Patient is 16 years old. PE shows bilateral motor weakness and loss of deep tendon reflexes, joint position, and vibration sense. MRI of brain and spine shows degeneration of the posterior columns and spinocerebellar tracts. What autosomal recessive degenerative condition?

Friedreich Ataxia *Hypertrophic Cardiomyopathy *Kyphoscoliosis *Pes Cavus (High Arches) *Loss of proprioception and Vibration senses

Inappropriate behavior, impaired judgment, poor problem-solving skills (executive dysfunction) possible gait and incontinence disturbances: where is the lesion?

Frontotemporal Dementia

Young man who is healthy, mother told him he has 'a problem metabolizing sugar' he has no dietary restrictions. Urine sample shows repeatedly positive copper reduction test but glucose oxidase dipstick testing is negative. What enzyme is deficient in this patient?

Fructokinase *Essential Fructosuria (benign condition) *Fructose is a reducing sugar and can be detected by COPPER REDUCTION TEST (+). *GLUCOSE OXIDASE is used to ascertain the presence of urinary glucose (-) not positive in presence of fructose or galactose.

Fructose 2,6-bisphosphate inhibits what?

Fructose 1, 6-biphosphatase (DECREASES Gluconeogenesis)

Maraviroc: MOA?

Fusion Inhibitor (inhibits viral entry) *binds CCR-5 which inhibits interaction with gp120

Enfuvirtide:MOA?

Fusion Inhibitor (inhibits viral entry) *binds bp41

Muscarinic receptors are what type of receptors?

G-protein-coupled receptor (act through 2nd messenger)

List the phases of the Cell Cycle in order?

G1 -> S -> G2 -> M G1: Organelle Duplication S: DNA duplication G2: DNA damage repair M: Cell Division

What phase of the cell cycle does Organelle Duplication occur?

G1 Phase

What phase of the cell cycle does DNA damage repair occur?

G2 Phase

Bleomycin & Doxorubicin affect what phase of the cell cycle?

G2 Phase *DNA damage repair phase

Huntington Disease: deficiency of what neurotransmitter causes these symptoms?

GABA

Filtration Fraction?

GFR/RPF

Hormone that makes you hungry?

GHRElin *make you hunGHRE

Validity (Accuracy): what MUST you have in order to evaluate?

GOLD STANDARD *in order to compare your new test you must compare it to the GOLD STANDARD TO COMPARE the VALIDITY (ACCURACY)

High blood levels of galactose in serum results in conversion to what?

Galactitol

Galactokinase Deficiency: what does this enzyme do?

Galactose --> Galactose-1-P

Galactose-1Phosphate Uridyltransferase: what does this enzyme do?

Galactose-1-P --> UDP-Galactose

Infant with lethargy, vomiting, and jaundice soon after birth. Placed on a galactose-free formula and shows gradual improvement. What steps in galactose metabolism is most likely impaired?

Galactose-1Phosphate Uridyltransferase *Classic Galactosemia

Bilateral lens opacities, otherwise asymptomatic, large urinary excretion of galctose: diagnosis?

Galactosemia

Porcelain gallbladder: patient is at high risk of what?

Gallbladder Adenocarcinoma

Symptoms of small bowel obstruction and gas within the gallbladder and biliary tree: diagnosis?

Gallstone Ileus *lodges in ILEUM

Why in Acute Cholycystitis is the gallbladder not visualized?

Gallstone obstructs Cystic Duct so the radiotracer is taken up by liver and is excreted into the common bile duct and proximal small bowel, but never makes it to the Gallbladder.

HIV positive with CD4 count less than 50. Eye shows Retinitis. What is the best initial therapy?

Ganciclovir *Cytomegalovirus (CMV)

Vibrio Cholerae is very sensitive to what?

Gastric Acidity *may grow well in patients with Achlorhydria or Antacid use

Persistent diarrhea, weight loss, abdominal pain. Postbulbar duodenal and jejunal ulcers. This patient most likely has a tumor secreting what molecule?

Gastrin *Zollinger-Ellison Syndrome (GASTRINOMAS) *may have DIARRHEA (as pancreatic/intestinal enzymes are inactivated by gastric acid and cannot digest nutrients properly.

Peptic ulcers involving the POSTERIOR DUODENAL BULB can erode into what and cause bleeding?

Gastroduodenal Artery

What does Dopamine cause at higher doses?

Generalized VASOCONSTRICTION by (alpha-1 adrenergic effect)

Chronic, worry, feeling on edge, muscle tension, and insomnia?

Generalized anxiety Disorder (GAD)

What has a greater impact on peak bone mass and rate of bone loss? (Environmental Factors, or Genetic Factors)

Genetic Factors *Environmental Factors (Potentially modifiable)

Reactive Arthritis: Preceding infection?

Genitourinary: Chlamydia Trachomatis Enteritis: Salmonella, Shigella, Yersinia, Campylobacter, Clostridium Difficile

Complete Penetrance?

Genotype is very likely to produce clinical signs and symptoms in individuals.

Incomplete Penetrance?

Genotype only sometimes produce the associated trait or condition.

Haploid genome, Peptidoglycan Cell Wall sandwich between an outer membrane and an inner plasma membrane?

Gram-Negative Bacteria

Patient receives treatment for bradycardia. However, she now complains of severe right-sided eye pain. What is the most likely cause of this eye pain? 2. What medication was given?

Glaucoma 2. Atropine *Atropine BLOCK VAGAL INFLUENCE on SA & AV nodes --> INCREASES HR

What is the most common primary cerebral neoplasm of adults that is typically located within the cerebral hemispheres and may cross the midline?

Glioblastoma "butterfly glioma" *highly malignant and grossly contain areas of necrosis and hemorrhage

The most common primary brain neoplasm in adults?

Glioblastoma Multiforme

Brain tumor with Pseduopalisaiding pleomorphic tumor cells?

Glioblastoma Multiforme *characterized by CENTRAL area of NECROSIS & HEMORRHAGE & new VASCULAR PROLIFERATION

Woman has "something stuck in her throat" sensation. Oropharynx is normal. Barium swallow is unrevealing. Upper GI endoscopy also normal. Diagnosis?

Globus Sensation (Functional Disorder) *Function Disorder means it is not due to a Structural abnormality or Motility Disorder *Globus Sensation (aka Globus Hystericus & Globus Pharyngeus) --> ABNORMAL sensation of a foreign body, tightness or fullness in throat and all TESTS ARE NORMAL!!

After 12-18 hours of fasting what becomes the principal source of blood glucose?

Gluconeogenesis

Aldose Reductase: Function

Glucose -> Sorbitol or Galactose -> Galactitol *both Sorbitol * Galactitol can cause damage to cells if it accumulates.

Patient has bilateral lesion in the mammillary bodies. During hospitalization, he develops acute confusion, horizontal nystagmus and bilateral abducens palsy. What most likely precipitated the worsening of his symptoms during hospitalization? 2. Why?

Glucose Infusion 2. *THIAMINE is a cofactor of the enzymes responsible for the glucose metabolism. INFUSION OF GLUCOSE will EXACERBATE the pre-existing THIAMINE DEFICIENCY.

glucose-6-phosphate dehydrogenase: function?

Glucose-6-Phosphate --> 6-Posphogluconate *part of the HMP Shunt

During the catabolism of proteins, amino groups are transferred to alpha-ketoglutarate to form what?

Glutamate *Glutamate is then processed in the Liver to form UREA (the primary disposal form of nitrogen in humans)

HbC: what is the residue substitution?

Glutamic acid --> Lysine (charged) *Lysine is charged which does not allow HYDROPHIC INTERACTIONS between Hb molecules like in HbS

HbS: what is the residue substitution?

Glutamic acid --> VALINE (Nonpolar, neutral charge)

Renal Metabolism of which of the following amino acids is most important for maximizing acid excretion? (alanine, arginine, aspartate, glutamine, histidine)

Glutamine *Acidosis stimulates renal ammoniagenesis, a process by which renal tubular epithelial cells METABOLIZE GLUTAMINE TO GLUTAMATE, GENERATING AMMONIUM that is exreted in the urine and bicarbonate that is absorbed into the blood. *This process is responsible for the VAST MAJORITY of RENAL ACID excretion in CHRONIC ACIDOTIC STATES *Alanine & Aspartate --> are glucogenic amino acids (Alanine is metabloized in the liver to produce pyruvate and Aspartate can be interconverted into oxaloacetate. *Histidine is converted to histamine

What is the MOST abundant amino acid in Collagen?

Glycine

A dermatologist is studying role of different amino acids in wound healing. She cultures mature dermal fibroblasts. After several days, the fibroblasts begin synthesizing polypeptide chains that assemble into triple helical structures, followed by fibrils. What amino acid is most likely found in highest quantity in these proteins?

Glycine *These proteins are COLLAGEN synthesized by fibroblasts *Collagen consists of (

Base Excision repair process?

Glycosylase (cleaves altered base from site) Endonuclease (cleaves 5' end) Lyase (cleaves 3' end) DNA polymearse (fills the gap) Ligase (seals the nick)

Progressively worsening impotence and loss of libido, blurring of peripheral vision, bitemporal visual field deficit, elevated serum prolactin. What are the changed in GnRH, LH, and testosterone?

GnRH: DECREASED LH: DECREASED Testosterone: DECREASED

Weight lifter's arms suddenly give out: what structure is responsible?

Golgi Tendon Organ

Leuprolide: what is it?

Gonadotropin-releasing hormone (GnRH) analog

Antibodies in a caucasian male are found to react with alpha3-chain of collagen type IV. What is the diagnosis?

Goodpasture Syndrome GBM is composed of collagen type IV

Patient with a history of polycythemia Vera has been noncompliant with phlebotomy treatment. In the past year she had 3 episodes of acute knee pain that had resolved with over the counter analgesics. What is causing the acute knee pain?

Gout *Myeloproliferative/Lymphoproliferative disorders are a cause of Gout

Mutation results in INCREASED phosphoribosyl Pyrophosphate Synthetase gene. What disease will this cause?

Gout *this will cause INCREASED production of PURINES due to fee-forward activation of the purine synthesis pathway *As a result more Purine molecule will undergo DEGRADATION resulting in increased risk of gout

Primary Biliary Cholangitis: histologically similar to what?

Graft-versus host disease

Takayasu Arteritis: what is observed in this patient's arteries?

Granulomatous inflammation of the media (leads to thickening and narrowing of the aortic arch and the proximal great vessels)

Lung that is pale, and firm suggests what phase of pneumonia?

Gray Hepatization

Microscopic Appearance: alveoli are filled with exudate containing neutrophils, fibrin and some fragmented RBCs?

Gray Hepatization

Antifungal that interacts with fungal cell microtubules, inhibiting mitosis?

Griseofulvin

Glucagon acts through what receptor when regulating glucose metabolism?

Gs -> Adenylate Cyclase -> cAMP

Weakness and tingling starting in feet and now at patients knees. Getting worse each day. Strength is 2/5 in both legs and absent DTF. PMH is insignificant except for mild respiratory infection 2 weeks ago.

Guillain-Barre Syndrome

Myosin filaments only?

H Band

A Gastric Biopsy is taken and place into a urea-containing solution containing a pH indicator. 30 minutes later, the solution has turned pink, indicating a pH increases. This test results suggests what?

H. Pylori Infection *Rapid Urease Test (PINK is POSITIVE)

The presence of which of the following serologic markers would increased the risk of vertical transmission of the virus? (HBcAg, HBsAg, HBeAg)

HBeAg

The presence of what predicts infectivity of Hepatitis B?

HBeAg *the higher the levels, the more likely the patient is to transmit the virus

Iron Deficiency Anemia: circulating Transferring?

HIGH

Tuberculoid Leprosy is characterized by what type of immunity?

HIGH-CELL mediated immunity with a large TH1-type immune response

Reactive Arthritis preferentially affects individuals positive for what?

HLA-B27

Immunocompromised patient with Esophagitis, endoscopy shows punched-out ulcers: organism?

HSV-1

Drug-induced Lupus Erythematosus: What drugs?

HYDRALAZINE PROCAINAMIDE ISONIAZID Minocycline Quinidine

Retinal Hemorrhage: what is the most likely cause?

HYPERTENSION

Cotton-wool spots: Cause?

HYPERTENSION *small, white foci of retinal ischemia

In Sweat gland impaired function of the CFTR transmembrane protein causes what changes in chloride, sodium, and water?

HYPERTONIC Sweat *INCREASED Chloride *INCREASED Salt

After 2 days of high altitude, arterial blood gas values are drawn. What is the initial initiator of the metabolic changes? 2. Then what follows?

HYPOXEMIA 2. Hyperventilation which lowers the PaCO2 and actually raises the pH because it causes a RESPIRATORY ALKALOSIS. *the partial pressure of oxygen in inspired air falls at high altitudes

Chancroid Disease: organism?

Haemophilus Ducreyi

Painful red papules on the genitals the eventually erode to become tender ulcers. Regional lymph nodes may swell and become chronic ulcers: Organism?

Haemophilus Ducreyi

3 month old with meningitis that is a gram-negative coccobacilli?

Haemophilus Influenza B

HIV drug(s) that may cause significantly elevated fasting blood glucose

Protease Inhibitors (NAVIR)

Most common type of Hypothyrodism in US?

Hashimoto Thyroiditis

What is the best way to ask about sexual preference?

Have your sexual partners been women, men, or both?

What is the predominant form of adult hemoglobin?

HbA (a2B2 or alph2Beta2)

Glutamic Acid to Lysine mutation in Beta-Globin?

HbC disease or HbC trait

Hemoglobin F (HbF) has Increased Oxygen affinity as a result from of what?

HbF has DECREASED affinity for 2,3-Bisphosphoglycerate (2,3-BPG)

What property of HbS explains why HbS is associated with more pronounced clinical manifestations than HbC disease?

HbS allows hydrophobic interaction among hemoglobin molecules *the usually negatively charged glutamic acid is replaced by a NONPOLAR residue which allows HYDROPHOBIC INTERACTIONS that cause AGGREGATION of HB molecules

Pale Infarcts: occur in what organs?

Heart Kidney Spleen

E. Coli Watery Gastroenteritis: Virulence Factor?

Heat-stable/heat labile enterotoxins

MHC Class I is made of?

Heavy Chain & Beta2-Microglobulin

Duodenal ulcers are most likely due to?

Helicobacter Pylori

Flinging of the limbs on one side of the body?

Hemiballism

Lesion of the subthalamic Nucleus causes?

Hemiballism (flinging movements of the contralateral extremities *Lacunar infarct (associated with hypertension) in or near the subthalamic nucleus is usually the cause

Hemosiderin can be identified on liver MRI or biopsy with Prussian blue stain: Diagnosis?

Hemochromatosis

Black Pigment Gallstones: caused by?

Hemolysis

Beta-thalassemia major patient has routine RBC transfusions and chelation therapies. Liver biopsy shows Kupffer cells containing coarse, YELLOWISH-BROWN cytoplasmic granules. This granules are composed of?

Hemosiderin *Hemosiderin: iron storage complex and microscopically apears as BROWN or YELLOWISH-BROWN pigments in either granular or crystalline form. *IRON OVERLOAD from HEMOLYSIS and frequent transfusions

Portal Triad? 2. What ligament

Hepatic Artery Portal Vein Common Bile Duct *Hepatoduodenal Ligament

Aflatoxins of greatly increased your risk of what?

Hepatocellular Carcinoma G:C --> T:A transversion of p53 gene

Glutamic Acid to Valine mutation in Beta-Globin?

Sickle Cell Anemia (Homozygous) Sickle Cell Trait (Heterozygote)

Nausea associated with vestibular nausea (motion sickness): medication targeting what receptors?

Histamine H1 Receptor Muscarinic Acetylocholine Receptors

Which histone protein is located OUTSIDE of the histone core?

Histone H1

What histones are located on the inside of the histone core?

Histone H2A, H2B, H3, H4

Ovoid cells within Macrophages: organism?

Histoplasma Capsulatum

Cave exploration to the central US, fever, cough, malaise, pulmonary infiltrates, and hilar adenopathy on chest x-ray? What organism?

Histoplasma Capsulatum **found as a mold in solid, present in bird and bat dropping and endemic in MISSISSIPPI and OHIO RIVER BASINS (patient may report history of exploring CAVES or CLEANING BIRD CAGES OR COOPS)

Dramatic, superficial, attention seeking?

Histrionic Personality Disorder

Meibomian Glands: what type of glands?

Holocrine

Sebaceous Glands: what type of glands?

Holocrine

Ventricular Septal Defect (VSD): murmur?

Holosystolic murmur over left sternal border

Alkaptonuria: what enzyme is deficient?

Homogentisate Oxidase

Alkaptonuria: what accumulates:

Homogentisic Acid

Alkaptonuria: what accumulates?

Homogentisic Acid

Severe arthritis in adult with blue-black depots on sclerae and ear: what is deficient?

Homogentisic Acid Dioxygenase Deficiency

If the middle cerebral artery (MCA) were involved in a stroke what visual abnormalities may be involved?

Homonymous Hemianopsia Any UNILATERAL visual pathway lesion beyond the optic chiasm can cause HOMONYMOUS HEMIANOPSIA.

Virus that causes Warts?

Human Papillomavirus (HPV)

HIV man with a single hard mass with superficial ulceration in the anal canal? What pathogen?

Human Papillomavirus (HPV) *type 16 and 18

What drugs cause SELECTIVE ARTERIOLAR VASODILATION?

Hydralazine Minoxidil

AIDS patient with neurodenerative disease. Describe what would be seen on CT scan?

Hydrocephalus Ex-Vacuo *AIDS dementia *caused by central NEURONAL VOLUME LOSS

What is the most important prognostic marker in bladder cancer?

Involvement of the Muscular layer (tumor penetration of the bladder wall)

Down Syndrome: Endocrine complication?S

Hypothyrodism Type 1 DM Obesity

Lesch-Nyhan Syndrome: what enzyme is likely DECREASED?

Hypoxanthine-Guanine Phosphoribosyltransferase (HGPRT)

Which 2 sections of the Sarcomere lengthen and shorten with relaxation of the Sarcomere?

I Band H Band *Not A band --> very confusing and guarunteed to try and trick you!!

Acute Pancreatitis: causes?

I GET SMASHED Idiopathic Gallstones Ethanol Trauma Steroids Mumps Autoimmune disease Scorpion Sting Hypercalcemia/Hypertriglyceridemia ERCP Drugs (Sulfa, NRTIs, Protease Inhibitors)

Actin filaments only?

I band

RNA Polymerase I, II, II hints?

I, II, and III are numbered in the same order that their products are used in protein I: rRNA II: mRNA III: tRNA

REVERSIBLY inhibit COX-1 & COX-2 enzymes?

IBUPROFEN NAPROXEN Diclofenac INDOMETHACIN Ketorolac Meloxicam Piroxicam NSAIDS

Hepatitis B (HBV): treatment?

IFN-alpha

Hepatitis C (HCV): treatment?

IFN-alpha Ribavirin, or Simeprevir or Sofosbuvir

Which cytokines has ANTI-INFLAMMATORY roles?

IL-10 TGF-Beta

NK Cells are activated by?

IL-12 IFN-gamma

Which of the following cytokines is produced exclusively by lymphocytes? (IL-1, IL-2, TNF-alpha, IFN-alpha)

IL-2

How do ACE-I and NSAIDS (not aspirin) cause Lithium Toxicity?

IMPAIRS LITHIUM CLEARNACE which leads to toxicity

A healthy 21 year old concerned about risk of developing cancer. She has a family history of early onset breast cancer and soft tissue sarcomas. The physician strongly suspects a family history of Li-Fraumeni Syndrome. If she has inherited this condition, germline DNA analysis would most likely demonstrate which of the following genetic abnormalities?

INACTIVATING mutation affecting 1 ALLELE *In a patient with an inherited inactivating mutation in a tumor suppressor gene, malignant transformation occurs when a single somatic cell subsequently acquires a "SECOND HIT" --> DEVELOPS A SECOND INACTIVATING MUTATION IN THE REMAINING COPY *this mechanism is behind Li-Fraumeni Syndrome and MOST HERITABLE CANCER SYNDROMES!

Abnormal phosphorylation of Retinoblastoma (Rb) protein results in?

INACTIVATION

PTH causes Osteoblasts to do what?

INCREASE production of RANK-Ligand & MONOCYTE COLONY-STIMULATING FACTOR (M-CSF)

Exudates: LDH and protein levels?

INCREASED

Paget's disease initial abnormality of Osteoclastic bone resorption is followed by?

INCREASED BONE FORMATION by OSTEOBLASTS *results in HIGH BONE TURNOVER

Dobutamine causes what effect on the heart?

INCREASED CONTRACTILITY

LOW Osteoprotegerin/receptor activator of nuclear factor kappa-B Ligand (RANK-L) RATIO?

INCREASED Osteoclast formation & survival

Chromatolysis (Axonal Reaction): these changes reflect what?

INCREASED PROTEIN SYNTHESIS --> in effort to repair damage

Elevated baseline Insulin: what best explains why baseline insulin would be constantly elevated?

INCREASED Peripheral Insulin Resistance *Type II DM

Corticosteroid: adverse effect on respiratory system?

INCREASED SURFACTANT production

A woman who is taking Estrogen and Progesterone supplements for severe hot flashes has a lab test performed. Shows normal TSH level. What will happen to the TOTAL T4 and TOTAL T3 levels?

INCREASED Total T4 and INCREASED Total T3

Meniere Disease: what is the cause?

INCREASED pressure of ENDOLYMPH of Inner Ear & INCREASED VOLUME of ENDOLYMPH of Inner Ear *Recurrent vertigo, earfullness/pain, unilateral hearing loss & tinnitus: diagnosis

What effect does Insulin have on protein?

INCREASED protein synthesis

Overexpression of HER2/neu oncogene is associated with what?

INCREASED rates *BREAST (present 30% of invasive breast cancers) *OVARIAN *ENDOMETRIAL *GASTRIC

Thyroid Storm (Severe Thyrotoxicosis) have an increased sensitivity to what?

INCREASED sensitivity to CATECHOLAMINES via Thyroid hormone-mediated upregulation of beta-adrenergic receptor expression.

Teriparatide: MOA?

INCREASES OSTEOBLASTIC Activity

Scrotal ultrasound reveals fluid surrounding the affected testicle: the specific embryologic defect giving rise to this patient's condition can also lead directly to which of the following?

INDIRECT Inguinal Hernia

Brown Pigment Gallstones: caused by?

INFECTION

Ovarian Torsion: involves what ligament?

INFUNDIBULOPELVIC Ligament (SUSPENSORY LIGAMENT of the ovary)

Biphosphonates: MOA?

INHIBITS OSTEOCLAST activity

Ribavirin: MOA

Interfering with the *DUPLICATION* of viral genetic material -inhibiting RNA polymerase -inhibiting inosine monophosphate dehydrogenase (depleting GTP) -causing defective 5' cap formation on viral mRNA transcripts -modulating a more effective immune response

DNA viruses replicate where?

In the NUCLEUS

Ether and other organic solvents causes a viral particle to lose their infectivity. How?

Inactivate the "enveloped" viruses *the solvent induces disruption or dissolution of the envelope lipid --> LOSS of VIRAL INFECTIVITY

Rabies vaccination: prophylaxis for people at risk like veterinarians, cave explorers, lab workers with infected tissues. What type of vaccine?

Inactivated Vaccine

Testicular torsion: what causes?

Inadequate FIXATION of LOWER POLE of the testis to the TUNICA VAGINALIS

Failure of fusion of the 2 medial nasal prominences: what does this cause?

Incomplete INTERMAXILLARY SEGMENT which is associated with SEVERE MIDLINE DEFECTS (such as HOLOPROSENCEPHALY

Down Syndrome newborn with a REDUCIBLE midline abdominal protrusion COVERED BY SKIN that is more pronounced when cries: What is the most likely cause of this finding?

Incomplete closure of the umbilical ring *Uncomplicated Umbilical Hernia *REDUCIBLE & COVERED BY SKIN!

Acyclovir: MOA?

Incorporation into newly replicating viral DNA --> terminates viral DNA synthesis *monophosphorylated by HSV/VZV THYMIDINE KINASE and not phosphorylated in uninfected cells

Patient with Myasthenia Gravis that is on treatment with low-dose Pyridostigmine but is experiencing symptoms. Infusion of Edrophonium ELICITS A SIGNIFICANT IMPROVEMENT in his symptoms: what is the best next step in management of this patient?

Increase Pyridostigmine Dosage *called MYASTHENIC CRISIS (when not enough Aceytlcholine is available at the neuromuscular junction.

Patient with Mild hemophilia A treated with desmopression (DDAVP). What is the mechanism of desmopressin for this patient?

Increase in ***ENDOTHELIAL*** protein release Factor VIII & vWF *not in liver protein synthesis

How do Class III antiarrhythmic affect the findings on ECG?

Increase the QT interval during. But does not affect QRS duration

What causes increased Estrogen in Klinefelter Syndrome?

Increased Aromatase activity

Gigantism: what lab results?

Increased GH Increased serum IGF-1

The Arrhythmias from Levodopa/Carbidopa treatment are caused by what?

Increased PERIPHERAL FORMATION of CATECHOLAMINES

Patient has increased urine output and decreased urine osmolality that does not respond appropriately to water deprivation (meaning her urine output is still increased). What is this patient's vasopressin level?

Increased Vasopressin levels *due to vasopressin resistance in the renal collecting ducts *NEPHROGENIC diabetes Insipidus

If RANK-L production is increased what is the observed effect on bone?

Increased bone resorption

Inherited defects in what result in disseminated mycobacterial disease in infancy or early childhood?

Interferon Signaling (INF-gamma) Defect *requires lifelong treatment with antimycobacterial agents

Lymph Nodes: Inferior portion of bladder?

Internal Iliac Nodes

Lichen Planus Histology

Increased thickness of Stratum Granulosum *HYPERGRANULOSIS

Normally increasing Central Venous Pressure predisposes an individual to PERIPHERAL EDEMA: however in the absence of peripheral edema what could best explain the compensatory mechanism to prevent buildup of fluid in the interstitial space?

Increased tissue lymphatic drainage *Moderate increases in capillary fluid transudation can be offset by a COMPENSATORY INCREASE IN TISSUE LYMPHATIC DRAINAGE THAT OCCURS DUE TO INCREASED INTERSTITIAL FLUID PRESSURE.

Desmopressin (DDAVP) effects on MILD Hemophilia A and von Willebrand Disease?

Increases FACTOR VIII & INCREASES endothelial secretion of VWF to stop bleeding

Patient has Sickle Cell Anemia and is given Hydroxyurea. Desribe how this works?

Increases Hemoglobin F Synthesis

Tetralogy of Fallot: What is the mechanism during squatting that relieves their symptoms?

Increasing SYSTEMIC VASCULAR RESISTANCE *squatting INCREASES SVR & DECREASES RIGHT-TO-LEFT SHUNTING, thereby INCREASING PULMONARY BLOOD FLOW and improving oxygenation status.

How is preload related to the contraction velocity of skeletal muscle?

Increasing preload increases contractile velocity up to a maximum value. *beyond this maximum, the ACTIN/MYOSIN OVERLAP becomes inefficient and contractile velocity will decrease

Carbamazepine: how does it affect P450?

Inducer *increased metabolism (decreases the drugs effectiveness)

Ataxia-Telangietasia: why is there an increased risk of cancer in these patients?

Inefficient DNA repair

Difficulty speaking like saying yes or no. Obeys spoken and written command, but cannot repeat words spoken to him. Where is the aphasia located?

Inferior Fronta Gyrus *Broca Aphasia

Splenic Flexure to Upper Rectum: what lymph nodes?

Inferior Mesenteric Nodes

Patient has been stabbed in RUQ of abdomen. BP is 70/42. Pringle maneuver is done and patient continues to hemorrhage. What are the 2 likely sources of this patient's bleeding?

Inferior Vena Cava Hepatic Vein

Pulsatile administration of GnRH stimulates release of FSH and LH and is useful for treatment of what?

Infertility

Vestibular Neuritis: What is the cause?

Inflammation of Vestibular Nerve (Viral or postviral) *SINGLE episode of vertigo that CAN LASTS, severe vertigo but no hear loss

Patient with fever, nonproductive cough, myalgias, clear lungs, and sick contacts at work likely has?

Influenza

What never runs along the orbital floor?

Infraorbital Nerve (continuation of Maxillary Nerve)

Persistent Asthma: the cornerstone of chronic therapy?

Inhaled Corticosteroids -FLUTICASONE -BUDESONIDE

High concentration of Fructose 2,6-bisphosphate inhibit what general processes and stimulate what general processes?

Inhibit --> Gluconeogenesis Stimulate --> Glycolysis

Cocaine: how does it physiologically exert its effect?

Inhibiting the presynaptic reuptake of Norepinephrine, dopamine, and serotonin.

Erythromycin: has what effect on P-450?

Inhibitor *Macrolides (except Azithromycin)

Monoamine Oxidase Inhibitors (MAO-I): MOA?

Inhibits Monoamine degradation which INCREASES levels of amine neurotransmitters (Norepinephrine, Serotonin, Dopamine)

Individuals with Diabetes Mellitus should do what daily?

Inspect their FEET!

Pt has Increased Anion Gap Metabolic Acidosis and treatment is started. Within several hours the patient mental status improves. Repeat lab studies show an INCREASE in serum Bicarbonate, DECREASE in serum OSMOLALITY, and a DROP in serum potassium. What was the treatment?

Insulin & Normal Saline *Diabetic Ketoacidosis *Hyperglycemia causes OSMOTIC DIURESIS which causes Urinary loss of SODIUM, POTASSIUM and FREE WATER (HYPEROSMOLALITY & HYPONATREMA) *in ACIDOSIS, POTASSIUM SHIFTS OUT of the CELL and ACID MOVE INTO CELL *Insulin causes INTRACELLULAR SHIFT OF POTASSIUM (decrease in serum potassium level also necessitates potassium repletion) *Saline decreases serum osmolality *BICARBONATE INCREASES because less KETONES ARE BEING MADE

TEGRA: MOA?

Integrase Inhibitor

Man undergoes colonoscopy which reveals adenocarcinoma. Patient appears visibly upset on hearing the diagnosis but quickly regains his composure. At dinner that night he describes to family members the different stages of colon cancer and the various treatments available. The patient reassures them that he has learned about the disease and knows what to expect. What defense mechanism?

Intellectualization *use of excessive thinking to avoid painful emotions or fears.

What is the hallmark of Atopic Dermatitis?

Intense Pruritus -diagnosis can be made without it

Where are dietary lipids most likely to be absorbed?

Jejunum

Patient loses a gallbladder where will the dietary lipids be absorbed?

Jejunum *same as a patient w/ gallbladder

Plicae Cirulares: where?

Jejunum Ileum

Biopsy of a skin lesion reveals nests of uniform round cells with inconspicuous nucleoli and few mitotic figures at the dermoepidermal junction: Diagnosis?

Junctional Nevus

Amiloride: what kind of drug?

K+ sparing diuretics

E Coli that causes Meningitis in infants what is the major virulence factor?

K1 Capsular Antigen (POLYSACCHARIDE) *prevents phagocytosis & complement-mediated lysis) ***FACTOR to keep in mind is that CAPSULES probably play a role in most MENINGEAL PATHOGENS

Patella injury will cause impairment in what movements

Knee extension

In the liver, Hemosiderin is typically seen in?

Kupffer cells (Hepatic Macrophages) -participate in RBC breakdown

Hip flexion and sensation loss in upper anteromedial thigh: what nerve root is compressed?

L2

Hip flexion, hip adduction, knee extension, sensation of lower anteromedial thigh is gone: what nerve root?

L3

Hip adduction, knee extension patellar reflex are lost. Lower anterolateral thigh, knee, medial calf and foot sensation are lost: what nerve root?

L4

Foot dorsiflexion & inversion, foot eversion, toe extension are lost. Sensation over buttocks, posterolateral thigh, anterolateral leg, and dorsal foot are lost: what nerve root?

L5

A patient with typical symptoms of acute rhinosinusitis: what sinus is most commonly affected?

Maxillary Sinuses

Eggshell calcification of hilar lymph nodes: diagnosis?

Silicosis

Structures arising from Neural Crest Cells?

MOTEL PASS Melanocytes Odontoblasts Tracheal Cartilage Enterochromaffin Cells Laryngeal Cartilage Parafollicular Cells (of the thyroid) Adrenal Medulla and all ganglia Schwann Cells Spiral Membrane

Most sensitive method in diagnosing Vertebral Osteomyelitis?

MRI

High Anion Gap Metabolic Acidosis list?

MUDPILES Methanol (formic acid) Uremia Diabetic Ketoacidosis Propylene Glycol Isoniazid/Iron Lactic Acidosis Ethylene Glycol (--> oxalic acid) Salicylates (aspirin)

IL-1 produced by?

Macrophages

Resolution stage of Pneumonia?

Macroscopic appearance: restoration of normal architecture Microscopic appearance: enzymatic digestion of the exudate

Buproprion: use?

Major Depressive Disorder & Smoking Cessation

Patient is hospitalized because of suicidal ideation. For the past month he has been depressed, withdrawn, and physically restless. Patient has been unable to work as a computer programmer because he refused to leave the house. He has lost 10 lbs and has difficulty falling asleep. During this time he has stated that he is responsible for evil in the world and has heard a voice telling him that he does no deserve to live. Diagnosis?

Major Depressive Disorder with Psychotic Features *diagnosis is differentiated from psychotic disorders in that the PSYCHOTIC SYMPTOMS are ONLY DURING THE EPISODES OF MAJOR DEPRESSION

Man has enlarged lymph node in anterior cervical chain. Biopsy reveals abnormal lymph node architecture and numerous lymphocytes and MONOCLONAL lymphocytic proliferation: this is strong evidence for malignant or benign lymph node?

Malignant

What is responsible for inhibiting Beta Oxidation?

Malonyl-CoA *Acetyl-CoA Carboxylase (Acetyl-CoA ---> Malonyl-CoA) --> this occurs in Cytosol

Improper Rotation of the the developing gut and the mesentery In Utero: diagnosis?

Malrotation & Midgut Volvulus

Tensor Tympani Muscle: innervated by?

Mandibular Branch of Trigeminal Nerve (CN V3)

Cystic Medial Degeneration is seen in what? 2. what does this potentially cause?

Marfan Syndrome 2. Aortic Dissection

Mast cells positive for KIT (CD 117)?

Mast cell proliferation *KIT receptor tyrosine kinase associated with MAST CELL PROLIFERATION

Women has trouble abducting her right arm past the horizontal position and she demonstrates winging of the scapula. If she had a recent procedure the patient most likely had what procedure?

Mastectomy with axillary lymph node removal (injury to LONG THORACIC) *deltoid and supraspinatus muscles abduct the arm up to the horizontal position after which the ***SERRATUS ANTERIOR** and trapezius are required to rotate the glenoid cavity superiorly, allowing complete abudction of the arm over the head

What type of Leukemia: CD19 and CD5

Mature B-Cell Leukemia (or chronic lymphocytic leukemia/small lymphocytic lymphoma, CLL/SLL) *CD5 (T-cell marker) but for some reason it is coexpressed.

Dopamine Receptors antagonist: medication name?

Metocloproamide Promethazine

Trichomoniasis (Trichomonas Vaginalis): treatment?

Metronidazole -treat sexual partner too

Bacterial Vaginosis (Gardnerella Vaginalis): treatment?

Metronidazole or Clindamycin

Trichomonas Vaginalis: treatment

Metronidazole (treat sexual partner too)

Clostridium Difficile treatment?

Metronidazole or Oral Vancomycin

An infant with hydrops fetalis is born to couple who emigrated from Thailand. Placenta is edematous. Infant is pale and edematous. Liver is massively enlarged. Both parents have strong family histories of hemoglobinopathy. Maternal Blood smear is most likely to demonstrate?

Microcytosis *Infant had Hb BARTS (severe form of ALPHA THALASSEMIA -> four nonfunctional alpha globin) *DON'T FALL FOR Sickle Cells

Salmonella and Shigella: what cells are primarily responsible for uptake of the bacteria?

Microfold (M) Cells

3 day old child on laparotomy fibrous bands are seen extending from the cecum and right colon to the retroperitoneum, causing compression of the duodenum. What embryologic processes has failed?

Midgut rotation around the superior mesenteric artery *the cecum rests in the RUQ *LADD's (fibrous) bands connect the retroperitoneum in the RLQ to the right colon/cecum by passing over the 2nd part of the duodenum, causing intestinal obstruction in the process.

Most common cause of Nephrotic syndrome in young children?

Minimal Change Disease

Lynch Syndrome (hereditary nonpolyposis colon cancer): what is the genetic defect?

Mismatch Repair

Medications used for pregnancy termination?

Misoprostol Mifepristone Methotrexate

Acute Rheumatic Fever: late valve lesion?

Mitral valve Stenosis

Poxvirus causes?

Molluscum Contagiosum

Severe hypertension, headache, blurry vision, tachycardia, tremors. Patient ate a lot of cheese and drank an iced tea. Patient has a medical history of severe atypical depression. What medication caused these symptoms?

Monoamine Oxidase Inhibitors (MAO-I)

Right Internal Capsule Stroke: manifestations?

Motor weakness of left arm and leg. Slurred speech, drooping of left lower face. On passive extension of left arm there is initial resistance followed by a sudden release of tension as extension is continue (clasp-knife spasticity) *these are all UMN lesion symptoms

Unconscious. History of substance of abuse. Recent hospitalization for depression. Cyanotic and unresponsive to painful stimuli. T 98. BP 100/60. Pulse 60. Respirations are 4/min. Pupils are constricted and poorly responsive to light. IV medications started and patient regains consciousness 2 minutes later. IV drug used to treat this patient has the greatest affinity for what receptor?

Mu (Naloxone is pure Opioid receptor ANTAGONIST) *Opioid Overdose *reversal with Naloxone can occur within MINUTES

Patient requests medication to reduce her craving for alcohol. A medication with what mechanism of action would be most effective in this patient?

Mu opioid receptor blockade *Naltrexone *REDUCES CRAVING (only one) *BLOCKS rewarding and reinforcing effects of alcohol

What is the mechanism used to clear the respiratory tract of deposited particles 2.5 - 10 micrometers in size?

Mucociliary Transport in the Trachea and Bronchi *MEDIUM dust particles

Ulcerative Colitis involves what part of the intestinal wall?

Mucosa & Submucosa

64 yo male, crampy abdominal pain while passing stool, blood in stool. Decreased femoral and dorsalis pedis pulses. What will be seen on Colonoscopy?

Mucosal Hemorrhage, ecchymoses and patchy areas of necrosis *Ischemic Colitis -weak pulses indicate EXTENSIVE ATHEROSCLEROSIS

Double Y Males (XYY): fertility?

Normal

Organophosphate Poisoning: what is a risk for a patient if they are only treated with Atropine?

Muscle Paralysis *Atropine reverses muscarinic efffects but does not prevent the development of nicotinic effects such as MUSCLE PARALYSIS. Atropine -> reverses the MUSCARINIC symptoms Pralidoxime -> reverses MUSCARINIC & NICOTINIC effects

Charcot-Marie-Tooth Disease [Hereditary Motor and Sensory Neuropathy (HMSN)]: defect?

Mutation of a MYELIN PROTEIN GENE involved in the structure and function of PERIPHERAL NERVES or the MYELIN SHEATH.

Beta-Thalassemia: is caused by?

Mutations that result in DEFECTIVE TRANSCRIPTION, PROCESSING & TRANSLATION of beta-globin mRNA. *leads to deficiency of the beta-globin chains required for normal hemoglobin synthesis

Shingles (Varicella Zoster Virus - VZV): what pathologic finding is most likely to be found in the affected area?

Mutlinucleated giant cells with intranuclear inclusions in Keratinocytes (Positive Tzanck smear)

Isoniazid (INH) must be processed by what in Mycobacterium Tuberculosis for the drug to be activated?

Mycobacterial Catalase Peroxidase

Sudden, brief, uncontrolled muscle contraction

Myoclonus *BRIEF *hiccups, hypnic jerks (when falling asleep), EPILEPSY, Creutzfeldt-Jakob disease

Damage to the Nucleus Ambiguous of the Medulla results in?

Myoclonus (a brief, involuntary twitching of a muscle or a group of muscles)

What bacteria produce IgA Protease?

N Gonorrhoeae N Meningitis S. Pneumoniae H. Influenzae

Neuroblastoma: associated with over expression of what gene?

N-MYC

Phenycyclidine: MOA?

N-methyl-D-Aspartate Receptor Antagonist

Six healthy children and 2 teachers develop acute vomiting and diarrhea within a 2-day period.

Norovirus *Rotavirus is less common in adults who already have antibody protection against it.

Acute bacterial arthritis in sexually active young adults is commonly caused by?

Neisseria Gonorrhoeae *intracellular organisms seen within Neutrophils

1 month old FULL TERM baby, lethargy, spontaneous vaginal delivery, home birth, no vaccines or medications at the delivery, head circumference is 99th percentile, weight and height 25th. Bulging anterior fontanelle. Eyes are driven downward, not able to track upward. Diagnosis?

Neonatal Vitamin K Deficiency *Intracranial Hemorrhage (ICH)

Neonate with severe respiratory distress has treatment. Later Ophthalmoscopy shows abnormal retinal vascularization that extends into the vitreous. What may have caused these eye findings?

Neonatal oxygen supplementation *potential adverse effect of oxygen therapy is retinal damage.

Renal Vein Thrombosis: is a well known complication of what?

Nephrotic Syndrome *Hypercoaguable State (Other Thromboembolic complications may happen too, this is just an example!)

Neuroblastoma: embryological derivative?

Neural Crest

Pheochromocytoma: cells responsible for this disease have what embryological origin?

Neural Crest *Adrenal Medulla is the origin of these tumors

Melanoma (malignancy of melanocytes): are embryologically derived from:

Neural Crest Cells

Use of anti-epileptic therapy (Valproate, Carbamazepine, Phenytoin) during PREGNANCY is a substantial risk factor for?

Neural Tube defects

Valproate during pregnancy causes what?

Neural tube defects

Phenytoin during pregnancy causes what?

Neural tube defects Orofacial cleft Microcephaly Nail or digit hypoplasia

Oseltamivir: MOA?

Neuraminidase Inhibitor (prevents VIRION PARTICLE RELEASE

Toddler, spontaneous bursts of non-rhythmic conjugate eye movements in various directions, hypotonia, and myoclonus, abdominal mass: diagnosis?

Neuroblastoma *may have HYPERTENSION

Alzheimer Disease involves accumulation of what intracellular?

Neurofibrillary Tangles (composed of TAU proteins that is HYPERPHOSPHORYLATED

A patient starts high-dose prednisone. On CBC differential What cells will increase as a result of this patient's therapy? (Basophils, Eosinophils, lymphocytes, monocytes, Neutrophils)

Neutrophils

Acute respiratory distress syndrome (ARDS): what cells release inflammatory mediators that cause damage in this disease?

Neutrophils

Gout: Synovial fluid analysis wil typically show monosidum urate crystals (needle-shaped, and negatively birefringement) and what cell?

Neutrophils

HIV drug that does not require activation via intracellular phosphorylation and inhibits the synthesis of viral DNA from the RNA template (Nonnucleoside reverse transcriptase inhibitors): name the 2 drugs?

Nevirapine Efavirenz Delavirdine *Nonnucleoside Reverse Transcriptase Inhibitors (NNRTIs)

What hyperlipidemia drug has an increased risk of acute gouty arthritis?

Niacin

Patient has severely elevated triglyceride level. Physician wards that the patient is likely to experience skin FLUSHING & WARMTH after taking the pills. What are these?

Niacin *Red, flushed face (only lipid-lowering agent that causes this is Niacin) *however DON'T get confused FIBRATES are best agent to LOWER TRIGLYCERIDES

Carcinoid Syndrome: associated with what vitamin Def?

Niacin Deficiency (Pellagra) (Vitamin B3)

Dopaminergic pathway functions to coordinate voluntary movements

Nigrostriatal Pathway

Parkinsonism: is related to a dysfunction in what dopaminergic pathway?

Nigrostriatal Pathway *functions to coordinate voluntary movements

What are Osteolytic cysts in the long bones associated with primary hyperparathyrodism called?

Osteitis fibrosa Cystica

Spongiosa filling the medullary canal with no mature trabeculae: characteristic of?

OsteoPETROSIS

Osteoclasts or Osteoblasts have PTH receptors?

Osteoblasts

Nadolol: what type of beta blocker?

Non-cardioselective Beta Blocker

Propranolol: what type of beta blocker?

Non-cardioselective Beta Blocker

Delavirdine: MOA?

Nonnucleoside reverse transcriptase inhibitors *DOES NOT require phosphorylation

Efavirenz: MOA?

Nonnucleoside reverse transcriptase inhibitors *DOES NOT require phosphorylation

Nevirapine: MOA?

Nonnucleoside reverse transcriptase inhibitors *DOES NOT require phosphorylation

Locus Ceruleus: release what neurons?

Norepinephrine L is close to N

The best method for determining whether a gene is being expressed?

Northern Blot *is to analyze for the presence of it mRNA

A mutation of a non-coding sequence may be affecting the gene coding for a specific fetal enzyme. What is the best method to determine if this gene is being transcribed in cultures of isolated cells?

Northern Blot (RNA) *the question did say TRANSCRIBED too *The best method for determining whether a gene is being expressed is to analyze for the presence of it mRNA.

Acyclovir: analog of what?

Nucleoside (Guanosine)

RBCs do no have what organelles?

Nucleus Golgi Apparatus Endoplasmic Reticulum

GABA neurotransmitter is made where?

Nucleus Accumbens

Organism reveals resistance to Drug X through a mechanism that decreases the ergosterol incorporation into the cell membrane. Drug X is most likely what?

Nystatin or Amphotericin B *this both bind Ergosterol in the cell membrane -> creating pores and causing cell lysis

What is the best way for the physician to initiate the medical interview?

OPEN ENDED QUESTIONS "What can you tell me about your pain"

Aminoglycosides abx if taken during pregnancy may cause what?

OTOTOXICITY Vestibulotoxicity -Gentamicin, Neomycin, Amikacin, Tobramycin, Streptomycin.

OVRPS

OVRPS (overpass) Optochin - Viridans Resistant - Pneumoniae Sensitive *STREPTOCOCCI

Meckel Diverticulum or Vitelline Fistula is caused by what failure of what embryologic process?

Obliteration of the Omphalomesenteric Duct *this duct connects the midgut lument with the yolk sac. Failed obliteration of the duct causes vitelline fistula (complete failure) or Meckel (partial failure)

What bias occurs when the investigators decision is affected by prior knowledge of the exposure status?

Observer Bias

Child has bacterial meningitis and has had inadequate treatment in the hospital. Mother wants to take him home and treat him with homeopathic remedies. What do you do?

Obtain a cour injunction to mandate continue treatments *discontinuing IV antibiotics after only 2 days places child and siblings at significant risk of serious harm and death. 1st talk to parents about concerns 2nd If efforts are unsuccessful, a court order or involvement of CPS may be necessary.

The only major nerve that exits the pelvis through the Obturator Canal?

Obturator Nerve

Cutaneous flushing, hypotension, secretory diarrhea, bronchospasm, cardiac valvular lesions. These symptoms will respond to what treatment?

Octreotide (Somatostatin analong) *Carcinoid Syndrome (5-HT)

no stress

On the office STAPH trip there was NO StRESs Novobiocin - Saprophyticus Resistant -Epidermidis Sensitive *STAPHYLOCOCCUS

5-HT3 Receptors antagonist: medication name?

Ondansetron

What other drug may precipitate a Pyridoxine (Vitamin B6) Def?

Oral Contraceptives

Beta-oxidation: Cytoplasm or Organelle?

Organelle (Mitochondria)

Citric Acid Cycle: Cytoplasm or Organelle?

Organelle (Mitochondria)

Ketogenesis: Cytoplasm or Organelle?

Organelle (Mitochondria)

Ornithine Transcarbamoylase & Carbamoyl Phosphate Synthetase from Urea Cycle: Cytoplasm or Organelle?

Organelle (Mitochondria)

Pyruvate Carboxylase: Cytoplasm or Organelle?

Organelle (Mitochondria)

Infant with Orotic Acid in blood and urine, Hyperammonemia, No megaloblastic anemia. What is deficient?

Ornithine Transcarbamylase Deficiency

Mifepristone: MOA?

PROGESTRONE ANTAGONIST Competitive inhibitors of PROGESTINS at PROGESTERONE RECEOTORS

Jugular Venous Tracing 2. what is abnormal on this Jugular Venous Tracing?

PROMINENT y descent

Muddy Brown Casts: what portion renal structures would most likely demonstrate signs of ischemic injury?

PROXIMAL Tubules (PCT) THICK ASCENDING LIMB

Vitamin D supplementation will cause what changes in PTH, Calcium, Phosphate?

PTH: Decreased Calcium: Increase Phosphate *Increases INTESTINAL absorption of CALCIUM & PHOSPHATE *Increases RENAL reabsorption of CALCIUM & PHOSPHATE

What is the cause of hypercalcemia in a PRIMARY BONE TUMOR?

PTHrP (activates PTH receptor, and excessive BONE RESORPTION)

Graves: treatment during 1st trimester of pregnancy

PTU *Methimazole (1st trimester teratogen)

Graves: what is given to a patient with life-threatening thyroid storm?

PTU *it ALSO decreases the PERIPHERAL CONVERSION OF T4 to T3

Alendronate is a analog of what?

PYROPHOSPHATE

In the absence of OXYGEN what happens to PYRUVATE?

PYRUVATE is converted to LACTATE (Lactate Dehydrogenase) *excessive lactate production results in lactic acidosis

In the absence of PYRUVATE DEHYDROGENASE what happens to PYRUVATE?

PYRUVATE is converted to LACTATE (Lactate Dehydrogenase) *excessive lactate production results in lactic acidosis

Lamellar bone structure resembling a mosaic: characteristic of?

Paget's Disease of Bone

Patient has pain and deformity of long bones with hearing loss: diagnosis?

Paget's disease of bone

Patient has dull periumbilical pain, but now the pain is now more severe and localized to the right lower abdominal quadrant: Explain the shift of pain?

Pain shifted from Visceral ---> SOMATIC Pain (INFLAMMATION of the Parietal Peritoneum) *as the appendix becomes more inflamed in this case it irritates the parietal peritoneum and abdominal wall and causes a MORE SEVERE SOMATIC PAIN that shifts from the umbilical region to MCBURNEY POINT

Many G protein-coupled receptors have carboxyl tails attached to cysteine residues on the Plasma Membrane: what is this called?

Palmitoylation

Alpha-1 Antitrypsin can cause what type of emphysema?

Panacinar Emphysema

Trypsinogen is secreted from what organ?

Pancreas

65 yo woman with weight loss, abdominal discomfort, jaundice, and an epigastric mass: diagnosis?

Pancreatic Cancer

Malabsorption: Normal D-Xylose absorption: what is the cause?

Pancreatic Insufficiency *Intestinal tract is not affect and D-Xylose is a monosaccharide so it can be absorbed normally.

Fine-needle biopsy of thyroid: Branching structures with interspersed calcified bodies?

Papillary Thyroid Cancer

Fine-needle biopsy of thyroid: Follicular hyperplasia with tall cells forming intrafollicular projections?

Papillary Thyroid Cancer

What part of the lymph node houses T Cells?

Paracortex

What part of the lymph node is not well developed in DiGeorge syndrome?

Paracortex *T Cells housed here

IV Propanolol is given to a patient with racing heart, shortness of breath, insomnia, increased anxiety for a week. Serum TSH is decreased. Free T4 is increased. How does Propranolol benefit this patient, in addition to beta-receptor blocking activity?

Peripheral conversion of T4 to T3 *Thyroid Storm (Severe Thyrotoxicosis)

Paralysis of upward gaze, ptosis, and pupillary abnormalities (blurry vision): what is this called?

Parinaud Syndrome

Impairment of the Ubiquitin-Proteasome system can contribute to the development of neurodegenerative disorders including?

Parkinsons Alzheimers

Woman has symmetric swelling and pain of her pIP, wrist, and knee joint. 5 yo had facial rash and fever last week. Conservative management with NSAIDS is recommended. Patient returns 4 weeks later. She says she discontinued the NSAIDS after 2 weeks because of GI upset. However the pain and swelling have decreased significantly. What is the most likely diagnosis.

Parvovirus Infection *causes SELF-RESOLVING ARTHRITIS in ADULTS *children --> ERYTHEMA INFECTIOSUM (5th disease)

Corynebacterium Diphtheriae: What intervention is most important to improve patient's prognosis?

Passive Immunization (Diphtheria Antitoxin)

Osteomyelitis: Cat & Dog bites?

Pasteurella Multocida

Infant with small head and eyes, cleft lip and palate. Small, round punched-out lesions with an overlying thin membrane on scalp. Small membranous sac with a loop of bowel protrudes from abdominal midline. What Syndrome?

Patau Syndrome (Trisomy 13) *Midline Defects -Microcephaly/Holoprosencephaly -Microphtlamia -Cutis Aplasia (punched out lesion on scalp) -POLYDACYLY -Cardiac defects -Renal defects -UMBILICAL HERNIA/OMPHALOCELE -Rocker-bottom feet

What is more common a Patent Foramen Ovale or an Atrial Septal Defect?

Patent Foramen Ovale *this is important when choosing the cause of a paradoxical stroke, you may be presented with both answer and go with PFO!!!

Incomplete fusion of atrial septum primum and secundum: what is this defect?

Patent Foramen Ovale (PFO)

Myxomatous Degeneration means what?

Pathologic weakening of connective tissue

Down Syndrome: explain unbalanced translocation?

Patient has 46 chromosomes but an extra arm of chromosome 21 is attached to another chromosome.

What drug works by binding to and inhibiting transpeptidase?

Penicillin *this enzyme catalyzes the final CROSS-LINKING step in peptidoglycan cell wall formation: the joining of the amino acid in the third position of a peptidoglycan molecule to the terminal D-alanine-D-alanine likely facilitates their binding to transpeptidase. **WHEN TRANSPEPTIDASE IS INHIBITED, CELL WALL SYNTHESIS CEASES AND CELL WALL DEGRADATION BY BACTERIAL AUTOLYSINS PROCEEDS UNCHECKED.

ESBL (Extended-Spectrum Beta-Lactamase) enzymes can inactivated what antibiotics?

Penicillins Cephalosporins (including 3rd and 4th generation) Monobactams (Aztreonam)

Corticosteroid: adverse effect on the GI Tract?

Peptic ulcer, GI bleed

The equilibration of O2 and Co2 is normally what (Perfusion Limited or Diffusion-Limited?

Perfusion Limited

In situations in which a parent's presence may interfere with obtaining honest answers from an adolescent patient, what should you do?

Politely ask the parent to wait outside and interview the patient privately *also important when discussing drugs, alcohol, tobacco, and sexual activity with TEENAGERS

Erythrocytosis is defined as what hematocrit level?

Polycythemia (Erythrocytosis) Hematocrit level *Men > 52% *Women > 48%

Eryhtrocytosis: what are the steps to diagnose?

Polycythemia (Erythrocytosis) Hematocrit level Men > 52% & Women > 48% 1. Absolute verse Relative (RBC mass is necessary -INCREASE RBC -> ABSOLUTE (GO #2) -NORMAL RBC -> RELATIVE (DONE here) 2. Primary vs Secondary erythrocytosis -PRIMARY -> Polycythemia (EPO is DECREASED) -Secondary -> INCREASED EPO 3. Hypoxic versus other causes -Hypoxic SaO2 < 925 or PaO2 < 65 mm Hg

Diseases that result from defects in numerous, often unrelated genes?

Polygenic Disease *Type 2 DM (which clearly has a heritable component but cannot be traced to any specific gene)

Encapsulated bacteria vaccines contain what?

Polysaccharide Capsule Antigens that are conjugated to a carrier protein

Tracheal pO2 150 mm Hg Alveolar pO2 145 mm Hg Alveolar pCO2 5 mm HG What is causing these abnormal values?

Poor Alveolar Perfusion *NOT Diffusion-limited because the diffusion capacity of CO2 across the respiratory membrane is approximately 20 greater than O2. So Alveolar CO2 concentrations are not nearly affected to the extend that O2 concentrations are in DIFFUSION-limited states.

In posterior and anterior knee dislocations: what anatomical structure is high risk of injury?

Popliteal artery *Tibial Nerve is also subject to injury (However it is more SUPERFICIAL)

A firm hard mass palpated in the RUQ, patient has hypercholesterolemia, and history of gallstones?

Porcelain Gallbladder *due to Chronic Cholecystitis

Acute Intermittent Porphyria: deficient enzyme?

Porphobilinogen Deaminase

Blistering cutaneous sun-sensitivity and tea-colored urine?

Porphyria Cutanea Tarda

Portal Hypertension in Cirrhosis: what vessels would have increased pressure?

Portal Venous System -Splenic Vein (may trick you on CT IMAGE) -Portal Vein (make sure on CT its not the IVC)

Portal venous system is valveless: why is this important?

Portal hypertension can cause retrograde flow of portal blood away from the liver

Patient has been getting excessively col. What hypothalamus nuclei may be involved?

Posterior Hypothalamus *Anterior nucleus= cool off (cooling , pArasympathetic). A/C = anterior cooling

The most common pathologic cause of BILATERAL fetal HYDRONEPHROSIS in BOYS?

Posterior Urethral Valves

Raloxifene indication?

Postmenopausal Osteoporosis

Drug's with higher affinity for their receptors have a increased?

Potency

Mathematically power is given by?

Power = 1 - Beta *Power is related to BETA

What is the exception to the rule that DNA viruses replicate in the NUCLEUS?

Poxvirus

What conditions are caused by Uniparental Disomy?

Prader-Willi Syndrome Angelman Syndrome

What type of Leukemia: markers deoxynucleotidyltransferase (TdT), CD19, CD10.

Precursor B-cell leukemia **TdT & CD10** markers of PRE-B CELLS

What type of Leukemia: CD1a, CD3, CD4, CD8, deoxynucleotidyltransferase (TdT).

Precursor T-cell leukemia **TdT** markers of PRE-T CELLS

PCR involves heating the solution followed by cooling the solution. Immediately upon cooling what happens?

Primer binding to single-strand DNA

TRABECULATED parts of the Atria and Ventricles are formed from what embryologic structures?

Primitive Atrium Primitive Ventricle

Drug-Induced Lupus Erythematosus: implicated drugs?

Procainamide Hydralazine Isoniazid Minocycline TNF-alpha Inhibitors (eg, Etanercept) *top 3 were bolded in UWORLD

All hormone-containing contraceptives prevent pregnancy through the action of what?

Progestins (synthetic equivalent to PROGESTERONE)

Infectious demyelinating disorder from JC virus which manifests as progressive neurologic deficits in immunocompromised individuals (lymphoma, leukemia, or HIV infection)

Progressive Multifocal Leukoencephalopathy

Child's parents are recently divorced. Parents bring child in because he is unusually quiet and won't speak to his parents anymore. The patient avoids his parents because he feels that they are angry with him. Child acknowledge that they have not expressed anger toward him but he can sense it. What defense mechanism best explains this behavior?

Projection *Child is likely angry with his parents about the divorce but unable to acknowledge these feelings. As a result misattributes ANGRY FEELINGS to his PARENTS even though there is LITTLE EVIDENCE THAT THEY FEEL THIS WAY. *Projection commonly occurs in patients who lack insight into their own motivations and feelings.

Ebstein-Barr Virus (EBV) commonly infects B cells, stimulating them to do what?

Proliferate CONTINUOUSLY ("transformation" or "immortalization")

Endometrial sampling reveals nonbranching, nonbudding, uniform glands evenly distributed throughout a uniform stroma: when did this biopsy of the uterus occur?

Proliferative Phase (days 1-14)

Woman has a surgery and several days later has fulminant hepatitis symptoms. What additional finding would be found? (Decreased serum albumin level or Prolonged prothrombin time)

Prolonged Prothrombin Time *ACUTE LIVER INJURY presents with NORMAL ALUBUMIN levels due to the long half-life of albumin (~20 days)

Group of subjects are selected and their exposure status is determined (smoker/non-smoker). Then followed for a certain period of time and observed for development of the outcome (breast cancer). What type of study?

Prospective Cohort Study

The vascular endothelium in the blood vessels of the heart secretes what prostaglandin that INHIBITS PLATELET AGGREGATION, ADHESION TO THE VASCULAR ENDOTHELIUM, VASODILATES, INCREASES VASCULAR PERMEABILITY, AND STIMULATES LEUKOCYTE CHEMOTAXIS?

Prostacyclin

Damaged endothelial cells lose the ability to synthesize what?

Prostacyclin *therefore predisposes to THROMBI & HEMOSTASIS

Niacin may cause Red, flushed face. What is the primary agent mediating these side effects? 2. what can DECREASED the Red, flushed face?

Prostaglandin 2. NSAIDS or long term use.

Misoprostol: MOA?

Prostaglandin E1 Agonist

Radical prostatectomy. What nerve within the fascia around the gland is injured? 2. What would be a consequence of injury to this nerve? * THIS IS MOST IMPORTANT

Prostatic Plexus (lies within the fascia of the prostate) *ERECTILE DYSFUNCTION

NAVIR: MOA?

Protease Inhibitor *NAVIR TEASE a PROTEASE

A significant Pulmonary Embolism causes what metabolic effects? (ph, PaCO2, and PaO2)

RESPIRATORY ALKALOSIS pH: INCREASED PaCO2: DECREASED *the PE causes ventilation-perfusion mismatch because it blocks transfer of Oxygen and CO2. this causes HYPOXEMIA that INCREASES VENTILATION & HEART RATE via a chemoreceptor reflex. This hyperventilation lowers CO2 because the rest of the long is still able to compensate and doesn't have the ventilation-perfusion mismatch. HOWEVER IT IS UNABLE TO FULLY COMPENSATE FOR HYPOXEMIA.

Normally, METABOLIC ACIDOSIS is partially compensated for by RESPIRATORY ALKALOSIS. When the steady-state PaCO2 persists above the range given by Winter's formula [(1.5 * HCO3) + 8 + or - 2]. Then what is MOST LIKELY occurring in the patient?

RESPIRATORY FAILURE (superimposed RESPIRATORY ACIDOSIS)

Protein Dynein is important for what?

RETROGRADE transport of intracellular vesicles and organelles toward the minus ends of microtubules (minus ends typically lie at sites NEAR THE NUCLEUS!!)

Change in Executive function, PERSONALITY, & DISINHIBITED BEHAVIOR: What lobe of the brain is affected be specific!

RIGHT Frontal Lobe *Disinhibited Behavior --> Left-sided lesions

Physical exam reveals a large deep wound directed posteriorly adjacent to the LEFT sternal border in the FOURTH intercostal space. What would be injured?

RIGHT VENTRICLE *Right ventricle composes the majority of the anterior surface of the heart and is at risk in penetrating trauma at the LEFT STERNAL BORDER

An experiment is ran to locate the position of a gene on a chromosome. Each DNA fragment is hybridized with mature mRAN. Under appropriate conditions, DNA partially denatures, allowing the mRNA to displace the homologous DNA strand and form a DNA-mRNA hybrid. Electron microscopy of these DNA-mRNA hybrids reveals several DNA loops. What is the best explanation for the formation of these loops?

RNA Splicing

Frontotemporal Dementia: Microscopic features?

ROUND PICK Bodies (inclusions of HYPERPHOSPHORYLATED TAU)

Hypovolemia: what are the changes in RPF, GFR, and FF compared to normal?

RPF: GREATEST DECREASE GFR: DECREASE Slightly FF: Increased

Tall, thin males around the age of 20, develops a pneumothorax: what condition may have led to this presentation?

RUPTURED SUPERFICIAL BLEB *called a Primary Spontaneous Pneumothorax

Restless, agitation, dysphagia progressing to coma within weeks of exploring a bat cave is concerning for?

Rabies Encephalitis *many patients with BAT RABIES are not aware that they had been biten

Triceps Reflex is absent: what nerve would this be?

Radial Nerve

Patient has a vertebral stress fracture and a mother died of breast cancer. What is the best drug to decrease the risk of BOTH bone fracture and breast cancer in a patient? Why?

Raloxifene *Estrogen AGONIST -> bone *Estrogen Antagonist -> Breast, Uterus PREVENTS both Breast Cancer and Osteoporosis and does not increase risk of Endometrial Cancer

Down Syndrome: most commonly caused by?

Random Meiotic Nondisjunction

Thiopental: what may cause its effect to be terminated early?

Rapid redistribution into tissue (SKELETAL MUSCLE and FAT)

Cold-induced vasospasm?

Raynaud Phenomenon

Individual transforms their unacceptable feelings or impulse into their extreme opposites.

Reaction Formation

Man had a laceration to his forearm 2 weeks ago and had sutures placed. Now he presents with a painful subcutaneous nodule that shows a granuloma formation on biopsy. What is most likely responsible for this condition?

Reaction to a foreign body (sutures)

Acute lower extremity unilateral arthritis and a sterile joint effusion following a GI infection?

Reactive Arthritis *question I had this was the only presenting symptom was the knee pain didn't have the other symptoms that may follow with Reactive Arthritis see 2 slides down

RBC show DECREASED hemolysis when exposed to increasing NaCl concentration: what disease (2)

Sickle Cell Anemia Thalassemia (Target Cell)

ALK oncogene: codes for?

Receptor Tyrosine Kinase

Inferior Thyroid Artery: What nerve travels in close proximity?

Recurrent Laryngeal Nerve

Lung that is dark, and firm suggests what phase of pneumonia?

Red Hepatization

Microscopic Appearance: alveoli are filled with exudate that contains RBCs, neutrophils and fibrin?

Red Hepatization

Pyruvate Kinase Deficiency: what is the likely cause of their splenomegaly (passive splenic congestion or red pulp hyperplasia?

Red Pulp Hyperplasia -passive splenic congestion is seen in portal hypertension, splenic vein thrombosis, or congestive heart failure

Achalasia is caused by?

Reduced numbers of inhibitory ganglion cells in the esophageal wall.

ACE-I started on a patient ant the tests reveal significant increase in serum creatinine level: explain the physiology?

Reduction in renal filtration fraction (DECREASE IN ANGIOTENSIN II --> DILATION OF EFFERENT ARTERIOLE) --> DROP IN GFR *normally Angiotensin II causes VASOCONSTRICTION at the EFFERENT ARTERIOLE however ACE-I INHIBIT Angiotensin II and block this mechanism

An interview technique that involves repeating what the patient has said to confirm understanding and convey that the patient's concerns are heard.

Reflection

Postprandial short-acting insulin: What are (Lispro, Aspart, Glulisine) better options for POST MEAL HYPERGLYCEMIA than Regular Insulin?

Regular: Peak (2-4) (Lispro, Aspart, Glulisine): Peak (45-75 min) *The REGULAR Insulin peaks after the POSTPRANDIAL PEAK in BLOOD GLUCOSE concentration

Hematocrit > 52% in a Man but RBC Mass is normal what does this mean?

Relative Erythrocytosis

Results of a new test are 12, 14 and 18. These results suggest that concerns should be raised concerning what about the new test?

Reliability *A RELIABLE test is REPRODUCIBLE

8 yo boy with growth retardation, MRI shows multiloculated, cystic, suprasellar lesion which is bulging into the floor of the 3rd ventricle and base of brain. Calcifications are present. From what structure is this mass derived?

Remnants of RATHKE pouch *Craniopharyngioma *usually 3 components 1. SOLID (actual tumor cells) 2. CYSTIC (filled with "MACHINERY OIL" like fluid) 3. CALCIFICATIONS

A woman with SLE has several type of autoantibodies directed against components of the cell nucleus. One specific antibody targets proteins complexed with SMALL NUCLEAR RIBONUCLEIC ACID. What is the function of these complexes?

Remove of introns from RNA transcripts (splicing of pre-mRNA) *snRNPs are an essential component of SPLICESOMES which remove INTRONS from PRE-mRNA to form MATURE mRNA *patients with SLE can have autoantibodies directed against snRNPs (anti-SMITH antibodies)

What are two fluid over-load states that could cause INCREASED Preload?

Renal Failure Congestive Heart Failure

Patient is treated for a bacterial infection with ampicillin. A week later he develops fever, skin rash, low urine output, T 99.5. Serum creatinine is 2.4, Urine shows 3-4 RBC, 5-10 WBC and 3-5 eosinophils. The pathologic process affecting this patient's kidney most likely involves what structure ?

Renal Interstitium *Acute Interstitial Nephritis *EOSINOPHILS *Penicillin-derivative *FEVER, RASH, ACUTE RENAL FAILURE (elevated creatinine, oliguria)

Sudden onset of abdominal or flank pain, hematuria, and right-sided varicocele suggests what?

Renal Vein Thrombosis

Angiotensinogen is converted to Angiotensin I by what?

Renin

What part of a membrane potential is most permeable to postassium ions?

Repolarization *picture is wrong because its -70

NRTI: require what to be active?

Requires Nucleoside phosophorylation

Internal bleeding is suspected after a cardiac catheterization in the femoral artery which is penetrated above the inguinal ligament. Where is the blood collection?

Rertoperitoneal Hemorrhage

Fetus has bilaterally enlarged fetal kidneys with diffuse small cysts. what is most likely to be present in the newborn after delivery (respiratory distress or vertebral anomalies)?

Respiratory Distress *Pulmonary Hypoplasia results from decreased amniotic fluid.

PAINFUL Erythematous vesicular lesion?

Shingles (Herpes Zoster) (Varicella Zoster Virus)

Osteosarcoma is associated with loss of what tumor suppressor gene?

Retinoblastoma (RB1)

Clumping of Nuclear chromatin: Reversible or Irreversible injury?

Reversible Cell Injury

Decreased Glycogen or Triglyceride droplet accumulation: Reversible or Irreversible injury?

Reversible Cell Injury

Disaggregation of Polysomes: Reversible or Irreversible injury?

Reversible Cell Injury

Disaggregation of nuclear granules: Reversible or Irreversible injury?

Reversible Cell Injury

Dissociation of ribosomes from RER: Reversible or Irreversible injury?

Reversible Cell Injury

Mitochondrial Swelling: Reversible or Irreversible injury?

Reversible Cell Injury

Myofibril relaxation: Reversible or Irreversible injury?

Reversible Cell Injury *early sign in cardiac myocytes (usually within first 30 minutes of sever ischemia)

What causes 99% of all Mitral Stenosis cases?

Rheumatic Fever

HLA Class II Genotypes include?

Rheumatoid Arthritis Type I DM Celia Disease

Organism has a mutated RNA polymerase: resistance to what class of antibiotics?

Rifamycins

STEMI in the Inferior leads are most often due to blockade of what artery?

Right Coronary Artery (RCA) *Inferior MI

Left Nasal Hemiretina damage: what geniculate body is affected?

Right Lateral Geniculate Body

Right Temporal Hemiretina damage: what geniculate body is affected?

Right Lateral Geniculate Body

Antigens that often cause Acute Serum Sickness?

Rituximab Infliximab Venom Antitoxins Penicillin Cefaclor Trimethoprim-Sulfa *big key is watch for a reaction 7-14 days after starting theses!!!

Nissl substance in neurons corresponds to what?

Rough Endoplasmic Reticulum

What Female Pelvic Ligament will be found in the Inguinal Canal?

Round Ligament

What phase of the cell cycle does DNA duplication occur?

S Phase

Antimetabolites would affect what phase of the cell cycle?

S Phase *The DNA Duplication phase

Influenza Virus: patients are at risk for a bacterial superinfection with what?

S. Aureus S. Pneumoniae H. Influenzae *elderly particularly at risk. *watch for signs of worsening symptoms (progressive dyspnea, chest pain, productive cough, recurrent fever, pulmonary consolidation)

Hip extension, knee flexion, foot plantar flexion, achilles reflex are lost. Sensation in buttocks, posterior thigh, calf, and lateral foot are lost: what nerve root?

S1

Pain in lower back and radiates down the right posterior thigh to the foot. Pain as "shooting". Straight leg raise testing is positive on the right. Right hip extension is weaker when compared to the left. Knee jerk reflexes are 2+ and bilaterally symmetric. But right ankle jerk reflex is absent. What nerve root is compressed?

S1

THIRD-DEGREE AV Block: what is going on with the impulses

SA node impulses cause atrial contraction while impulses generated by the AV node cause ventricular contraction. The Atria and the Ventricles depolarize independently of each other -> AV dissociation.

Retroperitoneal Abdominal organs?

SAD PUCKER Suprarenal (Adrenal) Glands Aorta & IVC Duodenum (2nd - 4th parts) Pancreas (Head & Body) Ureters & Bladder Colon (ascending & descending) Kidneys Esophagus Rectum (mid-distal)

Li-Fraumeni Syndrome: what all is included in this?

SARCOMAS BREAST CANCER ADRENOCORTICAL CARCINOMA LEUKEMIA BRAIN tumors

Hypertensive Emergency: criteria?

SBP > 180 DBP > 120 *END-ORGAN DAMAGE

Hypertensive Urgency: criteria?

SBP > 180 DBP > 120 *NO evidence of end-organ damage

Standard Error equation?

SD/(square root N) N=sample size

Diabetic, high blood pressure, sedentary lifestyle, smoker. What has the GREATEST EFFECT on REDUCING this patient's MORTALITY RISK?

SMOKING CESSATION

Bulbus Cordis is an embryological derivative of what?

SMOOTH parts of VENTRICLES (both left and right)

Post Traumatic Stress Disorder (PTSD): treatment?

SSRI SNRI CBT

Generalized anxiety Disorder (GAD): treatment?

SSRI (Fluoxetine, Paroxetine, Sertraline, Citalopram) SNRI

In a prolactinoma what does this do to GnRH?

SUPPRESS GnRH

Green vaginal discharge: what is the best test to confirm the diagnosis?

Saline Microscopy *Trichomonas Vaginalis

Carcinomas or Sarcomas spread hematogenously?

Sarcomas

1 year old boy with Dactylitis: diagnosis?

Sickle Cell Anemia

6 yo african american complains his "bones hurt." He has been hospitalized with "chest infection" and abdominal pain. And mom says he runs out of breath quickly: diagnosis?

Sickle Cell Anemia

15 with right arm numbness. member of high school baseball team. Fluctuating tingling and numbness involving right shoulder, arm, and hand. Dull pain in right little finger and hand. Symptoms worsen with overhead activities when throwing a baseball. Decreased sensation over medial 2 fingers and hypothenar eminence of the right hand. What structure is contributing to this condition?

Scalene muscles *Thoracic Outlet Syndrome *presents with upper extremity numbness, tingling, weakness. *Compression of subclavian vein can cause Upper Extremity SWELLING. *Compression of Subclavian Artery can cause exertional ARM PAIN

Praziquantel: use for?

Schistosoma Clonorchis Sinensis (Clonorchiasis) Paragonimus Westermani (Paragonimiasis)

Prefers to be a loner, detached, unemotional and is content this way?

Schizoid Personality Disorder

Cranial mass with elongated cells that have regular, oval nuclei. A biphasic pattern of growth is identified, consisting of areas of dense cellularity interspersed with less dense myxoid regions. Cells show S-100 immunoreactivity. Diagnosis?

Schwannoma *S-100 (only brain tumor that is S-100 positive) that is because it is NEURAL CREST CELL ORIGIN!)

Muscarinic Acetylcholine Receptors antagonist: medication name?

Scopolamine

The immortalized B cells maintain the ability to do what? (from an EBV infection)

Secrete Immunoglobulins & Secrete B-call activation products (CD23)

Villous Adenomas may cause what symptoms?

Secretory diarrhea Bleeding Intestinal Obstruction

Endometrial Sampling reveals coiled glands filled with carbohydrate-rich mucus edematous stroma, and tortuous spiral arteries: when did this biopsy occur?

Secretory phase (days 15-28) *during this time progesterone promotes development of secretory endometrium *UTERINE GLANDS COIL & SECRETE MUCUS (prepares for embryo implantation) *becomes EDEMATOUS *becomes COMPLETELY TRAVERSED by TORTUOUS SPIRAL ARTERIES

Maxillary sinuses what is the path of drainage into this sinus?

Semilunar Hiatus in the Middle Nasal Meatus

Aspergillus: what is seen on light microscopy?

Septate narrow hyphae with sharp angle branching

Raphe Nuclei: release what neurons?

Serotonin R is close to S

Patient is on metformin, lisinopril, paroxetine is treated for a cellulitis. 3 days later patient develops hyperthermia, hypertension, tachycardia. She is confused. She has bilateral hyperreflexia, and ankle clonus. And she begins to have seizures. What is going on?

Serotonin Syndrome *Paroxetine (SSRI)

Hyperreflexia, Clonus, myoclonus, rigidity, tremor, hyperthermia, tachycardia, diaphoresis, vomiting/diarrhea, altered mental status. Patient recently started a new depression medication: what is this?

Serotonin syndrome

Young person with acute pancreatitis and the cause is not alcohol or gallstones: what test should you consider?

Serum Triglycerides *if Serum Triglyceride levels rise >1000 the pancreatic acinar cells may be damaged.

Xanthelasmas (type of XANTHOMA): what is the next best step in management

Serum lipids & Blood Glucose *insulin resistance promotes increased VLDL production (diabetics may develop a secondary Type IV or V hyperlipidemia with increased VLDL, chylomicrons)

Young infant or child with Pneumocystis Jirovecii: Diagnosis?

Severe Combined Immune Deficiency

Young infant or child with recurrent viral or bacterial infections, thrush, chronic diarrhea?

Severe Combined Immune Deficiency

Child with chronic diarrhea, failure to thrive and low blood T Lymphocyte count and severely decreased serum immunoglobulin level: diagnosis?

Severed Combined Immunodeficiency

Albinism: increased risk of what?

Skin Cancer

Lamotrigine: patient should tell the physician immediately if what symptom occurs?

Skin rash *Stevens-Johnson Syndrome!!

Syndrome of Inappropriate Antidiuretic Hormone (SIADH): what type of lung tumor causes this?

Small Cell Carcinoma of Lung

Lambert-Eaton Myasthenic Syndrome (LEMS): associated cancer?

Small Cell Lung Cancer *1/2 of all LEMS patients have malignancy

Chronic Rejection in a lung transplant patient. What would primarily be damaged?

Small airways (SMALL BRONCHIOLES) *Bronchiolitis Obliterans (inflammation and fibrosis of bronchiolar walls lead to narrowing and obstruction)

Primary Spontaneous Pneumothorax: most important risk factor?

Smoking *taller individuals also are at higher risk (due to more negative intrapleural pressure in lung apices)

Calmodulin is an element of what type of muscle?

Smooth Muscle

Myosin Light-chain Kinase

Smooth Muscle

Right Horn of Sinus Venosus is an embryological derivative of what?

Smooth part of Right atrium (Sinus Venarum)

Primitive Pulmonary Vein?

Smooth part of left atrium

Glucose clearance best approximates the clearance of what?

Sodium Urea (reabsorbed at the PCT and inner medullary collecting ducts) *normally filtered at the glomerulus and completely reabsorbed by the proximal tubule

Sodium blocking effects of Class 1C Antiarrythmics are USE DEPENT: what does that mean?

Sodium-blocking effects ITENSIFY as the HEART RATE INCREASES

McCune-Albright syndrome: example of what genetic effect?

Somatic Mosaicism *Mutation arises from mitotic errors after fertilization and propagates through multiple tissues or organs.

Mutation arises from mitotic errors after fertilization and propagates through multiple tissues or organs.

Somatic Mosaicism *NO CHILDREN affected

Severe, well-localized pain

Somatic Pain

Well localized, more severe, and worsened with deep inspiration or pushing on the abdominal wall?

Somatic Pain

Inflammation of the Parietal Peritoneum would cause what type of pain?

Somatic Pain *More severe localized pain that is sensitive to gentle pressure or sudden release of deep pressure (rebound tenderness

IGF-1: whats another name?

Somatomedin C

Medical treatment of choice for Acute VARICEAL BLEEDING?

Somatostatin (OCTREOTRIDE) *reduces splanchnic blood flow (by inhibiting the hormones responsible for splanchnic vasodilation) without causing systemic vasoconstriction.

If a patient has somatostatinoma, how could this cause gallstones?

Somatostatins decrease the secretion of CHOLECYSTOKININ *DECREASED CCK causes REDUCED GALLBLADDER CONTRACTILITY

Sorbitol Dehydrogenase: Function?

Sorbitol --> Fructose

In tissues that do not accumulate Sorbitol what allows these cells to do this?

Sorbitol Dehydrogenase

Alpha-Thalassemia more common in what area?

Southeast Asia

What blotting technique is used to identify and isolate proteins that bind DNA?

Southwestern Blot

c-Jun, c-Fos what blotting technique is used to identify?

Southwestern Blot

Labeled double-stranded DNA probes are used to detect specific protein: what is the blotting technique?

Southwestern Blot *DNA-binding protein (substance) *dsDNA (probe)

ELISA: tests for?

Specific Antibodies in patient serum

Morphine (Opioid) can cause vasoconstriction of smooth muscle where?

Sphincter of Oddi (leading to SPASM)

Progressive neurodegeneration, hepatosplenomegaly, macular cherry-red spot: what is the deficiency?

Sphingomyelinase *Niemann-Pick *Sphingomyelin accumulates

Chiari II Malformation is associated with what?

Spina bifida Cystica (Myelomeningocele)

Imaging studies reveal a fracture of the LEFT 10th RIB? What structure is most likely to be lacerated by the fractured bone?

Spleen *9 - 11 overlie the spleen

Patient has a pruritic rash of worsening severity. Were the rash is the patient has been applying a topical analgesic cream to her thighs and buttocks after her workouts. Rash is Erythematous rash with blisters, ulcers, and weeping drainage. What dermatologic microscopic term is most likely found on skin biopsy?

Spongiosus *Eczematous Dermatisis (ECZEMA) *Patient is having Acute Allergic Contact Dermatitis (ACD) a form of Eczema *group of conditions characterized by erythematous, papulovesicular, WEEPING, lesions.

Vast majority of cancers in the head and neck are of what type?

Squamous Cell Carcinomas

Standard Deviation, Variance and Mode: which of these are sensitive to outliers?

Standard Deviation Variance *these are sensitive to outliers because outliers increase the dispersion of datasets

Acute Bacterial Parotitis: organism?

Staphylococcus Aureus

Osteomyelitis: Most common cause?

Staphylococcus Aureus

Osteomyelitis: childhood age?

Staphylococcus Aureus

Protein A: organism?

Staphylococcus Aureus

Lineweaver-Burke Plot: what would happen with an increased concentration of enzyme?

Start: Same Y-intercept: DECREASED

Lineweaver-Burke Plot: what would happen in the presence of an reversible competitive inhibitor?

Start: moved to the right Y-intercept: SAME

Severe Combined Immune Deficiency: treatment?

Stem Cell Transplant

Clavicle Fracture: what pulls the medial part superiorly?

Sternocleidomastoid

25 yo man who is a personal trainer has facial acne, testes are soft, and volume is decreased. Hematocrit is 60%. What is the best explanation for these findings?

Steroid drug abuse *Erythrocytosis *Testicular atrophy *Acne *Gynecomastia

IL-2: function?

Stimulates growth and differentiation of T cells, B cells, NK cells, and Macrophages

Pepsinogen is secreted from what organ?

Stomach

Patient is given a neuromuscular blockade and their response is assesed by electrically stimulating a peripheral nerve 4 times in quick succesion and observing the muscular response. It initially shows a constant but diminished response and the next response 30 minutes later shows a constant fade: what drugs?

Succinylcholine (only one!)

Patient with Myasthenia Gravis that is on treatment with low-dose Pyridostigmine but is experiencing muscle weakness symptoms. Infusion of Edrophonium DOES NOT elicit any change in his symptoms: what is the best next step in management of this patient?

Stop Pyridostigmine temporarily *called CHOLINERGIC CRISIS (patient receives TOO MUCH Acetylcholinesterase inhibitor and has EXCESS ACETYLCHOLINE in the synaptic cleft. *neuromuscular junction has become insensitive to acetylcholine and Edrophonium infusion produces NO IMPROVEMENT IN SYMPTOMS!

Hemosiderin: function?

Stores IRON

Impetigo: organisms?

Straphylocococcus Aureus Streptococcus Pyogenes (GAS)

Human Papillomavirus (HPV): has a predilection for what type of epithelium to cause warts? 2. Areas of concern that this is found?

Stratified Squamous Epithelium 2. Anal Canal, Vagina, Cervix, TRUE VOCAL CORDS (under a lot of friction)

Gram Negative that is Optochin Sensitive and Bile Soluble

Streptococcus Pneumoniae

Most common cause of bacterial meningitis in adults of all ages?

Streptococcus Pneumoniae

Encapsulated bacteria vaccines

Streptococcus Pneumoniae H influenzae Type B Meningococcal

Normal Pressure Hydrocephalus: symptoms are caused by what?

Stretching of EFFERENT & AFFERENT FIBERS (CORONA RADIATA) *expanded ventricles place traction on these fibers

Rare complication of measles infection that occurs several years after apparent recovery from initial infection.

Subacute Sclerosing Panencephalitis

Man has intense rage when he recalls childhood episodes of physical abuse by his stepfather. He is a devoted father and a supportive coach. He recently joined a rugby club where he is a fierce competitor. What defense mechanism is this?

Sublimation *when a person channels unacceptable thoughts or impulses INTO SOCIALLY ACCEPTABLE BEHAVIOR

Leiomyoma: which ones are associated with anemia from heavy menstrual bleeding?

Submucosal/intracavitary Fibroids

Patient with headaches, chronic nasal discharge, atrophic nasal mucosa, and perforation of the nasal septum. What is the most likely cause of this patient's symptoms?

Substance Abuse *COCAINE abuser (intranasal)

Parkinson Disease: patient with refractory treatment to levodopa/carbidopa wants high-frequency stimulation? What brain structures could be stimulated?

Subthalamic Nucleus or Globus Pallidus Internus *High-frequency Stimuation INHIBITS FIRING of these nuclei and would result in TALAMO-CORTICAL DISINHIBITION with IMPROVED MOBILITY Nigrostriatal degeneration in Parkinson disease results in excessive EXCITATION of the Globus Pallidus INTERNUS by the SUBTHALAMIC NUCLEUS, this causes EXCESSIVE INHIBITION of the THALAMUS.

Difference between True Diverticulum and Pseudodiverticulum?

True Diverticulum has MUSCULARIS Pseudodiverticulum--> only Mucosa & Submucosa THROUGH the Muscularis (false diverticula)

What 2 enzymes degrade polypeptides?

Trypsin Pepsin

Serotonin: what is the precursor amino acid?

Tryptophan

Ductus Arteriosus: what is this?

shunts blood from the Pulmonary Artery into the Descending Aorta

Patient is given a neuromuscular blockade and their response is assesed by electrically stimulating a peripheral nerve 4 times in quick succesion and observing the muscular response. It always display a fading pattern: what drugs?

Tubocurarine Vecuronium Pancuronium Rocuronium Atracurium

Pineal Gland (Pinealoma): histology?

similar to germ cell tumors (germinoma) (eg, testicular seminoma)

Patient has a irregular perihilar mass. 22 lb weight loss over 4 months. Streaks of blood in sputum. What cytokine is responsible for this patient's muscle wasting?

Tumor Necrosis Factor-alpha (TNF-alpha) *is thought to mediated paraneoplastic cachexia in humans by suppressing APPETITE and INCREASING BASAL METABOLIC RATE

Osteogenesis Imperfecta (OI): what collagen type?

Type I Collagen (genes encoded proteins that combine to form this)

Lupus: pancytopenia is caused by?

Type II Hypersensitivity (Autoantibodies attack the RBC's and platelets) *autoimmune hemolysis (warm IgG antibodies) *identical to Idiopathic Thrombocytopenic Purpura

Power is related to what type of error?

Type II error (Beta)

Acute Serum Sickness: what type of hypersensitivity?

Type III (immune Complex)

Acute Allergic Contact Dermatitis (ACD) a form of Eczema is what type of hypersensitivity?

Type IV - Delayed

Human Papillomavirus (HPV): which types result in warts?

Types 6 & 11

Insulin is an anabolic hormone that acts via what type of what receptor?

Tyrosine Kinase

Alkaptonuria: what conversion pathway is deficient in this patient?

Tyrosine to Fumarate Tyrosine -> Homogentisate -> ... -->... -> Fumarate --> TCA cycle

Hereditary Orotic Aciduria: deficiency?

UMP synthase

Prostaglandins have what effects?

VASODILATORY effects Prostacyclin --> Vasodilatory Thromboxane A2 --> Vasoconstriction

QRS duration represents?

VENTRICULAR DEPOLARIZATION

What gene mutation is found in the majority of patients with SPORADIC Renal Cell Carcinoma?

VHL gene on Chromosome 3p

Extrinsic Pathway Factors (PT)?

VII

Intrinsic Pathway factors (PTT)?

VII IX XI XII

How does the QRS complex change during exercise?

slightly REDUCED during exercise

HER1 & HER2 gene mutation: activating mutation or loss of function mutation?

activating mutation

MYC gene mutation: activating mutation or loss of function mutation?

activating mutation

SIS gene mutation: activating mutation or loss of function mutation?

activating mutation

TGFA gene mutation: activating mutation or loss of function mutation?

activating mutation

KRAS gene mutation: activating mutation or loss of function mutation?

activating mutation *ONCOGENE

Etanercept treatment is considered: what test should be performed before beginning treatment?

Tuberculin Skin Test *Tumor Necrosis Factor-Alpha (TNF-alpha)

Silicosis is associated with what disease?

Tuberculosis

Dopaminergic pathway controls prolactin secretion?

Tuberoinfundibular Pathway

Hyperprolactinemia: is related to a dysfunction in what dopaminergic pathway?

Tuberoinfundibular Pathway *Galactorrhea

HLA Class II proteins?

alleles -DR -DP -DQ

How is skeletal muscle afterload related to the contraction velocity of skeletal muscle?

The maximum velocity is indirectly proportionate to the afterload attached to the fiber. (The LESS afterload, the GREATER the velocity)

Splcing out of Introns is an energy-dependent process facilitated by what?

small nuclear ribonucleoproteins (snRNPs)

Type II error (Beta)?

The probability of concluding there is no difference between groups when one truly exists.

Infant has feeding problems and vomiting. Severe Metabolic acidosis and increased amount of branched-chain alpha-ketoacids in blood and urine. What enzyme do you supplement with?

Thiamine (Vitamin B1) *Maple syrup urine disease

What do you always give Wernicke-Korsakoff syndrome patients when you infuse glucose?

Thiamine (Vitamine B1)

Wernicke-Korsakoff: deficiency of?

Thiamine (vitamine B1)

Woman taking Lithium develops involuntary movements, ataxia, and tremor. What drugs may have precipitated the ataxia?

Thiazide Diuretics ACE-I NSAIDS (not aspirin)

How does Thiazide Diuretics cause Lithium Toxicity?

Thiazides -> limit sodium reabsorption in Distal Tubule causing MILD VOLUME DEPLETION which stimulates PROXIMAL TUBULAR SODIUM/LITHIUM REABSORPTION which leads to lithium toxicity over time.

Type II DM: After discussing medication options, the patient agrees to add an oral antidiabetic medication that induces differentiation of preadipocytes into adipocytes and increases glucose transporter-r expression on the adipocyte cell membrane. What agent was chosen?

Thiazolidinedione -PioGLITAZONE -RosiGLITAZONE *PPARY-y agonists

What plaques are generally more unstable and more vulnerable to rupture?

Thin-cap Fibroatheromas

Single amino acid substitution (glutamine for arginine) near the protein C cleavage site in her coagulation factor V gene product. This patient is at risk for developing what?

Thromoboembolism *Factor V Leiden --> causes Hypercoagulability *mutation occurs near the Protein C cleavage site -> FACTOR V becomes RESISTANT to CLEAVAGE

Myasthenia Gravis: associated cancer?

Thymoma

31 yo complains of difficulty in chewing so much that he doesn't buy gum anymore. Also has diplopia when reading or watching television for long periods. No extremity weakness. This condition is associated with what neoplastic process?

Thymoma *Myasthenia Gravis *Fatigable chewing (common presenting sign)

Thionamides: inhibit what?

Thyroid Peroxidase

In pregnancy, DIC is mediated by what?

Tissue Factor (Thromboplastin)

Topotecan inhibits what?

Topoisomerase I *no E in the first 2 letters so this is I

Woman has excessive sweating and no Thyroid problems: ablation of what structure is most likely to help this patient?

Thoracic SYMPATHETIC Trunk *Sweating is a SYMPATHETIC function

Irinotecan inhibits what?

Topoisomerase I IrInotecan has I

Etoposoide inhibits what?

Topoisomerase II

Teniposide inhibits what?

Topoisomerase II E in the first 2 letters name is II

Ring-enhancing lesions in the parietal or fontal lobes: what organism?

Toxoplasma Gondii

Multiple Ring enhancing lesions in brain associated with HIV?

Toxoplasma gondii

Failure of partitioning represents what GI malformation in Utero?

Tracheoesphageal Anomalies

Transfer of DNA from one bacterium to another by means of a bacteriophage: what is this called?

Transduction

DIRECT UPTAKE OF NAKED DNA from the environment by bacteria that are naturally competent: what is this called?

Transformation

Ligand binds to a tyrosine kinase receptor. What is the most likely IMMEDIATE EFFECT of ligand binding to this receptor?

Transient Dimerization *this just means the TWO RECEPTORS COME TOGETHER *SAME in non-receptor tyrosine kinase (search tyrosine kinase to see image)

Cytomegalovirus (CMV) Esophagitis can occur in what types of patients?

Transplant patients and immunocompromised

What part of the duodenum does the SMA cross?

Transverse portion of the duodenum

HER2/neu + breast cancer: treatment

Trastuzumab

Antifungal: inhibits the synthesis of Ergosterol?

Triazoles (any drugs with AZOLE)

Patient has polyuria and nocturia. Urine output and osmolality remain unchanged with water deprivation for several hours. After administration of desmopressin, urine output decreases and urine osmolality increases. Renal clearance of what substance would decrease the most after DESMOPRESSIN INJECTION?

UREA *vasopressin and desmopressin cause a V2 receptor-mediated increase in WATER & UREA PERMEABILITY at the INNER MEDULLARY COLLECTING DUCT. The resulting rise in UREA REABSORPTION (DECREASED UREA CLEARANCE) enhances the medullary osmotic gradient, allowing the production of maximally concentrated urine.

Since Class 1C bind to Sodium Channels MORE avidly. What does this effect cause?

USE DEPENDENCE -because of slow dissociation -blocking effects accumulate over multiple cardiac cycles

Trace the path of the Round Ligament?

UTERUS through the INGUINAL CANAL into the LABIA MAJORA

A 42 yo woman comes to the neurologist for enrollment in a study. She has all the signs of parkinsons disease. The results show a loss-of-function mutation in a gene that leads to an accumulation of misfolded proteins. Which of the following biochemical processes is most likely defective in this patient?

Ubiquitination *Ubiquitin labels proteins for degradation. The PROTEASOMES then recognize these and uses ATP to drive them through its tubular structure, DEGRADING them into SMALL PEPTIDES in the processs.

What plays an important role in disposal of misfolded proteins?

Ubiquitination degradation in PROTEASOMES

Fetal Circulation from placenta to the Right atrium?

Umbilical Vein -> Ductus Venosus -> Inferior Vena Cava (IVC)

Pigmented Gallstones arise from conditions that increase what in the bile?

Unconjugated Bilirubin *this promotes Calcium Bilirubinate precipitation

High incidence of fulminant hepatitis in pregnant women, who experience a mortality rate of 20%. Characteristics of the virus responsible?

Unenveloped ssRNA virus *Hepatitis E (HEV) *most concerning feature --> HIGH MORTALITY in pregnant women!!

Young child has elevated Arginine, the deficient enzyme in this patient is normally involved in the production of what?

Urea *Arginase deficiency *Arginine --> Ornithine (Urea is a Byproduct)

14 mo body with failure to thrive, developmental delay. At 12 months could barely lift head. Urine specimen contains large amounts of Orotic acid crystals. MCV 114. Hb 8.6. Normal ammonia levels in plasma. Supplementation with what substance would benefit this patient?

Uridine *Hereditary Orotic Aciduria *URIDINE will bypass mutated enzyme

35 yo, progressive fatigue, shortness of breath, bilateral leg swelling and abdominal distention. Weight Loss. Endocardial thickening with dense fibrous deposits around tricuspid and pulmonary valves. What should we measure to confirm a diagnosis?

Urinary 5-Hyroxyindoleacetic Acid *Carcinoid Syndrome *affects RIGHT-sided valves (Tricuspid Regurgitation, and Pulmonary Stenosis)

Nephrotic Syndrome: what causes the hypercoaguable state?

Urinary loss of ANTITHROMBIN III

Porphyria Cutanea Tarda: what accumulates

Uroporphyrin

Porphyria Cutanea Tarda: deficient enzyme?

Uroporphyrinogen Decarboxylase

Porphyria Cutanea Tarda: enzyme affected?

Uroporphyrinogen decarboxylase

Majority of bladder carcinomas are what type?

Urothelial (transitional cell) Carcinomas

How can you prevent the "Red Man" Syndrome?

Using Slower Rate of Infusion

If you grow Haemophilus Influenza on sheep blood agar and cross-streak the medium with S. Aureus. H. Influenza colonies will grow around the streaks because S. Aureus secretes what?

V Factor (NAD+)

What foramina does the 3 branches of the Trigeminal Nerve exit the skull?

V1 -> Superior Orbital Fissure V2 -> Foramen Rotundom V3 -> Foramen Ovale

Jaw deviates to the right side when the patient is instructed to open his mouth. What nerve is injured?

V3 (Trigeminal Nerve)

Sickle Cell Anemia: mutation?

VALINE substitution for GLUTAMIC ACID

When the endometrium is no longer exposed to PROGESTERONE during the menstrual cycle: what happens to the spiral arteries?

VASOCONSTRICTION OF SPIRAL ARTERIES (due to prostaglandin production increase) -this leads to endometrial ischemia and reperfusion injury

Multiple Lacunar Infarcts: characteristic of what dementia?

Vascular Dementia

Chronic Rejection in a Renal transplant patient. What would primarily be damaged?

Vascular Obliteration *Chronic Graft Nephropathy (VASCULAR FIBROSIS)

Is CHLORIDE content in RBC higher in venous blood or arterial blood?

Venous Blood "chloride shift" to maintain electrical neutrality

Varicose Vein: what is a very common complication?

Venous Stasis Ulcers -often occur over the Medial Malleolus

Pulmonary Embolism: what is the most likely cause of this patient's hypoxemia (Diffusion Impairment OR Ventilation/perfusion mismatch?

Ventilation/perfusion mismatch *think of Ventilation/perfusion mismatch as PERFUSION (BLOOD) not matching the Ventilation so it causes Hypoxemia

Multiple burns on hands. Cannot feel objects that are really hot and does not notice she has burns on her hands until she looks at her hands. Diminished pinprick and temperature sensation across the upper back, shoulders, and arms. Light touch, position, and vibration sense are preserved in upper extremities. Lower extremities exam is unremarkable: Where is the lesion?

Ventral White Commissure *Syringomyelia

QRS wave?

Ventricular DEPOLARIZATION

T wave?

Ventricular REPOLARIZATION

Oxygen saturation in cardiac chambers is taken in a young child and it shows an increase in Right Ventricle and Right Atrium. The Right Ventricle is slightly larger than the increase in Right Atrium. What accounts for this.

Ventricular Septal Defect (VSD) *Left-to-Right Shunt

Patient has a genetic defect that causes slow decreased outward potassium flow and resultant prolongation of the action potential. What is the most likely consequence of this patient's disease?

Ventricular Tachycardia and Sudden Death

Patient has persistent food-seeking behavior and weight gain. What hypothalamus nuclei may be involved?

Ventromedial Nucleus *If you zap your ventromedial area, you grow ventrally and medially

E. Coli Bloody Gastroenteritis: Virulence Factor?

Verotoxin (Shiga-like Toxin) *inactivates the 60S ribosomal component, halting protein synthesis and causing cell death

What should be suspected in patients with new or worsening back pain, fever and recent bacteremia or endocarditis.

Vertebral Osteomyelitis

What do you suspect in a patient with recent bactermia or endocarditis that has NEW NEUROLOGIC FINDINGS and fever?

Vertebral Osteomyelitis *diagnose with MRI

The minimal infectious dose for Vibrio Cholerae infection is usually what?

Very High

What is a predisposing factor for recurring pyelonephritis?

Vesicoureteral urine reflux *Anatomic or functional VUR is almost always necessary for development of acute pyelonepheritis

SINGLE episode of vertigo that CAN LASTS, severe vertigo but no hear loss: Diagnosis

Vestibular Neuritis

What is a gram negative rod that grows well on Highly Alkaline selective media?

Vibrio Cholerae

2 Drugs that inhibit microtubule formation and prevent synthesis of the mitotic spindle in dividing cells?

Vincristine Vinblastine both are Vinca Alkaloids *without mitotic spindle, replicated chromosomes are unable to align and segregate into their respective daughter cells during mitosis. This leads to failure of division and cell death. These are M-PHASE specific agents.

Viral vs Bacterial meningitis: WBC count and type of cell predominant?

Viral *WBC < 500 *LYMPHOCYTE predominant Bacterial *WBC > 1000 *NEUTROPHIL predominant

Viral vs Bacterial meningitis: glucose & protein levels?

Viral *normal glucose (like >50-60 *Protein < 150 Bacterial *Glucose < 45 mg /dL *Protein > 250 mg/dL

Viral vs Bacterial meningitis: stain and culture?

Viral: no organisms identified Bacterial: often positive for specific organism

Why are patients infected with Influenza virus susceptible to bacterial superinfections?

Virally induced damaged to the mucociliary clearance mechanism of respiratory epithelium

25 yo female reports to ED w/ difficulty breathing. Has had occasional chest tightness since she moved to a new house in the countryside two months ago. Prolonged expiration and scattered expiratory wheezes bilaterally. 20 minutes later treatment is begun and patient experiences significant relief. Intracellular increased in what most likely is responsible for the symptom relief?

cAMP (Beta-2 -> Gs receptor) *Allergic Asthma (most common type of asthma *inhaled Beta-2 agonists (like ALBUTEROL) can improve FEV! or peak expiratory flow rates within 5 MINUTES

A scientist finds an viral HIV protein that becomes glycosylated before being proteolytically cleaved into 2 smaller proteins in the endoplasmic reticulum and Golgi Apparatus. What is the function of these 2 newly formed HIV proteins?

Virion attachment to the target cells *the is the envelops (env) gene which codes for gp160 (this splits into gp41 and gp120)

Neuraminadase: function?

Virion particle release

Dull, non-localized pain

Visceral Pain

pain that is usually in the MIDLINE region and is POORLY localized and of a dull, constant, or cramping quality?

Visceral Pain

Measles Virus: deficiency of what vitamin is associated with a high rate of complications?

Vitamin A

Woman is particularly picky about eating and becomes pregnant. Her diet is mainly eggs, fruits, and vegetables. During her pregnancy which of the following is most likely UNNECESSARY to supplement with (Calcium, Folic Acid, Iron, Vitamin A, Vitamin B6)

Vitamin A *Excessive intake can pose a problem and is highly TERATOGENIC (spontaneous abortions, fetal defects) *Routine Vitamin A supplementation during pregnancy is not indicated unless in areas where deficiency and night blindness are problems

Cobalamin aka?

Vitamin B12

Spinal cord shows axonal degeneration involving posterior columns and lateral corticospinal tracts. What is the most likely condition?

Vitamin B12 Deficiency

Increased deposition of unmineralized osteoid is very characteristic of?

Vitamin D Def.

Osteomalacia: most common cause?

Vitamin D Deficiency *poor diet, inadequate sunlight, GI malabsorption

TB patient receiving treatment for Isoniazid and Rifampin. H has sensory ataxia and decreased pain sensation in distal extremities. Has numbness and tingling of hands. What is going on?

Vitamin Deficiency *Isoniazid is chemically similar to Vitamin B6 (Pyridoxine)

Friedreich Ataxia often mimics what vitamin deficiency?

Vitamin E Degeneration of the posterior columns and spinocerebellar tracts

Degeneration of the posterior columns and spinocerebellar tracts: what vitamin deficiencies?

Vitamin E Vitamin B12

Cystic Fibrosis teenager has chronic diarrhea, receives pancreatic enzyme therapy and a number of dietary supplements. PE shows decreased proprioception and hyporeflexia in lower extremities. Mild hemolytic anemia is shown in lab findings. What nutrient deficiency is most likely?

Vitamin E Deficiency *HEMOLYTIC ANEMIA *POSTERIOR COLUMN demyelination *SPINOCEREBELLAR TRACT demyelination -presents similar to Vitamin B12 but w/o Megaloblastic anemia, hypersegmented neutrophils, or Increased serum methylmalonic acid

LOSS OF MELANOCYTES and a COMPLETE ABSENCE of melanin pigment on histologic examination of the epidermis: diagnosis?

Vitiligo

Graves Disease patient develops fever and sore throat. What should you check?

WBC count with differential *Methimazole & PTU may cause Agranulocytosis

Aplastic Anemia presents with what spleen finding?

WITHOUT splenomegaly

Papillomavirus causes?

Warts

Typical symptoms of Anemia?

Weakness Fatigue Headache Irritability

Hernia that is MEDIAL to the Inferior Epigastric Vessels: what is the pathophysiology?

Weakness of Transversalis Fascia *Direct Inguinal Hernia

Femoral Hernia: pathophysiology?

Weakness of proximal femoral canal

Pioglitazone Therapy is initiated: what side effects may are most likely?

Weight gain and edema (Thiazolidinediones) *FLUID RETENTION *ADIPOSE WEIGHT GAIN

Clavicle Fracture: what pulls the lateral part inferiorly and anteriorly?

Weight of the arm and PECTORALIS MAJOR

Hemorrhage and necrosis are seen in mammillary bodies and gray matter surrounding the 3rd and 4th ventricles. Diagnosis?

Wernicke-Korsakoff syndrome *Vitamine B1 (Thiamine) Deficiency

Labeled Antibody are used to detect protein: what is the blotting technique?

Western Blot

Elfin facies, intellectual disability, well-developed verbal skills, extreme friendliness with strangers?

Williams Syndrome

Ulnar Nerve Injury: Weakness of what motor functions?

Wrist Flexion/ADDUCTION Finger Abduction/ADDUCTION 4th & 5th Finger Flexion

Infant with recurrent infection is diagnosed with a intracellular messaging abnormality. Abnormal gene codes for a cytoplasmic TYROSINE KINASE. Normally this tyrosine Kinase is activated in CD19-positive cells by antigen exposure, resulting in cell maturation. The abnormal gene responsible for this condition is on what chromosome?

X Chromosome *Bruton Agammaglobulinemia (X-linke agammaglobulinemia) *CD19 positive cells are the B Cells (which are Absent)

Haemophilus Influenza needs what to 2 factors to support growth?

X factor (HEMATIN) V Factor (NAD+)

Workup shows very low serum levels of all immunoglobulin types: diagnosis?

X-linked agammaglobulinemia (XLA)

A yellowish macule/papules found on the medial eyelid?

Xanthelasmas (type of XANTHOMA)

Actin Filaments are bound to structural proteins where?

Z line

Sarcomere is defined as?

Z line to Z line

Pseudodiveriticulum?

Zenker's Esophageal Diverticulosis

A CONSTANT AMOUNT of the drug is metabolized?

Zero-Order drug metabolism

Osteoprotegerin (OPG): function?

a physiologic decoy receptor that DECREASES BINDING OF RANK-L to RANK *reduces the differentiation and DECREASES THE SURVIVAL OF OSTEOCLASTS

56 yo patient started Lisinopril 2 months ago for hypertension. She has moderate swelling of her lips and tongue. Mild audible stridor without wheezing. What is the MOST LIKELY mechanism. responsible for this patient's symptoms? a. Bradykinin accumulation b. Hereditary C1-esterase inhibitor deficiency

a. Bradykinin accumulation *ANGIOEDEMA (most commonly affects tongue, lips, or eyelids) *BRADYKININ ACCUMULATION is the MOST LIKELY MECHANISM because C1-esterase inhibitor deficiency more likely to present in childhood and early adolescence. (Both cause these symptoms though!!)

Patient has blood in urine and proteinuria. Light microscopy reveals cellular proliferation, focal necrosis and crescent formation of most glomeruli. IF microscopy shows immunoglobulin and complement deposits. Which of the following additional findings is most likely to be present in this patient? a. Serum antiglomerular basement membrane antibodies b. Serum antineutrophil cytoplasmic antibodies

a. Serum antiglomerular basement membrane antibodies (LINEAR IF) *GOODPASTURE SYNDROME (Type II Hypersensitivity) *antibodies to the GBM and alveolar basement membrane) *Rapidly Progressive Glomerulopnehpritis (RPGN)

Hemoglobin F is composed of what chains?

a2y2 (2 alpha & 2 gamma chains)

Normal HbF contains what two globin chains?

a2y2 (alpha2gamma2)

Cystic Fibrosis patients: the most common mutation causes what?

abnormal posttranslational processing of a transmembrane protein *this is important because the protein doesn't even get to the membrane (some answers may say IMPAIRED ion conduction through a transmembrane protein which is true in other types of CF patients but NOT IN THE MOST COMMON TYPE!!

TNF-alpha produced by?

activated marcrophages

ABL gene mutation: activating mutation or loss of function mutation?

activating mutation

BRAF gene mutation: activating mutation or loss of function mutation?

activating mutation

Patient is on medication for HTN, stable angina pectoris, gout, and BOH. His BP is 120/80 and when he stands up it drops to 90/60. Blockade of which receptor is most likely contribution to this patient's current condition? (alpha1-adrenergic, alpha2-adrenergic, Beta1-adrenergic, Beta2-adrenergic)

alpha1-adrenergic -standing causes POOLINg of a significan amount of blood (500-1000 mL) into the veins below the heart. This initiates the following sequence of events in otherwise healthy individuals 1. Decreases venous return to heart 2. Decreased ventricular filling & drop in CO 3. Drop in BP that evokes compensator baroreceptor reflex 4. Increased sympathetic tone that increases peripheral vascular resistance (alpha1-adrenoreceptors) as well as Heart rate and myocardial contractility (Beta1-adrenoreceptors) *alpha1-adrenergic Antognists (TERAZOSIN, DOXAZOSIN) used for BPH often are associated with orthostatic hypotension resulting in LIGHTHEADEDNESS & SYNCOPE

Xanthelasmas (type of XANTHOMA): what is the most common abnormality that causes these?

an LDL receptor abnormality

Celiac Disease: screening with what serology testing? 2. confirmed with?

anti-tissue Transglutaminase antibodies IgA anti-endomysial 2. Biopsy

HIV drug that is an inhibitor of HIV protease that prevents assembly and maturation of the virus.

anything w/ NAVIR is a PROTEASE INHIBITOR -Ritonavir -Saquinavir -many other NAVIR *NAVIR TEASE a PROTEASE

An agarose gel electrophoresis of DNA extracted from the control cells shows fragments in multiples of 180 base pairs; the DNA from the patient cells is unfragmented. What does this mean?

appearance of DNA fragments of 180 base pairs is called DNA LADDERING. This finding is a sensitive indicator of APOPTOSIS *question may include something about this and refer to something about apoptosis

Meningococcal Pili (Fimbriae) are important for what?

attachment to epithelial surfaces (especially the Nasopharynx)

HIV: gp120?

attachment to host CD4+ T cell

Myasthenia Gravis: autoantibodies?

autoantibodies to POSTSYNAPTIC ACETYLCHOLINE RECPTOR

Lambert-Eaton Myasthenic Syndrome (LEMS): autoantibodies?

autoantibodies to PRESYNAPTIC CALCIUM CHANNEL --> DECREASE ACETYLCHOLINE release

34 weeks gestation woman admitted to the hospital because of nausea and vomiting and abdominal pain for 12 hours. Pregnancy otherwise unremarkable. Lab studies are negative for HIV and hepatitis. Blood pressure is 164/102. She has right upper quadrant tenderness, Hb of 7.4 g/dL, 24% Hematocrit, 72,000 Platelet count, AST 112, and ALT 126. Examination of a peripheral blood smear is most likely to show a predominance of which of the following in this patient? a. Acanthocytes b. Schistocytes c. Spherocytes d. Target Cells

b. Schistocytes Hemolysis, Elevated Liver enzymes, low Platelets HELP Syndrome

Patient has blood in urine and proteinuria. Light microscopy reveals cellular proliferation, focal necrosis and crescent formation of most glomeruli. IF microscopy shows NO immunoglobulin or complement deposits. Which of the following additional findings is most likely to be present in this patient? a. Serum antiglomerular basement membrane antibodies b. Serum antineutrophil cytoplasmic antibodies

b. Serum antineutrophil cytoplasmic antibodies PAUCI-IMMUNE Rapidly Progressive Glomerulopnehpritis (RPGN) -Granulomatosis with Polyangiitis (Wegener) -Microscopic Polyangiitis *most such patients have ANCA (antineutrophil cytoplasmic antibodies) in serum *Pauci-immune (means there are NO immunoglobulin or complement deposits on basement membrane)

Cardiac Tamponade: what is often seen in a very LARGE efffusion?

beat-to-beat variation in pulse amplitude (PULSUS ALTERNANS) *due to swinging movement of heart in LARGE effusion *important this is only in LARGE effusions

Biphosphonates: bind to?

binds HYDROXYAPATITE in bone, inhibiting osteoclast activity

Protein A: function?

binds the Fc portion of IgG antibodies at the COMPLEMENT-BINDING SITE preventing complement activation

Histone H1: function?

binds the LINKER segments of DNA that lie between nucleosomes and facilitate PACKAGING OF NUCLEOSOMES into more compact structures

NRTI: MOA?

binds to reverse transcriptase and is incorporated into the viral genome as a thymidine analog. Zidovudine has an azido group in place of hydroxyl group which is normally found on the 3' end of thymidine.

Pneumococcal CONJUGATE vaccines are strongly imunogenic in INFANTS: what?

both B & T cell recruitment *provide higher, longer-lasting antibody titers relatie to pneumococcal polysacchardie vaccines.

Thionamides: MOA?

decrease formation of thyroid hormones via INHIBITION OF THYROID PEROXIDASE which is responsible for the following 1. Iodine Organification 2. Coupling of iodotyrosines

Leukemoid Reaction: explain?

defined as a BENIGN LEUKOCYTOSIS > 50,000 cells that occurs in response to underlying condition like SEVERE INFECTION/HEMORRHAGE, MALIGNANCY (LEUKEMIA) ACUTE HEMOLYSIS.

Human Papillomavirus (HPV): what type of virus?

dsDNA & circular

Why is the Pneumococcal Polysaccharide vaccine poorly immunogenic in INFANTS?

due to their relatively IMMATURE HUMORAL ANTIBODY RESPONSE

Uric Acid Stones are a risk in what conditions?

during chemotherapy for LYMPHOMAS or LEUKEMIAS, and other tumors that have high cell turnover *TUMOR LYSIS SYNDROME (oncologic emergency)

Pseudohypoparathyroidism: caused by?

end-organ RESISTANCE to PARATHYROID HORMONE (PTH) due to defects in the PTH receptor and downstream signaling pathways.

HIV: Only the polyprotein product of what gene is glycosylated?

env

Hemochromatosis: initiating cause?

excessive GI ABSORPTION OF IRON

Hamartoma: definition

excessive growth of a tissue type NATIVE TO THE ORGAN OF INVOLVEMENT

Hydroxyurea is reserved for patients with what?

frequent pain crises.

HIV: gp41?

fusion and entry

In HIV replication cycle, polyprotein precursors are encoded by what structural genes?

gag pol env

HbF contains high quantities of what globin?

gamma-globin (y-globin)

Insomnia treatment: stimulus control therapy?

goal is to dissociate the bedroom from any stimulating activities that do not involve sleep or sex -reading -television -eating *if patient is unable to fall asleep within 20 minutes leave the room

The HIV env gene becomes what?

gp 160

HIV: gp160 is cleaved to become what?

gp120 gp41

Choriocarcinoma: what serum marker is the most useful for monitoring this patient?

hCG

RNA viruses replicate where?

in the CYTOPLASM

Pyrimethamine: MOA?

inhibit dihydrofolate reductase *similar to Trimethoprim (TMP)

von Willebrand disease (vWD): why is PTT increased?

it is a carrier protein for FACTOR VIII which is in the Intrinsic Pathway (PTT)

Haptoglobin function?

serum protein that binds to free hemoglobin and promotes its uptake by the reticuloendothelial system *DECREASES when significant quantities of hemoglobin are released into circulation

Lactase intolerance biopsy: what would it show?

normal small intestinal architecture *KEY IS THAT ITS NOT INDICATED FOR DIAGNOSIS!!!

c-Fos

nuclear transcription factor

c-Jun

nuclear transcription factor

The most common pathologic cause of UNILATERAL fetal HYDRONEPHROSIS is what?

obstruction at *Uteropelvic Junction (UPJ)* *this will either be found during ultrasound or as a NEWBORN --> w/ a PALPABLE abdominal mass (reflecting an enlarged kidney)

Diabetic Ketoacidosis: pH, HCO3-, H2PO4-?

pH: DECREASED HCO3-: DECREASED H2PO4-: INCREASED

In Metabolic Acidosis what will the urine chemistry pattern demonstrate?

pH: DECREASED HCO3-: DECREASED H2PO4-: INCREASED *Urinary pH decreased due to increased excretion of free H+, NH4+, H2PO4-

Acute Respiratory Acidosis values?

pH: Low pCO2: HIGH HCO3: NORMAL/MILDLY INCREASED *Renal compensation requires at least 24 hours of persistent respiratory acidosis *example of this is HEROIN OVERDOSE

Chronic Respiratory Acidosis values?

pH: Low/Normal pCO2: HIGH HCO3: HIGH (>30) *Renal compensation has been able to keep up *example of this is COPD

ACE-I can be very detrimental for what patients?

patients with Renal Artery Stenosis *they depend on Efferent Arteriole Constriction to maintain Renal Perfusion

deoxynucleotidyltransferase (TdT): is a marker of what?

pre-T Cells pre-B Cells

Ovarian Torsion: main risk factor for torsion?

presence of a large OVARIAN MASS

Hypophosphorylation (Active) of Retinoblastoma (Rb)?

prevents damaged cells from proceeding past G1 to S checkpoint

Influenza: treatment that will minimize the duration of symptoms. The medication most likely acts by directly inhibiting what process?

prevents the RELEASE of newly formed VIRAL PARTICLES by INHIBITING NEURAMINIDASES *Oseltamivir (Tamiflu)

Ketogenic?

produce ACETOACETATE or its precursors

Glucogenic?

producing intermediates of the CITRIC ACID CYCLE or PYRUVATE

Hemagglutinin: function?

promotes viral entry

Porcelain gallbladder: treatment?

prophylactic cholecystectomy *due to high rates of gallbladder adenocarcinoma

RNA Polymerase I: function

rRNA synthesis

With respect to skeletal muscle what is preload?

refers to the INITIAL MUSCLE FIBER LENGTH PRIOR TO CONTRACTION.

QT duration?

represents Ventricular DEPLOARIZATION --> REPOLARIZATION

Patients with Chronic Hemolytic Anemia (like beta-thalassemia major) who receive chronic blood transfusions should also undergo?

routine IRON CHELATION THERAPY

Patient has increased bruisability and mucosal bleeding, oral bleeding after tooth extraction. Father and Paternal grandmother also had this. Diagnosis?

von Willebrand Disease (vWD)

Patient is being evaluated for worsening fatigue. Lab evaluation shows an elevated alkaline phosphatase. What should be checked next?

y-glutamyl transpeptidase (GGTP) *INCREASED in liver and biliary disease NOT in Bone disease *MOST HELPFUL test in followup of ALP


Kaugnay na mga set ng pag-aaral

Chapter 11-Introduction to Contracts

View Set

Chapter 29: Management of Patients with Nonmalignant Hematologic Disorders

View Set

Knewton Alta Lesson 6 Assignment

View Set

Unfair, Deceptive, or Abusive Acts or Practices (UDAAP)

View Set

HESI Neurocognitive Disorder (Advanced Alzheimer's Disease)

View Set

Microbiology The Diversity of Prokaryotic Organisms

View Set

Chapter 8 Diagrams from test bank, CH.8 human a and p, Chapter 7 Diagrams from test bank, Anatomy Test 3

View Set

Physical Geology Chapter 12: Earth's Interior

View Set

L9/C3: Role of an Agent in a Real Estate Transaction/Dual Agency

View Set